Jump to content

Wikipedia:Reference desk/Science: Difference between revisions

From Wikipedia, the free encyclopedia
Content deleted Content added
→‎Bismuth: new section
Line 936: Line 936:


= January 29 =
= January 29 =

== Bismuth ==

The article on bismuth says that it is more or less nontoxic. However, its only naturally occurring isotope decays into thallium, which is highly poisonous. Why isn't bismuth dangerous to handle as a result? --[[Special:Contributions/75.40.204.186|75.40.204.186]] ([[User talk:75.40.204.186|talk]]) 01:23, 29 January 2010 (UTC)

Revision as of 01:23, 29 January 2010

Welcome to the science section
of the Wikipedia reference desk.
Select a section:
Want a faster answer?

Main page: Help searching Wikipedia

   

How can I get my question answered?

  • Select the section of the desk that best fits the general topic of your question (see the navigation column to the right).
  • Post your question to only one section, providing a short header that gives the topic of your question.
  • Type '~~~~' (that is, four tilde characters) at the end – this signs and dates your contribution so we know who wrote what and when.
  • Don't post personal contact information – it will be removed. Any answers will be provided here.
  • Please be as specific as possible, and include all relevant context – the usefulness of answers may depend on the context.
  • Note:
    • We don't answer (and may remove) questions that require medical diagnosis or legal advice.
    • We don't answer requests for opinions, predictions or debate.
    • We don't do your homework for you, though we'll help you past the stuck point.
    • We don't conduct original research or provide a free source of ideas, but we'll help you find information you need.



How do I answer a question?

Main page: Wikipedia:Reference desk/Guidelines

  • The best answers address the question directly, and back up facts with wikilinks and links to sources. Do not edit others' comments and do not give any medical or legal advice.
See also:


January 24

Maps of Australia

G'day! I'm looking forward to a drive across the Outback later this year. I will have a GPS and a road atlas, but as far as I can make out, Australia has no national grid reference system like I have in the UK. Are the reference grids on their maps just arbitrary or are they consistent from one publisher to another? Do the road atlases give any indication of latitude and longitude?--Shantavira|feed me 10:49, 24 January 2010 (UTC)[reply]

Possibly. Quoting from page 3 of the 2008 "StreetSmart" Perth Street Directory (bold is added by me for emphasis) - which is Perth metro area, not the outback!
I am not a cartographer so I don't know if this is what you are looking for, but perhaps it will help. Mitch Ames (talk) 11:41, 24 January 2010 (UTC)[reply]
Thanks, that's interesting. Looks like we need an article on that system. My GPS supports UTM, but I guess that is different. Does anyone know if I might be able to download the Aussie system to my Garmin GPS? Their website is not very informative. Maybe I should just buy a new GPS in Oz?--Shantavira|feed me 12:34, 24 January 2010 (UTC)[reply]
Don't know about downloading, but a spare GPS, if you are going to rely on them a great deal, would be a very good idea. Are you for example planning to go off paved/marked roads? Our A$ is high at them moment so buying it here will be more expensive, though I have seen them for less than ≈$A150. 220.101.28.25 (talk) 13:41, 24 January 2010 (UTC)[reply]
Nah, probably only on paved roads, so the GPS is only for location rather than navigation. Your dollar is really low at the moment compared to the GBP [1]. I was amazed to discover that Google Street View now includes a lot of the Outback, including some dirt roads.--Shantavira|feed me 14:43, 24 January 2010 (UTC)[reply]
You're reading the chart backwards, that graph shows GBP is low against AUD. The pound is only buying AUD $1.70, one year ago it was much stronger buying AUD$2.20. Vespine (talk) 22:14, 24 January 2010 (UTC)[reply]
Oops, thank you! Someone told me it was low. Looks like I'm not going to have quite the luxury holiday I was expecting!--Shantavira|feed me 10:33, 25 January 2010 (UTC)[reply]
You have to watch those exchange rates! Petrol/fuel/gas isn't cheap here either, probably(?) cheaper then the UK. (In the Outback I imagine it will be VERY expensive, compared to city prices). Possibly add several Jerry cans to your list of necessities? --220.101.28.25 (talk) 18:33, 25 January 2010 (UTC)[reply]
Some Australian road maps (example (PDF)) include latitude and longtitude. Mitch Ames (talk) 06:51, 26 January 2010 (UTC)[reply]

Garmin GPSs are sold here, and your map provider should also have Australian maps - mine has UK/europe maps, so I don't see why the reverse wouldn't be true! Mattopaedia Have a yarn 02:18, 30 January 2010 (UTC)[reply]

Hypochondria paradox

If hypochondriacs have a tendency to self-diagnose themselves with many diseases, why do they fail to self-diagnose themselves with hypochondria, and thus realise that they actually probably don't have those diseases? Also, is this related to the Dunning-Kruger effect? --Mark PEA (talk) 14:52, 24 January 2010 (UTC)[reply]

Hypochondria is a tendency to channel anxiety into worries about one's own body health. Perceived evidence of disease is given more weight than evidence of health or reassurance from other sources of health information. I do not consider hypochondria very closely related to the Dunning-Kruger effect. alteripse (talk) 15:16, 24 January 2010 (UTC)[reply]
Hypochondria isn't deadly or debilitating. Hypochondriacs have more to worry about than hypochondria. Even if they do diagnose themselves with it, it's not important enough to bother a doctor about when they've got so many other things that could be far more harmful. Vimescarrot (talk) 15:46, 24 January 2010 (UTC)[reply]
This is the difference between psychology and physiology. Vranak (talk) 22:55, 24 January 2010 (UTC)[reply]

live jasmin

how can i block live jasmin.com from my computer —Preceding unsigned comment added by 67.246.254.35 (talk) 14:58, 24 January 2010 (UTC)[reply]

  1. This question really belongs on the Computing ref desk...and they'll also want to know...
  2. You need to tell us what operating system you use and what browser/email client(s) you use.
  3. Do you mean to block email from that domain? Or prevent people who use your computer from browsing that web site? What precisely?
SteveBaker (talk) 16:50, 24 January 2010 (UTC)[reply]
nb: livejasmine.com is an adult webcam site that pops up (I guess from ad links or partnership links) when browsing many porn sites. I guess the original-poster wants to be able to block these popups. I don't know how to do that, but running Firefox's "flashblock" extension means that the popup window doesn't have a live webcam feed. Googling for "livejasmine popup block" finds lots of threads about how to block the popup itself. 87.114.29.120 (talk) 16:57, 24 January 2010 (UTC)[reply]
Presuming the popups need to access livejasmine.com and don't use an IP or have different domains that they continually vary, a simple (and common) solution that will work in most OSes (albeit you'll need to find out how to do it in each) would be to put livejasmine.com in the hosts file and point it to 127.0.0.1 or something similar. This will not block the popups windows from opening, it will simple mean you'll get an webserver doesn't respond message (unless you actually have a webserver locally or wherever you point it to) and as I've said will not help if the websites start using popups from an IP or they cycle between domains. And it's obviously not an effective way of stopping all software from accessing the domain, they could query a hardcoded domain name server or query the defined DNS themselves bypassing the OS DNS subsystem (if the DNS is a router you control you may be able to do something similar on the router) but it should work for browsers. Nil Einne (talk) 04:27, 25 January 2010 (UTC)[reply]

Expression of ratios

The title makes this sound like a mathematics question. I have never been able to understand an answer - or, to be fair, most of the questions- on the mathematics desk, however, so I am hoping someone with a science background will have the same information. (I sometimes understand science answers.)

I had always understood that ratios are expressed using a ":" in the form "A:B", and represent a mathematical relatipnship between A and B. In the article Vaccination controversy is the following sentence:

In the U.S. during the year 2001, routine childhood immunizations against seven diseases were estimated to save over $40 billion per birth-year cohort in overall social costs including $10 billion in direct health costs, and the societal benefit-cost ratio for these vaccinations was estimated to be 16.5.

(The emphasis is mine.) Thinking this was merely a typo for "16:5", I went to the footnoted source which contains the following sentence:

Direct and societal benefit-cost ratios for routine childhood vaccination were 5.3 and 16.5, respectively.

Thus the "ratio" presented in the WP article is correctly transcribed, but I still don't know how a ratio can be a single number. Is anyone willing to explain? Bielle (talk) 18:37, 24 January 2010 (UTC)[reply]

It's 16.5:1. The one's implied in a ratio when they only show one number like that. For every dollar spent on vaccinating kids, $16.50 is saved in health care costs down the road. You can also check out Benefit-cost ratio for information about this specific type of ratio, and cost-benefit analysis for more general information on the subject. Buddy431 (talk) 18:53, 24 January 2010 (UTC)[reply]
When the ':' and the second number is missing, it's generally accepted that ':1' is intended. So 5.3 and 16.5 really mean 5.3:1 and 16.5:1 respectively. In other words, the benefit is 5.3 times the cost or 16.5 times the cost. SteveBaker (talk) 18:54, 24 January 2010 (UTC)[reply]
Really, this is just a matter of different ways of thinking about the same thing. The ratio 14:4 is equivalent to the ratio 7:2, just as the fraction 14/4 is equivalent to the fraction 7/2. The value of the fraction as a number is 3.5 and the fraction can also be written as 3.5/1, just as the ratio can also be written as 3.5:1. But if you had the fraction 3.5/1, you would normally simplify it to just 3.5 (well, you might write 7/2 if you considered that simpler). Well, in the same way, the ratio 3.5:1 can be written as just 3.5. A ratio (of two nonzero terms) and a fraction -- or for that matter a quotient, like 7÷2 -- are really pretty much equivalent, and it's common to see a ratio expressed as a single number.
Incidentally, 7:2 is actually the way they write 7÷2 in many countries; they don't use what we call a division sign at all. --Anonymous, 19:01 UTC, January 24, 2010.
Your division sign ÷ is an Obelus. It is commonly seen today representing minus in Norway, for example in advertisements proclaiming "Opptil 30% på salgsvare" (= Up to 30% reduction on goods in sale). Pity the plight of little Norwegians who must use pocket calculators with buttons + - x ÷ while their math teacher teaches them to write + - . : for the same functions. With luck the little ones may grow up to be programmers who use + - * / respectively. Cuddlyable3 (talk) 22:20, 24 January 2010 (UTC)[reply]
Ummm... there's no obelus is that quotation... --Tango (talk) 02:42, 25 January 2010 (UTC)[reply]
Unfortunately the ratio article doesn't discuss this normalised format for expressing ratio; if this usage is as common as the above implies, then it clearly should. I did try to find a reliable source to back up this usage, but searching for "normalised ratio" and the like only finds specific ratios, rather than this general style of expressing as a ratio:1 and then omitting the :1 -- Finlay McWalterTalk 19:10, 24 January 2010 (UTC)[reply]
Doesn't any number qualify as a ratio that can have a /1 or ÷1 or :1 or whatever done to it without it really meaning anything? --Neptunerover (talk) 19:18, 24 January 2010 (UTC)[reply]
Yes, any number can be a ratio. Ratios don't, as you say in your original question, denote any kind of relationship between the two numbers. "The ratio of X to Y is A:B" means "For every A of X you have B of Y". It's a relationship between X and Y, not between A and B. --Tango (talk) 19:37, 24 January 2010 (UTC)[reply]
You mean that's not the same difference? --Neptunerover (talk) 19:48, 24 January 2010 (UTC)[reply]
I suppose it is, but the fact that a particular (ordered) pair of numbers happens to be the ratio of something isn't very interesting. For any pair of numbers you will be able to find infinitely many things that it is the ratio of. The fact that the ratio of a particular thing is some pair of numbers is far more interesting. --Tango (talk) 20:01, 24 January 2010 (UTC)[reply]
I have a feeling you are on a topic far removed from this page and this question, and from which you already bid farewell. There's no sense confusing the OP here because of something unrelated. --Neptunerover (talk) 21:12, 24 January 2010 (UTC)[reply]
Thank you all. I understand the protocol that has emitted the ":1" when something is called a "ratio" but is expressed as a single number. I am sure, after another read or two, I will also understand the point about A of X and B of Y. Bielle (talk) 19:44, 24 January 2010 (UTC)[reply]
It would probably be clearer if I didn't state it so generally and instead gave an example. "The ratio of boys to girls in the class is 3:2" means "For every 3 boys there are 2 girls". --Tango (talk) 20:01, 24 January 2010 (UTC)[reply]
I expect to see two numbers in a ratio, even if the divisor is one. There should be an antecedent and a consequent. A number, by itself, is not a ratio. Saying "The ratio is 5.3" is unnecessarily confusing. How hard is it to say "The ratio is 5.3:1?" As a college teacher, or as a journal editor, I would have questioned such a usage. Edison (talk) 04:35, 25 January 2010 (UTC)[reply]
Well, what one expects and what actually happens are often two very different things! It is very common to drop the ":1", whether we like it or not. --Tango (talk) 06:08, 25 January 2010 (UTC)[reply]
One as a divisor is assumed, even if it isn't always written fully out. --Neptunerover (talk) 08:14, 25 January 2010 (UTC)[reply]

Incidentally, is a differential equation basically the same thing as a ratio? --Neptunerover (talk) 15:14, 25 January 2010 (UTC)[reply]

No. Not even a little bit. Algebraist 15:16, 25 January 2010 (UTC)[reply]
When ":1" is omitted, we are using a factor (as in multiple), not a ratio, but the usage seems to be widely abused. Do scientific journals really say "cost-benefit ratio" when they mean "cost-benefit factor", or is it just second-hand reporting that makes the error? Dbfirs 15:38, 25 January 2010 (UTC)[reply]
I've never heard anyone say "cost-benefit factor". I think we just have to accept that this is a meaning of the word "ratio". The English language is defined entirely by how it is used. --Tango (talk) 20:44, 25 January 2010 (UTC)[reply]
Yes, I agree I've only ever heard "ratio", but then I would expect, like Edison, to see a ratio. I suppose we have to accept that the "to one" is just understood in the context. I've made a few minor edits to our article on Ratio, but it really needs a major re-write. Dbfirs 14:17, 26 January 2010 (UTC)[reply]

Use of Enoxaparin sodium in aortic dissection.

Is there any indication of use of Enoxaparin sodium in aortic dissection? Maybe it prevents the formation of thrombi in the false lumen (thrombi could migrate and induce arteries occlusion in brain, coronaries ...) or what else? Or Enoxaparin sodium just aggravates dissection by inhibiting coagulation? Thansk so much for replies. --62.98.29.51 (talk) 21:40, 24 January 2010 (UTC)[reply]

I've just checked the article on enoxaparin sodium -- it says that this stuff is indicated for prevention of blood clots during dialysis and abdominal and orthopedic surgery and for treatment of deep vein thrombosis and some kinds of heart attack. Nothing at all about aortic dissection or any other kind of open-heart surgical procedure. I'm not an expert on surgery, but my guess is that it would be contraindicated for open-heart surgery because it could aggravate bleeding and possibly cause a dangerous hemorrhage. Clear skies to you 24.23.197.43 (talk) 03:37, 25 January 2010 (UTC)[reply]
Aortic dissection is not a procedure. It's a real quick way to get dead. I suppose the only way to save you is with a surgical procedure, so maybe that was what you meant. --Trovatore (talk) 03:40, 25 January 2010 (UTC)[reply]
The first plausible-sounding answer is a great example of why we forbid medical advice here! DMacks (talk) 06:48, 25 January 2010 (UTC)[reply]
Speaking of which, I should retract part of my answer — according to the article, surgery is not always the treatment for aortic dissection. It depends on the details. Not that I think anyone's going to say: Doc, I have to have surgery! Some guy on the refdesk said so! But you never know. --Trovatore (talk) 08:05, 25 January 2010 (UTC)[reply]
Sorry, I really thought aortic dissection was a procedure where they open up the aorta to take out some kind of blockage and then sew it shut again. (Well, I told y'all that I'm not a medical expert!) Anyway, now that we all know what aortic dissection really is, I can say with certainty that enoxaparin sodium would ABSOLUTELY, DEFINITELY be contraindicated in that case cause all it would do is to make you bleed out faster. See, my answer was right after all, even though it was for all the wrong reasons! 146.74.230.82 (talk) 01:09, 26 January 2010 (UTC)[reply]

Quick seagull question

If there were no people to give them an incentive to venture inland to scavenge, would gulls be restricted only to coastal areas? --95.148.109.223 (talk) 22:45, 24 January 2010 (UTC)[reply]

I've seen seagulls in Colorado (far away from any coast). I'm not sure if they migrated there because of people. --Neptunerover (talk) 05:01, 25 January 2010 (UTC)[reply]
Of course, my reference to 'Colorado' seagulls may be irrelevant, as who knows when the species may have last seen an ocean. Where I saw them just happened to be at the dump as well. I suppose inland seagulls have lakes instead of oceans, where they might very well stay, if it weren't for the local dump. --Neptunerover (talk) 05:44, 25 January 2010 (UTC)[reply]
I can testify to them congregating around lakes, and not just seas. Here in Vermont we have quite a few near Lake Champlain and at least a good 10 or so miles in the parking lot at my workplace. Dismas|(talk) 11:56, 25 January 2010 (UTC)[reply]
They're a menace, I say! (jokingly; I like live seagulls) --Neptunerover (talk) 16:38, 25 January 2010 (UTC)[reply]
But not the dead ones? Seems an interesting distinction to point out. Dismas|(talk) 20:15, 25 January 2010 (UTC)[reply]
Interesting perhaps, but I felt it was necessary just in case someone wanted to come back with "where do you like them, on your dinner plate?" Sadly, it appears many hunters have this point of view when they speak about how much they love animals. The Native Americans were not like that at all. Hunting is a natural part of life on Earth though, so I'm not going to point fingers (hey, I used to have a BB gun, and I'm not proud of having used it to shoot things that would've kept living had I not shot them, but I was a kid seeking fun in a misguided way. When people grow up, hopefully by that time they realise that killing things isn't really any fun, assuming they ever might have at some point thought that killing is or might be fun, say from a video game.) --Neptunerover (talk) 13:11, 27 January 2010 (UTC)[reply]
Here on the Wikipedia Ref desk, we have physicists, chemists, biologists, mathematicians, computer scientists - and a seagull expert. I'm sure User:Kurt Shaped Box will be along in a moment to help. SteveBaker (talk) 05:06, 25 January 2010 (UTC)[reply]
As far as I know (and to echo Dismas), gulls will hang around/breed near lakes, inland seas and areas of marshland too, without human encouragement. If I'm remembering correctly, there are huge colonies to be found inland in Central Asia in these habitats. Gulls, being the tough, adaptable little generalists that they are, don't strictly *need* to be around accumulations of water in order to eke out a living - but they seem to be 'drawn' to it. --Kurt Shaped Box (talk) 19:57, 25 January 2010 (UTC)[reply]
I was thinking about this on my morning commute and I think that Life After People mentions what would (will?) become of gulls. Yep, just checked the article and it says that there is a large die-off but that populations come back once fish populations start to rebound. Dismas|(talk) 20:15, 25 January 2010 (UTC)[reply]

Incidentally, the book Jonathan Livingston Seagull is very enlightening as to the life of a 'seagull', although the reliability of the book's perspective, at least on this desk, is not verifiable. (still a good book though, and short too) --Neptunerover (talk) 13:11, 27 January 2010 (UTC)[reply]


January 25

why is THF called THF?

It seems to me that you're adding TWO equivalents of hydrogen across the furan ring... not four. Shouldn't it be called dihydrofuran? Shouldn't the current (2,3) dihydrofuran be called (mono)hydrofuran? I mean, I don't think you can add more than one H2 molecule across a double bond... John Riemann Soong (talk) 06:17, 25 January 2010 (UTC)[reply]

Two equivalents of dihydrogen (H2) are added. The "hydro" means "a hydrogen atom", not "the molecular form of that element". Like hydrohalogenation, hydroformylation, etc., each add "one hydrogen atom, one other thing". But the real why is because, just like the answer to any "why is some standard used". Something had to be agreed, and even if it is completely nonsystematic and makes no sense to anyone, it is how it is, and asking "why" doesn't lead anywhere useful (though in this case it sort-of does). [2] DMacks (talk) 06:47, 25 January 2010 (UTC)[reply]

name of those fenced-in forests of powerlines and insulators etc?

Do you know the places I mean? What are they called?

Thanks Adambrowne666 (talk) 13:04, 25 January 2010 (UTC)[reply]

Are you talking about an electrical substation? -- kainaw 13:10, 25 January 2010 (UTC)[reply]
it is indeed; thanks, Kainaw Adambrowne666 (talk) 20:51, 25 January 2010 (UTC)[reply]

Menstrual period pregnancy

is it possible to get pregnant during menstrual period? —Preceding unsigned comment added by 119.95.22.212 (talk) 13:20, 25 January 2010 (UTC)[reply]

Yes. The Feminist Women's Health Center has a rather nice article describing the full 28-day cycle here. -- kainaw 13:38, 25 January 2010 (UTC)[reply]
As an aside, there are some studies out there saying that women are not "born with all the eggs they will ever use", but continue to produce eggs after they are born. I'm not sure if that has been conclusively proved or not, though. Clear skies 146.74.230.82 (talk) 01:20, 26 January 2010 (UTC)[reply]

The FWHC site has mostly accurate information but really provides no info about incidence of fertility during menstruation other than the bald statement that "it could happen". The FWHC site is badly in need of copyediting: it's full of mistakes in spelling, capitalization, syntax, and the common but annoying failure to grasp the difference between adjectival mucous and nominal mucus. It also mixes western folk physiology and medicine in with the science-based information, though it has no trouble properly identifying as such the folk traditions of some other cultures. It also appears to promote consumer sexism by advising its readers to buy products from companies owned by women. Other than those small quibbles, a great site. A much better reference would be this one: [3]. alteripse (talk) 14:46, 26 January 2010 (UTC)[reply]

In practice, a body temperature measurement of the female is done first thing in the morning, and a certain change of a fraction of a degree indicates ovulation. The date can be off if something else, like a bit of fever, affected the basal temperature. Menstruation can similarly be mistaken for bleeding from other causes. Edison (talk) 05:56, 27 January 2010 (UTC)[reply]

Circadian rhythm studies

Hi,

I was told that many early studies into human circadian rhythms had not been careful enough about letting outisde light in, and this screwed up their results. Does anyone know any examples and can lead me to a paper or 2?

Cheers, Aaadddaaammm (talk) 15:23, 25 January 2010 (UTC)[reply]

I'm not really familiar with that objection. In the 1930s Nathaniel Kleitman was already conducting isolation experiments in caves in Kentucky, which would seem to give pretty good control. He had also conducted other isolation experiments as far back as the 1920s that might not have been as convincing, but I don't know much about them. He published a book called Sleep and Wakefulness in 1939 that summarized his findings, but I haven't read it. Looie496 (talk) 19:15, 25 January 2010 (UTC)[reply]
Sleep#Timing may be of interest. ~AH1(TCU) 20:23, 27 January 2010 (UTC)[reply]

wind turbine generator manufactures

I have patented a "Electric Power Train" this is for generating electricity from one of the forces (pressure)generated from automotive/railway traffic on road and rail.What I am looking for is someone who could give me the specks on the genrator/alternator being used on wind turbines at present.To name a few speck's I am looking for are how much force is needed to turn the shaft without the blades,what RPM is required to get the best results,I know you will be asking me what size generator or how much electrisity do I want to generate.I cannot say at this stage,but I do want a medium to large,For example one of wind turbine being used on the wind farm's,that will give me a good start and something to work from.If I could get contact details of a manufacture that would be good.I am based in the UK so one as neer as possible would be great. Thanks Pierre —Preceding unsigned comment added by 86.163.87.199 (talk) 15:23, 25 January 2010 (UTC)[reply]

If your plan is to mount the wind turbine on the car or train, you will be wasting more energy than you are creating by adding drag and decreasing fuel efficiency. You would do better to simply tap off the engine power directly with an alternator. This is already commonplace. If your plan is to mount the generator near the roadway or railway to collect wind from passing trains or cars, I think you will find that the amount of extractable energy is not cost effective (it will probably be darn near zero watts). In specific answer to your questions, there is no way we can answer details about force, torque, and RPM, unless you tell us your desired specifications. Perhaps you might take a look at wind power, which gives an encyclopedic overview of present technologies. If you're unfamiliar with engineering design in general, it may be worth reading up on that before you start a major venture and contracting manufacturers. Nimur (talk) 17:08, 25 January 2010 (UTC)[reply]
How did you get your patent without specifications? Perhaps I don't understand how you propose to generate your power. What pressure force are you planning to use? Here [4] is a manufacturer of small turbines who might be able to help you. Dbfirs 17:36, 25 January 2010 (UTC)[reply]
What country have you patented your invention in, and what is the patent number? Or did you mean simply "invented" or "thought up an idea?" Edison (talk) 21:26, 25 January 2010 (UTC)[reply]
Perhaps the OP's idea is to harness the recurring downward pressures of road and rail vehicles passing over a flexible roadway. Cuddlyable3 (talk) 21:44, 25 January 2010 (UTC)[reply]
That would produce even less energy! Really, though, the OP specifically said that he/she wanted to use a wind turbine, so we can safely assume that he/she is planning to harness the boundary layer created by passing cars/trains. (Oops, sorry, I just looked, the OP is definitely a he, based on his name.) Not that it would be any more cost efficient. And another objection to the OP's proposal: if he wants to use a wind turbine of a comparable size to the ones used on a wind farm, then (1) there won't be enough roadside/trackside space where to put it, (2) if the turbine is to be mounted low enough to catch the slipstream, then the blades will strike the ground, and (3) even if the turbine was somehow mounted low enough, only the very tips of the turbine blades will ever be exposed to the slipstream -- the rest of the blade will remain idle! This is clearly an impossible and even borderline ludicrous proposal, and I'm very astonished that the UK patent office has issued a patent for this contraption. Clear skies 146.74.230.82 (talk) 01:37, 26 January 2010 (UTC)[reply]
To be fair to Pierre, during the last year or so I have read in New Scientist magazine of schemes to recover energy from the pressure of foot or vehicular traffic on walkways and roadways, as Cuddlyable3 surmises, and I believe a couple of experimental setups are or have been trialled, though I can't find anything from hasty googling. One criticism of such schemes is that most of the energy they would recover would be extra energy that the person or vehicle would have to expend to negotiate such a surface, which otherwise would not be required: thus, they would in effect be using the vehicle's engine or the person's muscles as the generators and recovering the extra energy rather inefficiently.
It may be that Pierre only wants the specifications ("specs", not "specks," Pierre!) for wind turbine generators in order to crunch some comparative numbers - I can't see how such machinery could be repurposed for surface energy recovery if that's the idea, though the specification in his Patent application should make everything clear - could we have the Patent Office and Application number please, Pierre? 87.81.230.195 (talk) 03:02, 26 January 2010 (UTC)[reply]
Google energy from speed bumps will find some articles (not necessarily the New Scientist article). Mitch Ames (talk) 06:40, 26 January 2010 (UTC)[reply]

volumetric display using microscopic hemispheres? Has this been thought of yet?

Would pixel size hemispheres coated with red,green and blue phosphors with a pinhole on the flat side, inside a CRT allow holographic television. Or would the pinhole have be too small for the wavelength of light to allow standard or high definition images? Could such structures (transparent to let the phosphor colour through be constructed on such a small scale?Has anyone though of this or made a large holographic television with full vertical/horizontal parallax, (preferably with the quality of a good CRT) yet, and if so, where and when can I expect to buy one at a reasonable price? [[File:http://www.imagekind.com/services/frame_engine_https.ashx?IMID=568d016e-066a-4f08-b914-bb6e7ae59693&frameId=602&glazingId=4&topMatId=1324&topMatSpacingTop=2.5&topMatSpacingBottom=2.5&topMatSpacingLeft=2.5&topMatSpacingRight=2.5&middleMatId=-1&middleMatSpacingTop=0.25&middleMatSpacingBottom=0.25&middleMatSpacingLeft=0.25&middleMatSpacingRight=0.25&bottomMatId=-1&bottomMatSpacingTop=0.25&bottomMatSpacingBottom=0.25&bottomMatSpacingLeft=0.25&bottomMatSpacingRight=0.25&IMIDArray=&IMIDCropArray=&IMIDRotateArray=&typeId=-1&cropLeft=0&cropTop=0&cropRight=0&cropBottom=0&sizeId=1&materialId=1&maxWidth=200]] —Preceding unsigned comment added by Trevor Loughlin (talkcontribs) 15:29, 25 January 2010 (UTC)[reply]

For the second part of your question: Nobody has made any kind of holographic television yet... and even if you had one, it's not like all of your 2D television would suddenly be 3D. The signal just does not carry that information most of the time.
That being said, see 3D television. There are apparently a number of stereoscopic TVs (you wear funny glasses like in the movies) expected to hit the market this year, and ESPN is apparently starting the first full-time 3D channel, or something along those lines. These are not holographic—they are 3D in the same sense as a 3D movie. You can't look around at every angle, they just provide a little parallax.
I know SteveBaker has much to say about volumetric displays, so I'll definitely wait to see what he says about the first part. --Mr.98 (talk) 16:30, 25 January 2010 (UTC)[reply]
He probably said it last time Mr Loughlin asked about them.
I think what Loughlin is describing is a type of Lenticular display, Where each hemisphere/pixel shows a different color depending on the angle it's viewed. Certainly this idea has been thought of. (Everyone working on 3d displays thinks of this idea.) But it's just not practical. (Consider the number of pixels needed, multiplied against the number of angles each pixel could be viewed!) All that said, the technology is just now within grasp. Barely. I saw one at SIGGRAPH 09. It looked very primitive, but it was actually using a tremendous amount of computing power, and very expensive display elements to maintain the illusion. (If I recall correctly, it also had a serious overheating problem.)
I'll see if I can quickly find the reference.
APL (talk) 17:22, 25 January 2010 (UTC)[reply]
Found it. It's called the gCubik. See a (poor quality) video here. The paper on it is here, but I can't find a free copy. APL (talk) 17:27, 25 January 2010 (UTC)[reply]
Here's a better video [5]. APL (talk) 17:38, 25 January 2010 (UTC)[reply]
To get a hologram you are going to have to control the phase of waves between the pixels, and to get a wide angle view, your pixels will have to half a wavelength or less. This will be of order of a megapixel per square millimeter, or about a terapixel for a square meter display. The information required is more because you need phase as well as brightness for each pixel, to get an uncompressed TV picture you will need around a petabyte per second of data. Graeme Bartlett (talk) 00:10, 26 January 2010 (UTC)[reply]
We have to be very careful to define what we mean by a 3D display. They come in roughly four kinds:
  1. Single viewer (you have to be sitting in the right place) - hence a very limited view direction - but no special glasses needed. Lenticular displays are like this.
  2. Multiple viewers - but all of whom get the same view - with special glasses (that's what you get in a 3D movie or using red/cyan glasses or a head-mounted display).
  3. Multiple viewers in multiple locations - all getting the correct view for where they are standing without glasses or anything. Holograms are pretty much it.
  4. Multiple viewers in multiple locations - all getting the correct view for where they are standing...but with one horrible drawback (the kind of thing our OP is describing, bubble tanks, spinning grids of LED's, vibrating mirror displays, etc, ad nauseum).
The problem with the OP's idea is that you can't control transparency. You can't stop light from the back of the object from shining through to the front - or from the left through to the right - and vice-versa. So everything looks ghostly. You can't ever display a realistic picture - period.
So - we're back with holograms - which are still ruinously expensive to generate in realtime. But we're getting there. Graeme is technically correct about needing a petabyte per second to broadcast raw volumetric data at hologram densities - but that's not the plan. You can send (for example) octree-encoded data about the shape of the objects and use on-the-fly wavefront reconstruction to generate the hologram. That blows away the bandwidth issues (well, relative to a petabyte per second at least!) - and shifts it into doing crazy high numbers of calculations. But that's actually more do-able. So I think it's possible that we'll see animated, interactive true holographic 3D displays in our lifetimes - but it's not certain.
Bottom line then: We're stuck with glasses until we get crazy amounts of CPU time...and sadly, the OP's idea isn't gonna cut it. Vibrating or spinning mirror displays can do the same job better, cheaper and much more easily. SteveBaker (talk) 01:50, 26 January 2010 (UTC)[reply]

I would dispute the transparency claim, the hemisphere has an opaque flat side with a pinhole, it would be like looking through a tiny hole in a fence from all angles, except there could be a lot of them, too small to see. I think opacity would occur, but the other practical problems including data rates are grim. —Preceding unsigned comment added by Trevor Loughlin (talkcontribs) 03:50, 26 January 2010 (UTC)[reply]

So you're saying that you build a cube packed with these hemispheres - each of which can produce light in any color and has a hole in it to allow light from behind to come through it? Your explanation is less than clear and the image you linked to doesn't help! SteveBaker (talk) 14:54, 26 January 2010 (UTC)[reply]
I really think you're describing something similar to the gCubik device I mentioned above. It worked on little hemispheres. It looked like a toy, and had serious problems, but it pushed the limit of modern technology.
Incidentally, I think you would have really enjoyed the "Emerging technologies" section of the SIGGRAPH 09 show. Besides the gCubic, there was this, which is very similar to what you described last time you asked. However, there's still nothing even close to the quality that you described the first time you asked.
Of course, you're the one with a direct communication line to the future, why don't you tell us what the future holds for 3d displays? I'd very much like to know. :-) APL (talk) 05:37, 26 January 2010 (UTC)[reply]
The gCubik is just a standard 'lenticular' display (a small improvement on the prismatic 3D display that's been used in things like 3D postcards for 50 years or more). Each display produces a 3D image - but only over a relatively small field of view. By gluing them onto the six faces of a cube, you hope that people concentrate only on the side that's facing towards them - ignoring the other sides that aren't making much sense from this angle. So the limited lenticular display only has to produce a fairly small field of view (out to maybe 90 degrees) through each side of the cube to make something that's kinda-sorta useful. SteveBaker (talk) 14:54, 26 January 2010 (UTC)[reply]
gCubic is lenticular, but it's lenticular in two directions, it's the only display I've ever seen that's like that. The lenses are not linear prisims, they're (roughly) hemispherical dots. Loughlin's post is not entirely clear to me, but this device uses clear "pixel size hemispheres", to create a coherent 3d image that is roughly correct (Within 5 degrees or something. So, let's say 'very roughly'.) when viewed from any angle. Then they built it into a cube, creating a heat management disaster. APL (talk) 16:17, 26 January 2010 (UTC)[reply]
Anyway, my real point in bringing it up, and I think I didn't make that clear enough, was that this obvious idea ("What if we made pixels that looked different from every angle!") isn't really practical. The absolute cutting edge, state of the art is nothing more than a fun tech demo, and the technology requirements go up asymptotically with your resolution. So it will probably never be practical for high-def, almost certainly not in our lifespans. APL (talk) 16:24, 26 January 2010 (UTC)[reply]
I was about to ask a question about the '3D' glasses as used to see the movie Avatar in 3D. But I looked up the relevant article and here's the link for anyone interested. RealD Cinema 220.101.28.25 (talk) 09:54, 26 January 2010 (UTC)[reply]
For clarification, the type of 3D glasses used to see Avatar depends on which cinema you go to. Different cinemas have adopted different technologies, and of course these are compatible enough that the studios or distributor can easily make versions suitable for whatever technology the cinema uses and for a mass release like Avatar they do since there's no reason they're going to limit the movie to a subset of cinemas but just sell it to whoever wants it. I personally watched it with Dolby 3D and there's also a local Imax 3D cinema that had it (for NZ$3 more). Nil Einne (talk) 08:21, 28 January 2010 (UTC)[reply]
Thanks for that info. Nil. I was expecting the lenses to be polarised at 90° to each other, (which apparently doesn't work well in practice) didn't think of opposite circular polarisations. I wonder which system (Dolby 3D vs RealD) is best? Dolbys system sounds more complex. And if I go back to the same theatre to see Avatar again will they make me pay for another set of 3D glasses? (I kept mine) Almost certainly!...220.101.28.25 (talk) 12:07, 29 January 2010 (UTC)[reply]

(NOTE TO RESPONDENTS: In case you missed APL's reference to our OP having a 'direct communication line to the future', Trevor has asked about 3D displays here on WP:RD/S a couple of times in the past (see APL's links above) - but he also claims to have built a practical device that can predict the future using "retrocausal data transfer".) SteveBaker (talk) 14:54, 26 January 2010 (UTC)[reply]

Voltage divider

I had a question about the voltage divider article, specifically the derivation of the formula V_out=V_in*R_2/(R_1+R_2). I see that V_in should be I(R_1+R_2), and that therefore the potential after the first resistor is IR_2. But when a path for the output voltage is provided, why should the current running through the first and second resistors be the same? —Preceding unsigned comment added by 173.179.59.66 (talk) 19:18, 25 January 2010 (UTC)[reply]

By definition if R1 and R2 are in series, the same current flows through them. The formula is for ideal conditions, and does not assume any of the current through R1 flows through a physical voltmeter instead of R2. In practice a modern digital voltmeter has an input resistance of perhaps 6 megohms. Older cheap analog voltmeters could load down very high resistance circuits and affect the measured voltage appreciably. Whatever the input resistance of the voltmeter, it could be modelled as a parallel resistor across R2. If R2 has less than 1/100 of the resistance of the voltmeter, then the voltage would be off by less than 1%. A Potentiometer (measuring instrument) (in the older sense, rather than a variable resistor in the modern usage of the term) can be used to measure voltage without extracting any current once the adjustment is complete, by having a very sensitive galvanometer between the point and a known voltage, with the known voltage adjusted until no current flows through the galvanometer. A potentiometer I used had a standard cell, with a pretty constant known voltage, from which almost no current was ever drawn, and a slidewire, where a resistance wire of known composition and known resistance per unit length was tapped by a slider. By the bridge circuitry, an unknown voltage could be measured by tapping the key which conected the galvonometer into the circuit and adjusting the slider until there was no deviation of the galvonometer. This 19th century technique was still used in the late 20th century for calibrating some telemetry circuits. Edison (talk) 20:46, 25 January 2010 (UTC)[reply]

extrasolar planets

I did not note any mention in the article, but have there been any extrasolar planets discovered in binary star systems? Googlemeister (talk) 19:57, 25 January 2010 (UTC)[reply]

Yes. There are at least three planets in the Upsilon Andromedae system, for example. Algebraist 20:01, 25 January 2010 (UTC)[reply]
Alpha Centurai#Possibility of planets mentions Gamma Cephei also. —Akrabbimtalk 20:03, 25 January 2010 (UTC)[reply]
With the Upsilon Andromedae system, how can they tell the difference between a planet proper, and a red or brown dwarf star? Googlemeister (talk) 20:08, 25 January 2010 (UTC)[reply]
Our theories tell us the minimum mass of a brown dwarf and the observations of Upsilon Andromedae give estimates of the masses of the planets, and they are well below the brown dwarf threshold. Our article only gives lower bounds for the masses of the planets for some reason, but upper bounds will also be known. --Tango (talk) 22:11, 25 January 2010 (UTC)[reply]
Not necessarily, observations of gravitationally induced wiggling generally constrain the product M*sin i, where M is the mass and i is the inclination of the orbit. So you get a lower bound if the orbit is perfectly aligned for our viewing and sin i = 1, but it is unbounded in the upper limit since the planetary orbit might be directly skew to what we hope to observe. And there is no direct way to constrain i from wiggling alone (usually). You'd need to follow up gravitational detection with some other method to find an upper bound and that hasn't yet been done in many cases. Dragons flight (talk) 22:50, 25 January 2010 (UTC)[reply]
If there is just the one planet, then that's true. In this case, it's a whole system of planets and the interactions between them put constraints on the inclinations. The article mentions this. --Tango (talk) 23:40, 25 January 2010 (UTC)[reply]
What if we had a planet rotating around the center of gravity of the binary system? Is that possible? ~AH1(TCU) 20:22, 27 January 2010 (UTC)[reply]

Laws of Thermodynamics

What are some everyday examples for all the Laws of Thermodynamics or how can I explain these laws to a kindergartener? --Reticuli88 (talk) 20:35, 25 January 2010 (UTC)[reply]

On At the Drop of Another Hat, is the song "First and Second Law" by Flanders and Swann. Its annoyingly catchy and a fun way of teaching kids that basic concept. a brief search of the web will let you hear it for yourself. Rockpocket 21:02, 25 January 2010 (UTC)[reply]
Try it with something like lego blocks. The first law says that nothing is created or destroyed, it only changes form. Any legos you use can't be used to build something else. If all of your legos are part of a house, and you want to build a car, you need to take some legos from the house. Legos are thus, conserved. The second law says that things just become messier unless you do something to clean it up. Hence, the legos don't pick themselves up! The room would just get messier and messier unless you spend some time to pick up the legos and put them away. However, time spent putting legos away means you can't do something else during that time. So you can't get something for nothing. Either the room gets to messy to play in, so you can't play with anything because its a complete mess, OR you spend time cleaning it up, and then you don't have any more time to play. Either way you can't play all that you want. Entropy is a mean bitch that way. The third law is a bit tougher, but basically it's the cabinet where you put your toys away. The third law simply tells you where all the toys go when you pick up the room. Or, as someone eloquently put it to me:
  • The first law says you can't win or lose, you can only tie.
  • The second law says that you can't even tie, you can only lose.
  • The third law is the rules of the game.
Just some ideas bouncing around my head.--Jayron32 21:18, 25 January 2010 (UTC)[reply]
The third law is that you can't quit the game. --Anon, 22:42 UTC, January 25, 2010.
You might like to try the 'simple English' Wikipedia article.
I like the idea of a Lego analogy - but I think we could do better (and list all four laws):
0) If I have six lego bricks and you have six lego bricks and your best friend has the same number of bricks as you - then she also has the same number of bricks as me.
1) You only have just so many bricks. No matter what you build or how you put them together, there are always exactly the same number of bricks. Even if you lose bricks down the back of the sofa or your dog eats some - the bricks still exist...somewhere!
2) If you make a nice tidy pile of lego bricks - carefully arranged by size and color - then if someone shakes them up - they never get any tidier. It takes a lot of work to make a tidy pile of lego bricks - but hardly any effort for your little sister to mess them up again.
4) 3) If you had a small box that was only just big enough to fit all of the lego bricks inside - they'd have to be really, really tidy to fit inside! If you jumble them all up - they wouldn't fit properly.
Physicists call untidyness "entropy".
SteveBaker (talk) 00:31, 26 January 2010 (UTC)[reply]
0, 1, 2, 4? Thermodynamics is weird! ;) --Tango (talk) 03:21, 26 January 2010 (UTC)[reply]
Binary counting perhaps? 2, 4, 8, 16, 32, 64 etc. :-) 220.101.28.25 (talk) 05:36, 26 January 2010 (UTC)[reply]
I never liked rule 3 and it's too hard to do with Lego - so I invented my own rule #4....or it was a typo...you choose! SteveBaker (talk) 13:58, 26 January 2010 (UTC)[reply]
I don't see the point in teaching such abstract concepts. Small children should be encouraged to be inquisitive of knowledge, appreciative of nature, and skeptical of any claims. 66.65.139.33 (talk) 02:24, 26 January 2010 (UTC)[reply]
Yeaaah. Who says we can't build a perpetual motion machine ? Scientists ? What do they know ? I bet they just haven't been inquisitive enough. Let's try flying a kite in a thunder storm ... Gandalf61 (talk) 11:00, 26 January 2010 (UTC)[reply]
Certainly, if anyone ever does build a perpetual motion machine, it'll be with Lego. :-) SteveBaker (talk) 13:59, 26 January 2010 (UTC)[reply]
I hope you don't mind my being forward, but I must wonder what the purpose of explaining thermodynamics to a five year old would be. Even if they are prodigal and could understand it, I shouldn't think they would have the intellectual curiosity to understand it. Vranak (talk) 14:52, 26 January 2010 (UTC)[reply]
You said it better than I did. 66.65.139.33 (talk) 15:15, 26 January 2010 (UTC)[reply]
While it's probably unlikely that they'd really grasp the concepts enough to appreciate your answer, they would learn something much more important - that such things can be questioned. My daughter was 6 when I started showing her Eureka! episodes. While she obviously can't get out of it what a high school student would, she's at least familiar now with concepts such as "entropy" and "atoms". Young kids are smart - never doubt it for a moment; a five year old has a better grasp of using English than any programmed AI system and can run and climb better than any robot - and they do it almost entirely without instructions of any kind. Matt Deres (talk) 17:14, 26 January 2010 (UTC)[reply]

do E. coli process cellulose?

If so, if you eat lots of pure fiber, can you get drunk off the EtOH they produce? John Riemann Soong (talk) 21:16, 25 January 2010 (UTC)[reply]

Probably not. Remember that if it worked that way, someone before you would have thought of it. Plus, intestinal flora (of which E. Coli is but one part) produce prodigious amounts of CO2 during the ethanol producing process. So, anything which could produce enough ethanol in your gut to make you drunk would probably also produce enough gas to make you painfully crampy and bloated to the point that being drunk wouldn't be worth it. If you REALLY want to make booze and don't care much about the enjoyment of drinking itself, then it's just best to let the fermentation go on outside of your body. See pruno for a simple recipe. --Jayron32 21:23, 25 January 2010 (UTC)[reply]
Wouldn't that just mean I would fart a lot? For why it's not been thought of yet (or why it's not popular), people usually don't eat pure fiber. And other sugars would be digested before it could get processed into ethanol.
And the main idea is to get drunk without tasting a single drop of alcohol. John Riemann Soong (talk) 21:33, 25 January 2010 (UTC)[reply]
In theory, I suppose an alcohol enema would work. But the alcohol would hit the bloodstream fairly directly, I reckon, and you wouldn't have the safety of gradual absorption or vomiting to prevent acute alcohol poisoning. Much safer and more pleasant to just drink it. Brammers (talk) 21:40, 25 January 2010 (UTC)[reply]
Naturally, Wikipedia has an article (section) on this: Enema#Rectal drug administration. Comet Tuttle (talk) 22:07, 25 January 2010 (UTC)[reply]
Very few organisms can digest cellulose -- our termite article discusses this a bit. Looie496 (talk) 21:42, 25 January 2010 (UTC)[reply]
"Designated drunk: Can you get intoxicated without actually drinking alcohol?" APL (talk) 22:53, 25 January 2010 (UTC)[reply]
The kind of E.coli that you got inside your azz do not digest cellulose, as far as I know. That said, there are some mutant / genetically engineered strains of E.coli that do convert cellulose to ethanol -- I've studied those firsthand for the oil company's cellulosic ethanol project that never got past the lab studies stage. You really wouldn't want to drink the product, though, because of the high concentration of fusel oil -- not unless you wanna get really drunk really quick and don't give a dam about the horrible taste or any tummyaches you might get later. You can burn the stuff in a car engine, though, but it won't be cost-effective in the current energy market. Clear skies 146.74.230.82 (talk) 01:49, 26 January 2010 (UTC)[reply]
If there were bacteria capable of efficiently digesting cellulose, why wouldn't they very quickly reduce all trees, woods, grasses into a bubbly slime? 95.115.188.228 (talk) 07:59, 26 January 2010 (UTC)[reply]
Plants have their own defences. John Riemann Soong (talk) 11:30, 26 January 2010 (UTC)[reply]
Even inside the lifeless wood of the trunk? 95.115.188.228 (talk) 17:13, 26 January 2010 (UTC)[reply]
Well there you start to get lignin, and plus all those tannins are kind of acidic. John Riemann Soong (talk) 18:55, 26 January 2010 (UTC)[reply]

Is Indonesia part of Eurasian plate or Australian plate. Since Scotese map concludes indonesia will collide with Australia, but the map we have Eurasia in green shows indonesia is not in that county but in orange this shows Indonesia is part of Australian plate. how can Indonesia collide with Australia when the sense is like "taking a train to school" and "what you ate for lunch". This is strange when something inside a plate collides with something inside a plate. --209.129.85.4 (talk) 20:54, 25 January 2010 (UTC)[reply]

In fact there are some smaller plates involved (at least in some versions), most of Indonesia lies on the Sunda Plate, a tiny part near Aceh is on the Burma Plate while the eastern part is part of a complex mess of microplates. None of Indonesia is on the Australian Plate. However, the Sunda Plate is actively colliding with the Australian Plate at its eastern end. Mikenorton (talk) 21:48, 25 January 2010 (UTC)[reply]
Here's a link to microplates. ~AH1(TCU) 20:20, 27 January 2010 (UTC)[reply]
And here's one to tectonic microplates. Mikenorton (talk) 20:37, 29 January 2010 (UTC)[reply]

how do I get rid of awful vibration artifact in my speaker setup?

Just to be clear that this isn't some "audiophile" mumbo-jumbo, I'm not speaking of some subtle effect that's bothering me on a subconscious level, I'm talking about a really clear, awful sound that's really as clear as day. The sound I'm talking about is this awful vibration sound that you usually get when you record something and for example yell into the microphone much louder than it can support. Problem is, I'm not playing anything that has that sound in it, and further, I am NOWHERE NEAR the limit of the speaker set up -- it could be way, way louder (WAY louder). So my question is: how do I get rid of that awful noise? Like, am I supposed to turn the line out (playing from a music device/my computer) down, or up, or almost completely up, etc. The setup is: computer -> remote (with volume control) -> speaker system (with separate sound volume on the back, independent of the volume knob). Is there some way to set these to get the desired effect? THanks! —Preceding unsigned comment added by 84.153.202.229 (talk) 23:31, 25 January 2010 (UTC)[reply]

Your speakers are farting. Possible cause 1: you are over-driving them. From your description, this seems unlikely. Possible cause 2: one or more cones is physically damaged. The general cure for that is reconing them or replacing them entirely. To assist in the diagnosis, are you able to detach the speakers from the current setup & drive them from some other set-up (to see whether the problem stays with the speakers, or is associated with the PC/Remote setup?. And in any event, think yourself lucky. I've spent half an evening under my desk trying to get my PCs headphone socket working, with no joy :( --Tagishsimon (talk) 23:38, 25 January 2010 (UTC)[reply]
The artifact you report might be clipping due to overdriving the system with too high an input. If you have a friend with an audio oscillator(signal generator) and an oscilloscope, it might be possible to input sine waves (pure tones) at varying frequencies and amplitudes and determine when the objectionable sound comes out of the speaker. If the recorded and played back sounds are free of clipping, then maybe the fault is somewhere between sound source, amp, and speaker. Perhaps the speakers are damaged, as mentioned above. Try borrowing and listening to good speakers, while keeping the volume below a level which would damage them. The square waves resulting from clipping can damage speakers. Edison (talk) 02:29, 26 January 2010 (UTC)[reply]
I've noticed I can get clipping (audio) on my headphones, plugged directly into the computer, if I have the computer's volume turned way down low and the volume on the media player turned up high... or was it the other way round? Anyway, worth considering that sort of internal clipping, whatever it is. 81.131.17.2 (talk) 11:16, 26 January 2010 (UTC)[reply]

January 26

Cloning question

Have we yet devised a theoretical method by which the memories and personality from the original might be restored/awakened in a clone? I know that there was some idea a while back of actual memories being encoded in genetic material but I don't know if that's been discredited now. —Preceding unsigned comment added by 95.148.105.150 (talk) 00:20, 26 January 2010 (UTC)[reply]

It sounds pretty fishy to me. Considering we don't really have a great understanding of the neurological basis of memory anyway, but the formation of memories is clearly somatic and not genetic, the idea that you could code memories in genes seems rather... unlikely. We'd need a far better understanding of how memory itself worked in the brain to begin with, much less a light-years superior understanding of the human genome. --Mr.98 (talk) 00:37, 26 January 2010 (UTC)[reply]
Aside from any other consideration - there is nowhere near enough 'space' in a DNA molecule to store that much memory. At best - each 'letter' of the DNA is one of C,T,G,A - two bits of information per base-pair. Human DNA has about 3x109 'letters' so it can store at best 6,000,000,000 bits - less than 1Gbyte. That's just pathetic! You can easily store more than that on a regular CD-ROM. My PC has five times that much main memory! And bear in mind that this DNA also has to contain enough information to construct our clone's entire body, to run it's biochemistry, to repair and regrow bits of the body that fail, etc. There is simply not enough space to store the memories of even a few days let alone a lifetime. So - a clone is no different than an identical twin - it would be born just like any other baby - grow, learn, experience the world and end up a very different person. Memories in the brain are retained over the long term as physical connections between neurons which change over time. The clone would have to have an exact duplicate of all of those neural connections - but that can't be because a clone has to be grown from a baby - and the baby clone's brain doesn't have enough space - enough cells to store an adult brains' worth of memories. So this is a complete and utter bust. There is no way to do this. SteveBaker (talk) 02:14, 26 January 2010 (UTC)[reply]
Though when you consider that ~30 billion memory-associated neurons in the cortices and hippocampus have 6,000,000,000 bits each, the capacity is not quite so pathetic. Each cell is terminally differentiated, so it doesn't actually need to contain enough information to construct our clone's entire body, just enough to code for the transcriptome of that particular neuron. How much genome variation (coding capacity) there is between neurons is currently unknown, but there are some suggestions that they could be quite significant and play a role in brain development and maintenance. The idea of genetic memory is a bust, of course, but not necessarily because of limited coding space. Rockpocket 06:32, 26 January 2010 (UTC)[reply]
Oh, sure, memory could be stored using DNA (although there is no evidence to suggest it is and plenty to suggest otherwise), but that wouldn't help a clone. When you clone someone you only get to use one set of DNA, not one for each cell. --Tango (talk) 10:30, 26 January 2010 (UTC)[reply]
Yep - and that's why Rockpocket's answer is not relevant to this question. If our memories were routinely stored in DNA, consider how long it would take to retrieve one. Somehow, the brain would have to locate the right cell that contained the DNA where the answer was stored - then somehow duplicate the right section of that DNA and use that to provide the answer. How would the indexing mechanism work? How would we find the right molecule? No - the brain is a computing machine and the connectivity of the neural net is what stores information AND allows it to be retrieved so quickly. SteveBaker (talk) 13:54, 26 January 2010 (UTC)[reply]
You would need to use some kind of nano-tech duplication technology (which no-one has invented yet) and actually copy the body, including the brain. If you use a normal cloning technique it is, as Steve says, just like having an identical twin. --Tango (talk) 02:19, 26 January 2010 (UTC)[reply]
This is another form of Lamarckism, and has long been discredited save for some fringe scientists who keep trying to bring it back. It is futile. 66.65.139.33 (talk) 02:20, 26 January 2010 (UTC)[reply]
Such memory transfer is of course a popular device in Science Fiction, where it may be used to raise philosophical questions about the nature and continuity of identity and selfhood in addition to being a useful plot device. A recent example of the many novels, etc, exploiting the idea is John Scalzi's Old Man's War. 87.81.230.195 (talk) 02:31, 26 January 2010 (UTC)[reply]
If by 'restored' you mean, copy the mind of the original into the clone? As Mr.98 says, our understanding of the brains memory (or even its 'programming') is not good enough. It seems an unlikely possibility. But, then so was flying etc. I would think ESP is more likely than Genetic memory. Not deriding your question, but the general concept (with a different mechanism) is also nicely addressed in the novel "To Your Scattered Bodies Go" by Philip Jose Farmer. Note I wrote this(slowly) while others were posting so it doesn't take into account all the erudite responses before me. --220.101.28.25 (talk) 02:44, 26 January 2010 (UTC)[reply]
Lamarckism as referred to by IP 66.65.139.33 above, DNA & Genetics--220.101.28.25 (talk) 05:43, 26 January 2010 (UTC)[reply]

Pretty much nobody nowadays believes that memories are stored using DNA -- the almost universal belief is that they are stored by modifying the synaptic matrix, that is, the connections between brain cells. So, replicating the full set of memories would mean replicating the synaptic matrix, which would involve nanoengineering far beyond our current capabilities. It isn't even just a matter of making the synapses match: the numbers and positions of brain cells are themselves dependent on experience, so a clone and an original won't even have matching sets of brain cells. Looie496 (talk) 16:39, 26 January 2010 (UTC)[reply]


What our question-asker is probably thinking of is the theory of Memory RNA. The Wikipedia article isn't very good, but it hits the important fact that the theory is "now discredited". APL (talk) 16:46, 26 January 2010 (UTC)[reply]
Appreciate the answers. Thanks. So it's completely beyond the realms of possibility that if/when humans are cloned, that the clones might grow up and experience unexpected 'echoes' of the memories or character traits of the original? --95.148.104.185 (talk) 00:42, 27 January 2010 (UTC)[reply]
That's correct. The clone would essentially be an identical twin. Identical twins can sometimes be quite similar to each other in thought and behavior, though some of this is undoubtedly due to simmilar upbringing. So I suppose that a clone might have certain underlying personality traits that are the same as the original. Anything developed after birth, though, could not show up in the clone. Inheritable genetic material is not changed by the environment, except in extreme cases like exposure to ionizing radiation. The clone could not recognize an old friend of her parent or find her way around her parent's childhood hometown. Buddy431 (talk) 02:17, 27 January 2010 (UTC)[reply]

moonrise

Is there a pattern/schedule of the moon setting and rising like the sun? One night at around 7:00 PM, I happened to see the moon in a short distance from the horizon. On another night at around 7:00 pm, it was already at the "noon" position. --121.54.2.188 (talk) 00:55, 26 January 2010 (UTC)[reply]

Basically, it rises ~49 minutes 50m28s later each time, because it is in orbit around the earth and the earth has to turn a bit farther than one day's worth to catch up. --Tardis (talk) 01:09, 26 January 2010 (UTC)[reply]
Cool, so the moon doesn't only change faces but schedules as well. --121.54.2.188 (talk) 01:19, 26 January 2010 (UTC)[reply]
Not sure what you mean by "change faces". It only ever shows the same face to Earth, but not all parts of it are illuminated all the time. Re the "schedule", did you ever notice that a full moon always rises shortly after sunset? That's when its Earth face is getting maximum illumination from the Sun; but non-full moons rise at other times, when the Earth face is at some angle to the Sun. -- Jack of Oz ... speak! ... 02:54, 26 January 2010 (UTC)[reply]
My bad, I meant phases, not faces.--121.54.2.188 (talk) 05:51, 26 January 2010 (UTC)[reply]
If you are trying to understand what Jack is saying and struggling to work out why it works, try drawing a diagram, it can be very helpful. --Tango (talk) 03:18, 26 January 2010 (UTC)[reply]
The article Lunar phase contains several (potentially) helpful diagrams. -- 174.21.135.237 (talk) 03:59, 26 January 2010 (UTC)[reply]
(I improved my approximation; I had been in a hurry.) --Tardis (talk) 19:58, 26 January 2010 (UTC)[reply]
A short answer is, when the moon is full it rises when the sun sets. Each day after that the moon rises later and later, and wanes, until it rises when the sun rises--the new moon. After that it appears as a crescent in the morning, setting soon after sunrise. Every day it rises later and later, and the waxes larger, until when full it once again rises when the sun sets. The article linked above will explain all this in more detail. Pfly (talk) 09:43, 26 January 2010 (UTC)[reply]
Small correction. A waxing crescent moon rises soon after the sun rises and sets soon after the sun sets. It is in the sky for most of the day, but not easily visible in daylight; it is most visible around dusk. A waning crescent moon rises shortly before sunrise and sets shortly before sunset. It is also in the sky for most of the day, but most easily visible around dawn. Gandalf61 (talk) 13:43, 26 January 2010 (UTC)[reply]
The position of the ecliptic changes depending on the time of night and the time of year, and the moon changes its devriation (distance) from the ecliptic as well. ~AH1(TCU) 20:19, 27 January 2010 (UTC)[reply]

Red bull exhaust pipe and rust

http://thereifixedit.com/2009/11/26/red-bull-gives-you-ignition/

One of the commentators said that at least the pipe won't rust since the red bull can is made of aluminum. But would it only hasten the deterioration of the exhaust pipe by acting like a Sacrificial anode or something.

By the way I think Steve would enjoy (or get horrified by) the "fixes" given in the site.--121.54.2.188 (talk) 06:01, 26 January 2010 (UTC)[reply]

Looks like it's well away from the outlet manifold so heat (melting/softening) shouldn't be an issue. Looks like a good job actually, for temporary use. If the 'aluminum' acts as the anode it will corrode, not the steel pipe. If the pipe is mild steel the Sacrificial anode article under examples says "protection of voids in the glass lining of mild steel water heater tanks via use of magnesium or aluminum alloy anodes". Not 100% sure of this. Depends on exactly which dissimilar metals are involved 220.101.28.25 (talk) 11:08, 26 January 2010 (UTC)[reply]
Yeah, that's right. Aluminum has a much higher reduction potential than steel. So what will happen is the Aluminum will corode, and the iron will stop rusting. I have lost several good aluminum pots and pans when I made the mistake of keeping them in the same drawer as steel ones; when the steel ones started to rust, even a little bit, it caused the Aluminum pans to develop a nasty black film (aluminum oxide) which rubbed off on everything, and resisted washing off. I eventually ditched the pots, and learned to keep those materials in seperate locations. --Jayron32 13:32, 26 January 2010 (UTC)[reply]
I've seen worse "fixes". My concern with this "fix" is that the connection between the can and the pipe doesn't look gas-tight - so in all likelyhood it's leaking nasty carbon monoxide into places where it can get sucked into the cabin. The vibration that's transmitted between the bucking/shaking engine and the fixed-to-the-frame tailpipe will find the weakest point. In a very short amount of time, the can will be bent enough it won't be a tight fit and then the occupants of the car will start losing brain cells to CO poisoning (although evidence suggests that this had already happened when they were considering the fix!) Also, there is scope for the can to melt - although it's hard to predict that without knowing a lot more about the vehicle. Exhaust pipes are made from heavy gauge steel - not thin aluminium. Car manufacturers would certainly use thin aluminium if they possibly could. The fact that they don't speaks volumes about the suitability for the job! This is the kind of botch that I'd certainly consider as a "get me to the nearest garage on a dark and stormy night" fix...but no more than that.
It's the 'invisible' fixes that are most worrying. When I bought my 1963 Mini, the previous owner had started to restore it - and had not understood that the pitch of the threads on the 'whitworth' bolts that British cars used back then is not the same as the thread pitch on US cars. Hence, when he replaced a nut or a bolt, he would - with 100% reliability - strip the threads. Of the 16 lug nuts holding the wheels onto the car, 8 were stripped in this way. One wheel had three out of four stripped - not one wheel had four good lug nuts! All of the bolts on all of the shock absorbers were stripped. His efforts to add seatbelts to a car that was manufactured without them (and without hard attachment points to which to add them) were...um...creative and exciting. SteveBaker (talk) 13:48, 26 January 2010 (UTC)[reply]
Does this mean I can stop any iron thing from rusting - handtools for example - by attaching some aluminium cooking foil to them? Or wrapping them in aluminium foil? 92.24.54.79 (talk) 22:13, 26 January 2010 (UTC)[reply]
Certainly not! If it did then every car would come with a small zinc plate attached to the bodywork someplace and cars simply wouldn't rust. Since that doesn't happen - we may deduce that there has to be a catch. On the other hand, this kind of thing is done routinely in steel ships - see Sacrificial anode. What that article says is "For this mode of corrosion protection to function there must be simultaneously present an electron pathway between the anode and the metal to be protected (e.g., a wire or direct contact) and an ion pathway between the anode and the metal to be protected (e.g., water or moist soil) to form a closed circuit; thus simply bolting a piece of active metal such as zinc to a less active metal, such as mild steel, in air will not furnish any protection." - so it works for ships that are bobbing around in salty water - but not for cars that are exposed to highly non-conductive air. SteveBaker (talk) 06:19, 27 January 2010 (UTC)[reply]
Cars are exposed to rain water, humid air and any amount of electrolyte solutions of acid pollution and road salt. Zinc in the form of a layer of galvanising or primer protects steel from corrosion. Cuddlyable3 (talk) 13:40, 28 January 2010 (UTC)[reply]

is this a valid question here?

Is this desk the proper place to ask a physics question? Is there perhaps a better place where physics discussions take place, such as the argument pages there are concerning mathematics? --Neptunerover (talk) 08:46, 26 January 2010 (UTC)[reply]

General Physics goes here, but Mathematical Physics (which looks just like Maths) goes on the Maths Desk, I would say. Dbfirs 09:58, 26 January 2010 (UTC)[reply]
Okay then, my next question concerns the form of the question I want to ask, since I want to make sure I am not asking what would be an inappropriate question, no matter on which desk I ask it. I'm wishing to make a short statement that I consider valid, after which I'm hoping to be offered suggestions concerning my perceived validity of the statement, since I'm wondering if perhaps there might be things I am not taking into consideration. --Neptunerover (talk) 10:31, 26 January 2010 (UTC)[reply]
The intention of the Wikipedia Reference Desks is to respond to factual questions that can be answered by reference to Wikipedia articles, reliable sources outside of Wikipedia, or, occassionally, through the previous experience of individual editors. The reference desks are not intended to be a chatroom, a soapbox for promoting individual opinions, or a forum for debating controversial topics - see Wikipedia:Reference desk/Guidelines. Whether your question will meet these criteria or not depends upon the contents and context of your "short statement". Gandalf61 (talk) 11:16, 26 January 2010 (UTC)[reply]
Please don't keep us in suspense Neptunerover. Ask if it's suitable 220.101.28.25 (talk) 11:24, 26 January 2010 (UTC)[reply]
If you want to know if your understanding of existing physics is correct, then ask away. If you want to ask if your own theory that you have come up with might be correct, then you'll need to go somewhere else. --Tango (talk) 12:57, 26 January 2010 (UTC)[reply]
Is this desk the proper place to ask a physics question? - Yes. Physics is science and this is the science reference desk and our job here is to answer questions.
Is there perhaps a better place where physics discussions take place, - Yes there are better places to hold discussions. Discussions (as in: general chit-chat) are not encouraged here - we're here to provide answers to specific questions. Sometimes we do get derailed into discussions but that's not really supposed to happen. So there is undoubtedly a better place (outside of Wikipedia) to hold discussions (as opposed to questions).
SteveBaker (talk) 13:27, 26 January 2010 (UTC)[reply]

I wouldn't say that I'm looking for a discussion, but rather just input. It's not like I generally have very many people of intellect around me who I can bounce things off of, not that anyone here would know that. So I'll make my statement and answer anything if I am asked anything, but I'll do what I can to avoid expressing anything else. --Neptunerover (talk) 16:17, 26 January 2010 (UTC)[reply]

The direction of the force of gravity is reversible depending upon how one looks at it. While this may seem to go against everything we've ever been taught, the direction of the force of gravity is not necessarily 'down', but just as well it can be considered as a force going 'up'. Not to say that gravity would make things go up, but what is meant is that because of gravity's upward force, things go down. No matter how counter intuitive the idea may seem, this idea of gravity being based upon a constant upward acceleration is well supported by General Relativity. Is the Earth pulling us against it, or is it pushing up against us? In either case, the math is exactly the same. I think pushing upward explains the force better than a weird suction downward, considering that if the Earth is pushing against me, it makes sense that I should be held firmly against it. However, if the earth is pulling me toward it, how is that accomplished without a rope or tether of some type? Occam's Razor says that if you can explain something simply, stick with that. (Important note: How 'pushing outward' might be accomplished in 3 dimensions is not part of the subject of the preceding statement, and should not be considered when judging its validity, please.) --Neptunerover (talk) 16:17, 26 January 2010 (UTC)[reply]

For Earth to be pushing against everyone on its surface at once, it would have to been expanding outwards at a rate of 9.8 m/s, wouldn't it? Which it demonstrably isn't. And I seem to recall this conclusion being reached last time you tried to discuss this. Vimescarrot (talk) 17:33, 26 January 2010 (UTC)[reply]
In addition to the above, "pushing" fails to explain orbits. Also, let's be clear about what Occam's razor really is. It's a recommendation (and nothing more) to prefer (not "always select") the explanation with the fewest unsupportable assumptions. Leaving aside the initial caveats, "ignore the problems of three dimensions" is a pretty major unsupportable assumption. — Lomn 19:10, 26 January 2010 (UTC)[reply]
Gravity is a natural phenomenon by which objects with mass attract one another. The OP talks about Earth's gravity but it can be demonstrated that all objects have gravitational attractions to each other. Classical mechanics gives Newton's law of universal gravitation. The invisible force of attraction between two objects is a pull in the direction of the other object and therefore when one changes one's reference frame from one object to the other, the pull direction reverses. It is however the same attraction. If the OP is making the point that gravity both pulls the Earth towards Neptunerover and pulls Neptunerover towards Earth then that is correct. It is consistent with what is taught and there is no good reason to invoke General relativity, a need for a rope or to suppose another explanation would better suit Occam's Razor. Kepler's laws of planetary motion require Newton's laws to work in 3 dimensions, which they evidently do. Cuddlyable3 (talk) 19:13, 26 January 2010 (UTC)[reply]
Hmm... reconsidering this, it looks more like you're just redefining all forces to "go" in the opposite direction. I think you'll find this a far less intuitive approach when carried to its logical extremes, even if you find it sensible in the case of gravity. Pushing boxes south to make them move north? Friction in the direction of motion as a braking effect? Nope, I don't think it'll fly. — Lomn 19:16, 26 January 2010 (UTC)[reply]
Well, in the case you mention, pushing the box south cause the box itself to push north, no matter how unsuccessfully its pushing proves to be. The box may not move north, but that is the direction of its force based on its inertia or friction or whatever. --Neptunerover (talk) 06:59, 27 January 2010 (UTC)[reply]
GR tells us that gravity and acceleration are equivalent locally. That means a small object can't tell the difference between the two situations without observing distant objects. If you do observe distant objects, the difference becomes immediately apparently - acceleration is only equivalent to a uniform gravitational field, and the Earth's isn't. Objects on different parts of the Earth are accelerated in different directions (all toward the centre of the planet), that means the Earth would have to be accelerating in different directions, which isn't possible without it being ripped apart. --Tango (talk) 19:38, 26 January 2010 (UTC)[reply]
Consider a sphere with a painted stripe around the middle (like one of the striped balls here). The width of the stripe is constant, but the boundaries of the stripe curve outward, locally, at every point. If that's hard to visualize, note that the standard of straightness is a great circle and the boundaries of the stripe are not great circles. A great circle tangent to the boundary at one point would pass through the colored region and be tangent to the other boundary at the antipodal point. If you think of different cross-sections of the stripe as different times, the great circle (a geodesic) oscillates back and forth from one side of the stripe to the other, like a ball dropped through a hole in the Earth in that old thought-experiment. This is very closely analogous to what happens in general relativity. Every point on the surface of the Earth is accelerating outward, but the overall size doesn't change because of the spacetime geometry. If Earth was uniformly dense and spacetime was Euclidean then Earth's internal geometry would be exactly the geometry of the painted stripe (with two more dimensions added). It's different only because the density isn't uniform and spacetime geometry isn't quite the same as Euclidean geometry. -- BenRG (talk) 23:27, 26 January 2010 (UTC)[reply]
Well, yes, you could say the Earth is expanding in some really weird coordinate system and, technically speaking, the weird coordinate system is just as valid as the more familiar ones, but I don't think that really helps the OP. When we say "expand", without qualification, we mean expanding with respect to conventional coordinates. --Tango (talk) 23:35, 26 January 2010 (UTC)[reply]
Question (please ignore if required) Does the overwhelming amount of astronomical data evidencing the expansion of the universe itself tell us how to qualify its coordinates? --Neptunerover (talk) 04:39, 27 January 2010 (UTC)[reply]
I think Ben and Tango are talking about equivalent coordinate systems, which they are just different ways of saying the same thing, and neither one is more correct than the other. You might also be interested in Shape of the Universe, although it sounds like you might have had some exposure to this type of thing already. Buddy431 (talk) 04:55, 27 January 2010 (UTC)[reply]
Thank you. Yes, I've looked at that previously, although I will admit I'm a skimmer, so I'll go over it more thoroughly now, since it doesn't seem overly complex with density of details beyond my instant interest. My curiosity is that perhaps an artificial coordinate system is being imposed on the astronomical data in order to get a 'big bang' out of the evidence. --Neptunerover (talk) 05:09, 27 January 2010 (UTC)[reply]

"Cosmology still lacks a definitive theoretical model for inflation..."

— Alejandro Jenkins and Gilad Perez, Looking for Life in the Multiverse, Scientific American, January 2010 --Neptunerover (talk) 05:19, 27 January 2010 (UTC)
Re: artificial coordinate system: No, no coordinate system is being imposed to find evidence of the big bang. Due to the nature of expansion, there is no one direction you can point and say "the big bang happened that way". Rather, the cosmic microwave background radiation, our best direct evidence for the big bang, is observable in all directions. The big bang happened everywhere, and no coordinate system is required (or meaningful). As for the quote about inflation, it's likely that the authors are referring to the lack of why/how inflation occurred, not doubts that it did occur. — Lomn 14:03, 27 January 2010 (UTC)[reply]
Thank you. --Neptunerover (talk) 15:19, 27 January 2010 (UTC)[reply]
I think the Cosmic Background Radiation is evidence of a two dimensional universe, which could explain, or at least evidence, my weird ideas about gravity (evidence that they are weird ideas). --Neptunerover (talk) 15:37, 27 January 2010 (UTC)[reply]
On your second note, yes, the context of that quote is not in any way doubting the fact of there being inflation. I believe it is a why/how reference to the big bang model (along with various other models) not being definitive due to various deficiencies as the theory currently stands. --Neptunerover (talk) 15:29, 27 January 2010 (UTC)[reply]
Re: "deficiencies": while there is no doubt that mechanical gaps in the how/why exist, the CMB is one of the best-measured phenomena in all of science. Big bang cosmology is extremely well-attested. — Lomn 15:58, 27 January 2010 (UTC)[reply]

Refrigeration not supported at low ambient temperature

Resolved

I've read the article on refrigerators which seems to confirm my understanding of how household fridges work. I've also searched the archive but couldn't find an answer to my query. I have a Smeg fridge (http://www.smegtech.com/site/smeg/pdf_libretti/914773907-GB.pdf) which is currently in an unheated garage. I wanted to use it for extra capacity around Xmas but, although it clearly had power (the interior light worked) there was no sign of the compressor(?) starting up. On checking the manual, I found that it is only designed to work if the ambient temperature is at least 16c. At the time it was probably about 2c. Firstly, why is that? I'd have thought that it would assist the fridge in dissipating heat if the ambient temperature is cold. Would the refrigeration cycle not work properly or could some harm come to the device? Is it likely that the device has a cut-off so that it won't 'start-up' if the temperature is too cold? --Frumpo (talk) 11:02, 26 January 2010 (UTC)[reply]

That's a normal problem. The refrigerator requires outside warm air to evaporate the liquid that's flowing through it (when it evaporates, it cools), which will return to the refrigerator, be compressed by the pump, and absorb more heat (it's a cyclical thing). If the outside air temperature gets too low, it won't efficiently evaporate the gas, which effectively halts the cycle. See Vapor-compression refrigeration. If your refrigerator cuts out at 16c outside air temperature (which will quickly become inside air temperature), you will want to be very careful about what you eat from it - there are many foods that should not be stored at such a high temperature. Falconusp t c 12:44, 26 January 2010 (UTC)[reply]
If you garage is consistently around 2C over the xmas period, then you don't need a fridge - just put stuff in the garage as it is. If the garage spends a significant amount of time at temperatures between about 5C and 16C, then you have a problem - it's too warm to do without a fridge and too cold for the fridge to work. A lot of people keep extra fridges or chest freezers in their garages without problems, though, so I guess there are ones out there that can handle the low temperatures. Maybe you just need to get a different fridge. --Tango (talk) 12:55, 26 January 2010 (UTC)[reply]
Thanks for the information but I'm still rather confused. I thought the evaporation and cooling happened as a result of the decrease in pressure beyond the expansion valve. The article on vapor-compression refrigeration says that additional evaporation occurs when warm air inside the fridge is passed over the evaporator but why do we need warm air outside the fridge. Introducing heat from outside seems contrary to what we're trying to achieve.--Frumpo (talk) 10:12, 27 January 2010 (UTC)[reply]
Because we need the liquid to turn to gas in order to be compressed. If the outside temperature is too low, this will not happen efficiently and therefore the compressor may be trying to compress a gas/liquid mixture, which it is not designed for. As has been suggested, just turn the fridge off and put your food in to keep it cool. --Phil Holmes (talk) 12:05, 27 January 2010 (UTC)[reply]
OK, I understand now. The fridge is just being stored in the garage - I don't normally need to use it but couldn't figure out why there was a minimum operating temperature. Now I know. Thanks all.

←undent
(I was researching this before your last post so, here it is.)
I was speculating that perhaps this particular fridge was specifically designed to work in a country where the ambient temp. is at the higher end of 'normal' (always(?) above 16° C). And therefore not able to operate in more temperate climates. Had a look at the manual on p.4/sec. 3.1, as you said, climate class "N (Normal) from + 16° C to + 32° C" so there are models for different climates, but not down to +2°. Same page says "Never ..... install it outdoors",p.4 and p.5 "The appliance is specifically constructed for domestic use". It is not outdoors, but the manufacturers appear to have assumed that you would have it 'indoors' in a kitchen (not garage), where presumably < +16°C is unlikely. However even in warm Australia, we get below that and 'garage fridges' seem to work.

Checking the refrigerant used (R600a), according to this p.2: "The properties of R 600a differ from other refrigerants commonly used in household applications,". See Also: Refrigerant
Normal boiling point in °C
R600a @ -11.6°C ...isobutane
R134a @ -26.5°C ...1,1,1,2-Tetraflouro-ethane
R12... @ -29.8°C ...Dichloro-difluoro-methane
So it seems to boil at a significantly higher temp. than the other gases. I can see how this could raise the minimum temperature at which it can work effectively. If your fridge used R12, then its minimum should go 18°C lower This might explain why other fridges apparently have no trouble working in similar conditions. Note I am not an expert in this area. See Also Heat pump and Refrigeration for more detailed explanations of fridge operating principles.
220.101.28.25 (talk) 15:25, 27 January 2010 (UTC)[reply]

Thanks very much. I started looking at refrigerant boiling points but it's hard to know what pressures are involved in the various parts of the fridge and what the corresponding boiling points would be. However, as you say, my fridge seems to be filled with a refrigerant that's liquid at temperatures when other refrigerants would be vapour and, as noted above, that's no good for the compressor.--Frumpo (talk) 18:01, 27 January 2010 (UTC)[reply]
Welcome!. You seem to be a 'victim' of the desire to reduce the use of CFCs and similar chemicals to help save the ozone layer. Speaking of 'garage fridges' the one I'm most familiar with is an old one about 40 years old, which is undoubtedly filled with nasty CFC and works merrily in hot (40°C+) conditions, and would almost certainly get down to 16°C in winter in Sydney (but not 2°C). Be interesting to compare the specified operating range of such an 'old school' fridge to yours. Only other thing I could suggest is to look into what gas is used in places that are cold even in summer. PS. How long did you leave the fridge on when you tested it? Possible you simply didn't wait long enough? ;-)? We have Computer Engineers, Neuro-scientists, doctors, Rocket Scientists and even a seagull specialist(!) answering queries here, you'd think we would have a refridgeration technician! 220.101.28.25 (talk) 00:43, 28 January 2010 (UTC)[reply]

Hall effect EEG?

Supposing a conventional EEG was taken using head electrodes. Would a powerful magnetic field around the subjects head, oscillating in a three dimensional raster pattern, alter the "focus" of all the electrodes in such a way as to vastly increase the spacial resolution down to the level of individual brain cells?Trevor Loughlin (talk) 13:58, 26 January 2010 (UTC)[reply]

No. --BozMo talk 14:13, 26 January 2010 (UTC)[reply]
From the title I guess the OP is thinking of using Hall effect sensors as the head electrodes. Hall effect sensors cannot generate magnetic fields so the 3-D magnetic raster scan (interesting notion) would be generated by an array of electromagnets. Focussing the magnetic field would be a difficult design problem and the only use for the Hall sensors would be to measure how well it was achieved. Cuddlyable3 (talk) 14:58, 26 January 2010 (UTC)[reply]
The article on Transcranial magnetic stimulation might be helpful here. It can't be used for the proposed purpose, but it might give you an idea of what would happen if you created such strong magnetic fields. Looie496 (talk) 16:52, 26 January 2010 (UTC)[reply]

I was thinking of normal electrodes, not Hall Effect sensors.The Hall Effect would take place in the brain material itself due to the altering magnetic field (I hope) slightly changing the brain areas all the EEG electrodes pick up electrical signals from, increasing resolution.Trevor Loughlin (talk) 06:26, 27 January 2010 (UTC)[reply]

Psychological reason for former students talking about how strict/tough teachers were

I was under the impression, and the article on memory seems to agree, that we remember mostly the good things. Obviously, people remember some negatives. Still, on a few alumni Facebook pages for American Junior High Schools, I notice former students, now in their 30s and 40s, talking more than usual (maybe 30-35% of posts) about teachers' discipline and how strict they were in 6-9th grade. My question is, why? Why are they choosing to discuss this when reminiscing?

I'm theorizing maybe it's how bad the students were; maybe not all the posters, but perhaps they're remembering some really rough kids but can't recall specifics on that - but they can recall teachers and how they handled it. While the ones I'm reading are from a city of 100,000 - not exactly a crime-riddled urban area - I'm sure the neighborhood plays a part.

It just seems strange that so much time is focused on negative thing, instead of different assignments, social activities, and so on; which are mentioned, but not as much. It especially seems strange because of the notion that "what's too painful to remember, we simpy choose to forget." (Okay, the line is fromt he song "The Way We Were," but you know what I mean.)209.244.187.155 (talk) 14:40, 26 January 2010 (UTC)[reply]

I don't know that there's a psychological reason for it, but I can give you a social one - happy stories are boring. Nobody wants to hear "Oh man my teacher was soo much better than yours" for very long, but everybody wants to swap/compare horror stories. Every tell a story about getting injured? A group of friends can go back and forth telling different, painful stories for an hour. Same thing - it's rude and annoying to try and compete for "best" but if you're competing for "worst" the only person who loses is you. School-age people also do it with sleep - "I only slept five hours last night." "Yeah well I got 3.5!" "Lame, I pulled an all-nighter." Someone saying "Damn, I slept 9 hours last night I feel great" is no fun. ~ Amory (utc) 14:47, 26 January 2010 (UTC)[reply]
Presumably because their teachers were strict. And at that critical age, the conduct of adult figures in your life can have a profound influence on your psyche for a very long time. So I guess it's just 'digging in the dirt' as Peter Gabriel puts it. Vranak (talk)
Perhaps they have children in school now and they are witnessing the dumbing-down of the school system. I see it at a college level. I have students who invariably made straight A's. They almost all took advanced courses. They scored very high on the SAT. Then, when I show them long division (to explain what modulo means), they say that they've never seen it. That is just one example. It also shows in the fact that the exams for the class have not changed in the last 10 years, but the scores on the exam go down every year. They used to be above 90% for most students. Last semester, the average was around 65%. It is clear from my perspective that there is a dumbing-down in the school system. I don't blame the students. They are not genetically dumber. I could blame less-than-strict teachers, but I don't. I blame the parents who feel it necessary to give A's to students without requiring them to earn the grade. -- kainaw 14:57, 26 January 2010 (UTC)[reply]
To be fair, long division is a skill that's fading fast. I'm trying to recall the last time I had to do it - and I'm pretty sure it was over a decade ago - and I'm someone who does a LOT of math and arithmetic. It's worth teaching it for lots of reasons - but expecting everyone to know it well is asking a lot for what is essentially an unneeded skill. We don't teach kids to use slide-rules either. SteveBaker (talk) 15:11, 26 January 2010 (UTC)[reply]
It's very hard to compare student ability in 2010 to student ability in 1965. A child nowadays is a computer expert - in 1965, if they'd heard of computers, you'd be lucky! Sure, they waste some of that technical expertise on video games (Sorry Steve), various online garbage, but they also know how to type, how to use a word processor, how to query the internet for information. Their skills toward those goals are greatly enriched over those of a student in 1965, who consequently had more mental acuity to focus on "pure" subjects. So, while there aren't all that many 9th grade hackers (despite what you see in the mass media), there is an enormous repository of cultural and institutional knowledge about computers which doesn't get graded. Technological expertise is only one example of a huge class of useful skills which are never graded. When you cite lower total scores on tests, without accounting for the redirection of intellectual activity to other, un-tested subjects, you're failing to account for this sort of skill-set displacement. In other words, our kids aren't stupider, they're just focused elsewhere.
I sincerely hope that this does not mean focus is lost on core subjects I consider critical - basic math and literacy, for example. But to some extent, this defocusing is not a total loss. I grew up wasting a ton of my time on video games - and I learned a ton about graphics acceleration, programming design, and computer architecture by proxy. I was never tested on those concepts in a math class, or even in a programming class. Yet, now, when I encounter older programmers, traditionally trained in more conventional ways, who could probably best me in a long-division competition, they're often unable to discuss the merits of PCI-X vs. PCIe - or even comprehend this entire way of thinking about program design. But, that skill is very relevant in today's computer engineering job market - a lot more so than many concepts that I was formally tested on during schooling. I imagine the same argument can be made for a variety of other extra-educational knowledge that is acquired.
So, while it may be accurate to say that the scores lower on the same tests as they are given year by year, this line of reasoning suffers from a fundamental flaw. If the test does not change from year to year, it implicitly assumes that the same material is relevant from year to year - that performance of the same skills of the previous generation is a merit. In some sense, that encourages stagnation and repetition - if our kids do the same thing we do, but only incrementally better, then what progress has been made? As much as it's a tear against a dumber or less-motivated populace, it's equally an indictment of an education system that does not know how to adaptively adjust its testing to normalize for relevance and currency.
This is a very hard problem - we aren't going to just throw formal education out the window (well, we could, and many prominent philosophers such as Ivan Illich suggested that we should). But there has got to be a happy medium - an education system which adjusts to current needs, accepts that society isn't going to collapse just because long division, aether theory, and FORTRAN 77 are dying arts, and trains kids for what they need to know today. Nimur (talk) 15:33, 26 January 2010 (UTC)[reply]
Tossing out long division is acceptable, but having students perform worse every year at college level is not acceptable. Instead of improving education, we do things like "recenter" the SAT to make the less-educated students feel better. There is no genetic reason for modern students to perform worse on the SAT. They should perform better than those students in the 50's and 60's who didn't have computers and the Internet to fill them with valuable information. The fact that they don't implies that something is broken. Some blame the less strict teachers who just shove the students from one class to the next. I already stated that I blame the parents. -- kainaw 15:55, 26 January 2010 (UTC)[reply]
I also want to point out that this question is about why 30-40 year olds state that their teachers were stricter. It is not about how smart students are. My answer to the question leads to the debate about how smart current students are, but that is not the topic here. Further, this is not a forum for debate. So, please feel free to disagree with me and tell the person next to you what an idiot I am, but don't make my answer a means of hijacking the thread to debate modern vs past students. -- kainaw 16:04, 26 January 2010 (UTC)[reply]
Sorry, I am probably guilty of WP:SOAPBOX, in at least a few recent posts. I will try to curtail this tendency. Nimur (talk) 08:15, 27 January 2010 (UTC) [reply]
The point I think some people are trying to make is that "recentering" the SAT is not, in itself, a bad thing at all. A hypothetical SAT from a century ago would have little meaning to us now (Latin conjugation, what the hell?), so it shouldn't be surprising at all that it would constantly be in need of shifting. Modern primary school math places a heavy emphasis on learning how to approximate answers and very little on obtaining the results. To people of my age that seems like a dumb thing to do, but it's simply an acknowledgement that precise mathematics is not used by most people and that calculators can perform any function flawlessly. Approximating that 21/5 is "about 4" is much more useful to most people than doing the long division. If SATs don't keep step with what's being taught, the grades will shift downward. Matt Deres (talk) 17:44, 26 January 2010 (UTC)[reply]


The most basic principle of storytelling is that every good story involves a conflict and a resolution. Without a conflict there is no story. Nice teachers don't tend to generate strong conflicts, so they don't tend to generate good stories. Looie496 (talk) 16:47, 26 January 2010 (UTC)[reply]
Thakns for the answers; I hadn't looked at it from the storytelling/one-upping angle, but yes, that's very likely. Also, considering that a couple of them talked about sneaking out to smoke then, while I don't like to pass judgment on how they acted, maybe some of it is reliving the thrill of trying to away with stuff, too; even if they did get caught at times. I imagine that for some, there is a point to reminiscing about the thrill of trying to get away with stuff, whereas that thrill isn't there anymore now; as adults, they have the freedom, and any stuff they would have to "get away with" would lead ti the threat of jail or large fines if caught; they've matured enough to realize that's not worth the risk.209.244.187.155 (talk) 19:05, 26 January 2010 (UTC)[reply]

Improving Loudspeakers

                            LS1              LS2
             |¯¯¯|  |\        /|            |\
frequency____| ? |__| \___|¯|/ |   Room     | \|¯|__
   sweep   | |   |  | /   |_|\ |            | /|_| |  |\
           | |___|  |/        \|            |/     |__|-\
           |                                          |  \___test
           |__________________________________________|+ /   output
                                                      | /
                                                      |/

I have a stereo audio system and want to improve the sound in my listening room. I think it is possible to compensate for the non-linear frequency responses of my loudspeakers and room acoustics by adding a filter at the preamplifier stages of both channels. I have a graphic equalizer but I hope to get a better result than from adjusting it by ear. If I could measure the exact correction needed then I could optimise the equalizer settings or possibly build a better filter. I propose to measure the audio performance as shown in the diagram where:

"?" is the equalizer or filter to be designed
LS1 is one loudspeaker that is driven with a swept frequency test signal
LS2 is an identical loudspeaker operating as a microphone.

I assume that the loudspeakers' frequency responses are the same whether they are driven at low level or used as a microphone. I intend to adjust the stage "?" to minimise the test output at all frequencies. That should compensate for the room acoustics. But can you suggest how to handle the following snag with my plan: the optimised stage "?" will introduce twice as much compensation as is needed for the non-linearity of a loudspeaker. How do I design a compensator for one loudspeaker? Cuddlyable3 (talk) 14:46, 26 January 2010 (UTC)[reply]

Back in the early 1980's Philips used to make "Motional feedback speakers" that did more or less exactly this - so what you're trying to do is certainly possible...and with 1980's technology too! However, you'll need to allow for the time delay in the audio getting from LS1 to LS2 - sound travels V-E-R-Y--S-L-O-W-L-Y compared to electricity! You're definitely going to need either some phase compensation or a programmable delay between the input on the left and the comparator on the right. You might also look at Powered_speakers#Servo-driven_speakers - where you use an accelerometer to directly measure the motion of the speaker cone in order that you may compensate for it's frequency response - although (obviously) that doesn't take account of the room acoustics. SteveBaker (talk) 15:06, 26 January 2010 (UTC)[reply]
Nitpick, Steve - sound travels slowly compared to the electromagnetic signal in the other wire. Nimur (talk) 15:10, 26 January 2010 (UTC)[reply]
What does it mean when SteveBaker writes "for it is frequency response" ?
We do (of course!) have an article about Motional Feedback speakers - but it's not exactly helpful! SteveBaker (talk) 15:45, 26 January 2010 (UTC)[reply]

Motional feedback is interesting, expensive, has only been applied to low-frequency (woofer) speakers and it is not what I propose. The block "?" will provide forward correction and not introduce feedback. I shall disregard phase, sound delay LS1 to LS2 and longer delays in the room acoustics by making the test comparator compare powers of the two signals and the frequency sweep will be slow. The ear is relatively insensitive to constant phase error so phase changes that "?" introduce will probably not be noticed. In any case a phase linearising stage can be added to "?" if that is found necessary.
Where is the snag is with the following procedure? 1) Adjust "?" until test output is constant (or zero with gain adjustment) for all frequencies. 2) Build two compensator circuits that each implement half the distortion of "?" and attach them to the two channels. 3) Now I have a system where speaker non-linear frequency responses are fully compensated but room acoustics are half compensated. 4) Run the measurement again (one of the compensators is in the "microphone" channel). Only the remaining uncompensated room acoustics need to be corrected and that can be done by adding the second "?" in full to both channels.That sounds unclear but at least Nimur I understand what SteveBaker means by electricity.Cuddlyable3 (talk) 17:02, 26 January 2010 (UTC)[reply]

Tyre pressure

If you inflate a car's tyres to a certain pressure and then add weight to the car, does the pressure in the tyres increase, or does the pressure remain the same because the tyre has retained the same volume, albeit that it has deformed slightly? I imagine that if the tyre were to be equally compressed on all surfaces (say by taking it underwater) then the internal pressure would increase, but I am not clear if compression on one part of the surface would be compensated by the expansion on other parts of the surface leading to static volume and pressure. Any help appreciated. AChangeOfPressure (talk) 15:02, 26 January 2010 (UTC)[reply]

Pressure depends on the force and area ,force is mass * acceleration so if the Mass is increased the pressure increases .If by compression you mean applying force then the pressure increases--NotedGrant Talk 16:08, 26 January 2010 (UTC)[reply]
Not sure if I was sufficiently clear in my original question, so let me clarify. Imagine an example where I inflate the four tyres of a car to a pressure of 30 PSI, then I add a significant amount of weight to the car (in terms of passengers, luggage etc) equally distributed across the four tyes and then take a new reading of the internal pressure of the tyres with a pressure gauge. Will they have increased above 30 PSI? Thanks AChangeOfPressure (talk) —Preceding undated comment added 16:19, 26 January 2010 (UTC).[reply]
There are two effects happening at the same time. Increased load will deform the tyres; if this reduces their volume then pressure in the tires will increase. But at the same time the size of the contact patch is increased, so the load is supported over a greater area. So if you add a load that is 10% of the weight of the car, the tire pressure will not necessarily increase by 10% - it may only increase by 5%, while the area of each contact patch also increases by 5% (yes, I know there is a simplification there). I don't know the relative sizes of the two effects in practice - it probably depends on all sorts of factors such as the size and design of the tyres. Gandalf61 (talk) 17:07, 26 January 2010 (UTC)[reply]
For the vast majority of tires, the load is carried almost exclusively by the increased contact patch area. It takes a great deal of deformation to increase the pressure even slightly (a quick mental calculation says that flattening the typical tire so that the rim is in contact with the ground will increase the pressure by 10%). --Carnildo (talk) 00:50, 27 January 2010 (UTC)[reply]
As weight is added to the car the tyre pressure increases. As weight is added to the car the tyre deforms so that the internal volume of the tyre decreases. The reduced volume causes an increased pressure. The area of the contact patch also increases in size. Dolphin51 (talk) 06:42, 27 January 2010 (UTC)[reply]
Yes, that sounds correct, but I think that the percentage difference in the air pressure inside the tyre will be minimal because tyres are designed to make the change in volume with extra weight as small as possible. A change in temperature makes a much bigger difference to the tyre pressure. The pressure of the tyre on the road surface is a separate argument (only slightly linked to the air pressure in the tyre because of deformation). This is only a wild guess, but, within the design limits of the tyre, doubling the weight supported probably increases the tyre pressure by 1 or 2%, but almost doubles the contact pressure with the road. Can anyone find any test data? Dbfirs 07:21, 27 January 2010 (UTC)[reply]

Human Underwater Speed Record

What is the world record for speed acheived by a human underwater? I mean unassisted though I suppose they could be in some sort of human powered sumarine, wetsuit with fins, etc. TheFutureAwaits (talk) 16:44, 26 January 2010 (UTC)[reply]

That might depend upon a lot of things, such as water salinity and turbulence, etc. --Neptunerover (talk) 13:24, 27 January 2010 (UTC)[reply]
The closest thing I've seen is the 50m freestyle, where the world record is presently 2.4 m/s. I'd guess that this provides a reasonable balance of advantage from push-off with disadvantage from mandatory surfacing. It'll be hard to find anything else that's certified, but this guy's human-powered sub claims about 3.6 m/s. You might also consider the entries in our article on human-powered watercraft. — Lomn 19:32, 27 January 2010 (UTC)[reply]

Evolution

What proof is there for evolution? How do I know it isn't just an atheist hoax? --J4\/4 <talk> 16:54, 26 January 2010 (UTC)[reply]

First, read the article on evolution. It will detail some of the arguments that have been made. This is the subject of a huge amount of scientific and popular literature and it would be impossible for us to provide all the information here. If you have a specific question regarding evolution, please re-phrase your question. --- Medical geneticist (talk) 16:59, 26 January 2010 (UTC)[reply]

Do I look like I would hoax you? File:Ape shaking head.gif

Cuddlyable3 (talk) 17:27, 26 January 2010 (UTC)[reply]
Also, I think it's been done. Vimescarrot (talk) 18:01, 26 January 2010 (UTC)[reply]
The largest organized Christian denomination in the world has stated there is no conflict between any part of the scientific theory of evolution and christian faith. Many Christians of many other demoninations also see no conflict, nor do many members of other faiths. Many scientists are also devoutly religious, and have no problem with the fact that evolution is happening, and also having a devout faith. The conflict is a false one, perpetuated by people who need there to be a conflict for their own selfish reasons. --Jayron32 21:33, 26 January 2010 (UTC)[reply]
One thing I would like to add is that your primary premise is very flawed. Not all people who believe in evolution are atheists, not by a long shot, even Darwin wasn't strictly an "atheist", he didn't believe in a personal God. In fact, Catholics who don't believe in evolution are a minority, even in the US 58% of Catholics answered a poll that they believe evolution to be the best explanation of the origin of human life on earth. An even larger percentage of Jews, Hindus and Buddhists believe in evolution and even as much as 45% of Muslims in the US. We even have an article about it: Theistic evolution. Vespine (talk) 22:14, 26 January 2010 (UTC)[reply]
I assume you must be talking about the OP. I have never, here or anywhere, made the claim that there is any connection between the theory of evolution and athiesm. Quite the contrary, if you read my comments, I make the exact opposite arguement. Could you clarify the antecedant of your pronoun, por favor? --Jayron32 22:28, 26 January 2010 (UTC)[reply]
There is no strict "proof" in science, only theories and evidence. The evidence for evolution, however, is overwhelming. The Origin of Species lays down the basic case so convincingly that it has nearly immediately been widely accepted - despite the fact that at the time no hereditary mechanism supporting evolution was known, we had few fossils, and no genetic evidence. Indeed, Darwin comes as close to a proof as science can come - geometric increase of populations, finite carrying capacity of habitats, and variability of heritable traits more or less implies evolution mathematically. The suggestion that it is a atheist hoax is entirely unsupported. Many of the past and current researchers supporting evolution are religious people. Many large Churches have accepted evolution. And any conspiracy would need to be so gigantic as to be impossible to keep secret. --Stephan Schulz (talk) 22:23, 26 January 2010 (UTC)[reply]
Proof is everywhere. The evolution of bacteria in hospitals in response to our best efforts to nuke them with antibiotics. Rats that are resistant to rat poison. Rabbits that survive mixamatosis. Lactose (in)tolerance in humans. My current favorite is the Recurrent laryngeal nerve in giraffes. Rows and rows of fossils in just about any decent natural history museum. However, here is a good one:
There is a species of moth in the UK that are normally white - this is because they live on trees with very light colored trunks and it helps them be camoflaged from hungry birds, very rarely, a mutant dark colored moth (of the same species) would show up. When the industrial revolution hit the UK, and everything was run from coal, there was a vast amount of dirty soot in the air and the places where these moths came to rest became blackened. Within a very small number of years, butterfly collectors noticed that the rare dark variety were becoming much more common and the 'normal' light colored ones were becoming harder and harder to find. The moths had evolved an adaptation to all of that sooty pollution. Moreover - when the "Clean Air Act" was passed in the UK and the clouds of filthy carbon-laden emissions more or less ceased, the process reversed itself and the dark colored moths started to vanish with the light colored ones again becoming the most common. Again, the moths had evolved.
I could sit here and type in convincing examples all day - but I'll restrict myself to just one - and recommend that you read almost anything by Richard Dawkins who is truly excellent at providing convincing examples that the "intelligent design" loonies can't possibly counter (which is why they hate him with such a passion - and why our OP will probably never read a book by him).
SteveBaker (talk) 22:40, 26 January 2010 (UTC)[reply]
SteveBaker is referring to Peppered moth evolution, which is a long article that goes into a great deal of detail to creationist alternative explanations, and the political controversy. Comet Tuttle (talk) 23:45, 26 January 2010 (UTC)[reply]
The biggest rationally formulated complaint of the intelligent-design/creationists about the Peppered Moth story is that it only represents "microevolution". Where is the fish turning into a rabbit? Where is the long-eared furry fish? But evolutionists don't claim that a fish turned into a rabbit. Fishes turned into slightly different fishes which turned into slightly more different fishes - and so on, "microevolution" by microevolution until one day, you look at the millionth generation of microevolutionary change and...guess what? It's a rabbit! So those who seek a macroevolutionary proof won't find one. Even evolutionists don't claim that - evolution cannot work in big steps, it requires a vast expanse of time for any kind of large change. When pressed on this point, the less rabid creationists will say that there is not one case in documented biology of one species changing into another by an intermediate step. That's also true - but it's another misconception. (I'm going to use the word 'species' loosely to mean "different group of animals") If biologists find two animals that are significantly different, they give them different names. Even if it's just a teeny-tiny microevolutionary step from one to the other...they have different names so it's easier to talk about them. If we find another animal that's a clear intermediary and is different from both its ancestral and descendent species in any measurable way - it gets a different name. Hence you'll never find a clearly intermediate in a biology textbook simply because of the way biologists name things. So this lack of an intermediary isn't a failure of evolutionary theory - it's just a consequence of the way biologists carry out their craft. So the example of the peppered moth is indeed a tiny step - but a tiny step is all we need to establish the theory of evolution as fact.
There are just three things we need to demonstrate:
  1. That mutations sometimes happen. We see this all the time in the more dramatic 'freaks of nature' - but also that not all humans are identical. "Tallness runs in my family" is occasionally confounded by a short offspring.
  2. That 'survival of the fittest' actually works - which we see so clearly in the peppered moth story. When the trees are light colored, light colored moths thrive and black colored moths are hard to find. When the trees change color - the moths that have the best camoflage survive.
  3. That lifeforms inherit significant characteristics from their parents. We all see in daily life, my kid has my eyes and my wife's nose. Photos of me when I was 10 years old are hard to tell apart from photos of my kid at the same age. The offspring of two black labradors is a black labrador.
If all three things are true - then it's very hard to see how evolution couldn't happen. (1) A random mutation makes a change (a black moth in a population of white moths for example). (2) If the change is an improvement (because we have dark trees these days) - then more of the mutants will survive and (3) their offspring will inherit their gene for dark coloration. That's it. That's all it takes to get from a bacterium to a human given a few hundred million generations. SteveBaker (talk) 02:40, 27 January 2010 (UTC)[reply]
If you look at the sort of people who have done work on how evolution actually works—e.g. Ernst Mayr, E.O. Wilson, Stephen J. Gould, Charles Darwin, and so on—rather than the real evolution evangelicals (e.g. Richard Dawkins), what you see is a body of people who have spent an immense amount of time looking at very small pieces of the world (birds, ants, snails, barnacles), compiling lots and lots and lots of mind-numbing data that seems to indicate, in quite a detailed way, exactly how evolution appears to have worked. They disagree with one another on many points—they are not in strict collusion. But it seems like an awful lot of effort to go through for just a hoax. These are people who have devoted their lives to very tedious and basically obscure details that shed light on a bigger whole. This is not, generally speaking, how hoaxsters operate. Whatever they are doing, it is not a hoax. They could certainly be wrong, or could interpret evidence incorrectly, and so on. But their sincerity in the effort is fairly evident in the work itself. They are not trying to pull a fast one—they are pulling a rather slow one, if anything.
If you'd like to know about evolution, read one of the books by any of the aforementioned scientists. Go ahead, it won't hurt. Make up your own mind as to whether they are serious about it. They are subtle thinkers and not one of them is trying to proclaim atheism upon the world.
Dawkins is not so subtle, though he is clever. But he definitely does believe that evolution leads to atheism. Plenty of scientists disagree with him on this point, though. In the end, whether evolution and religion are ultimately compatible is a philosophical/metaphysical question, not a scientific one. Evolution is not compatible with a very literal and narrow reading of the Bible—neither is modern medicine, or really any basic scientific outlook. Whether you see that as a reason to reject science—despite all of its apparent mastery over nature (your computer that you are reading this on right now would not work if quantum mechanics wasn't basically correct)—or whether you see that as an imperative to read the Bible in a more interpretive way, is obviously a personal decision. --Mr.98 (talk) 23:37, 26 January 2010 (UTC)[reply]
Sorry Jayron yes I was talking to the OP. In regards to the above point made by Mr.98, maybe it has been a while but Dawkins has certinally very much done "real work" on evolution. The Selfish Gene is highly regarded as an important work in the field. Vespine (talk) 00:57, 27 January 2010 (UTC)[reply]
It's true that Dawkins doesn't get out in the field or in the lab anymore. What he does is important though. He collects information from the real scientists out in the field and puts together the disparate stories into a coherent and populist whole. There is no way I'm going to wade through a bazillion issues of some obscure journal looking for interesting and convincing examples of evolution in action. In one of Dawkins' books, I can find the story of the pepper moth - and the ridiculous way one of our laryngial nerve is connected up and the complete craziness of the way the giraffe has gotten stuck with such an incredibly unintelligent design! So Dawkins deserves a place on the bookshelf as a collector of stories. (He's also a Wikipedian...he gets bonus points for that!) SteveBaker (talk) 02:53, 27 January 2010 (UTC)[reply]
Evolution precludes original sin and its variants. It precludes the selection of Homo sapiens by a god. To say that it and religion should co-exist is incorrect, I think. 66.65.139.33 (talk) 01:42, 27 January 2010 (UTC)[reply]
There are plenty of religious people who are happy to accept it. There are few atheists who don't. The thing that makes intelligent, open minded people doubt this entire description is that there are two gigantic holes in the scientific account of the universe: What caused the Big Bang? and Where did the first life-form come from? These are questions that science doesn't have good answers for - and it's going to be exceedingly tough to find and prove them. So there is the room for your god or gods. God pushed the big red "GO" button to start the universe running - then (s)he stepped back until the earth appeared with it's nice warm oceans and appropriate chemistry and God pushed the large green "LET THERE BE LIFE" button to cause the first self-replicating molecule to appear, then (s)he steps back and lets evolution produce people. I don't personally believe that story - but it's pretty much the only one that allows religion to co-exist with science. Sadly, this is a "God of the gaps" argument - and it's only a matter of time until one or other of those two gaps gets plugged and this kind of rational religion is left clinging by a fingernail. Will people jump ship to the atheist side of the fence or will the be forced to dump their rational instincts and wind up with the loonies? SteveBaker (talk) 02:40, 27 January 2010 (UTC)[reply]
It's not necessary that god/s had any direct bearing on the 'creation' of the first self-replicating molecules. There's no reason why a religious person can't believe that god/s started the big bang, fully aware of the eventual outcome including the evolution of humans. It's surely fully within the realm of a fully omnipotent god/s.
Evolution does contradict any human creation stories as well as any literal acceptance of the garden of Eden and original sin. However a number of Christians regard much of the bible as allegory [6] and I would say accepting the bible as the entire literal truth raises a lot of issues outside of evolution. Ditto with similar religious concepts.
On the issue of Homo sapiens being chosen, that's not really precluded by evolution. Evolution says humans are just one possible evolutionary outcome and should not be thought of as some sort of goal or ideal outcome. In other words, in a biological evolutionary sense, it's nonsense to think of humans as chosen and god/s had no influence on the development of life forms on earth including humans. However this doesn't preclude god/s selecting that outcome as special, which he/she may have fully know was going to happen. (The question of 'why humans' raises some interesting issues but it's easy to see a number of possible answers.) Note also the idea of god/s having a direct influence in the world is something quite a number of scientists disagree with however it's not something that evolution itself in some way precludes. (Of course doing science generally means automatically discarding such a suggestion when considering anything you want to study scientifically.) It clearly doesn't preclude god/s becoming involved in human society after they evolved, even sending his son (however you think that happened) who sacrified himself for humans.
In terms of 66's point, his/her view of religion is a little simplistic. Not all things often thought of as religions have an original sin, the selection of Homo sapiens by god/s, or even a concept of god/s itself. Of course without any concept of God, you could define a religion as atheist, and that argument is commonly made for Buddhism but on the other hand a number of the spiritual ideas of Buddhism are clearly anathema to many Western atheists, particularly strong atheists.
As a final consideration, it's probably true that many religious people who accept evolution (particularly of the Abrahamic religions) don't do it completely, e.g. they may like to think god/s had some influence in the evolution of humans or even that humans didn't evolve but as I've been emphasising this clearly isn't essential for accepting religion.
Some links that may be of general interest [7] [8] [9]
Nil Einne (talk) 05:44, 27 January 2010 (UTC)[reply]
In my experience, a small number of theists ridicule the writings on natural selection by Darwin and others, saying The theory of evolution is nothing but a theory! Now that Darwin's writings on natural selection have been vindicated by vast amounts of new material from a large number of independent sources, Darwin's thesis can legitimately be called the Law of Natural Selection.
Scientific skeptics and others who aspire to be rational thinkers know that proofs are rare. It is only in man-made sciences like mathematics that proofs can be constructed. In nature, nothing comes with a proof. One could easily say What proof is there that God exists? (I think I know my own name but there is nothing that I would accept as absolute proof of my name - I haven't actually sighted my original birth documentation; my parents could have made errors in the documents; or my spelling or pronunciation of my name could be in error.)
So there will never be a proof that evolution, or Darwin's Law of Natural Selection, is correct. Scientists are happy to live with this situation. It appears to be theists with alternative motives who demand What PROOF is there of evolution?
Is natural selection an atheist hoax? There are many so-called conspiracy theories. Our personal world-view immediately tells us which theories we want to accept and which we want to dismiss as conspiracy theories. It is a legitimate field of enquiry. One aspect which appeals to me is that if a theory requires a very large number of people to know the truth, and a very large number to be ignorant, but the first group to keep it secret from the second group, it is worthy of being dismissed as nothing more than a conspiracy theory. For example, if natural selection is an atheist hoax it requires that a very large number of atheists know that natural selection is nothing but a hoax, but none of those atheists leaks that knowledge to a believer. My experience of small groups of people is that they are hopeless at keeping secrets. If natural selection were nothing more than an atheist hoax the believers would have uncovered the secret over a century ago. The reverse is true - every year brings new knowledge that corroborates the Law of Natural Selection. The religious world of believers continues to be divided. The various major religions of the world agree on almost nothing. Even the multitude of religious denominations choose to remain divided by maintaining a variety of churches, synagogues, mosques etc in every city. There are many who believe in a God but almost none of them show much interest in achieving unity or finding agreement. Dolphin51 (talk) 07:13, 27 January 2010 (UTC)[reply]

I found this talk by Francis Collins' (leader of the Human Genome Project and outspoken evangelical Christian) to give an interesting insight into the mind of a religious scientist. He speaks of the overwhelming evidence for evolution at about 0:44:00 and evidence against both young earth creationism and "intelligent design" at about 1:00:00. He warns against worship of the "God of the gaps", as such worshipers are setting themselves up for a fall when science catches up. 124.157.247.221 (talk) 07:49, 27 January 2010 (UTC)[reply]

Happy Brains à gogo, TMS Experiment

A Boscovich/Nikola_Tesla type Electromagnetic Household Theory. I'd like to prove similarties to the Apple IPOD users with an unawared type of Transcranial_magnetic_stimulation. Lets say some magnetic chemically bonded minerals of sorts is in the American foodstuffs and/or water supply. And the magnetic field of the two Ipod Headphones (if you just hold the two white factory Apple Ipod earpieces together, they stick magnetically) creates similar effects to TMS. As well as Cellphones and Bluetooth tech users. My experiment in this would be, and here's the question in regards, to buy a used Etch-a-Sketch, break open, and put in a Fishbowl full of water. With this magnetic dust, if my Theory is correct, would the Etch-a-Sketch dust form a visible magnetic field in the bowl if I held the two Ipod earpieces at opposite ends? --i am the kwisatz haderach (talk) 17:18, 26 January 2010 (UTC)[reply]

What makes you think that an Etch A Sketch is filled with "magnetic dust"? --LarryMac | Talk 17:22, 26 January 2010 (UTC)[reply]
Pretend he said Magna Doodle instead of Etch A Sketch. (And that just about exhausts my ability to make sense of the question.) -- Coneslayer (talk) 17:24, 26 January 2010 (UTC)[reply]

I don't know, I discriminated it by its gray color. Maybe one of those Magnetic Moustach Pen/Dust toys. So usage of Finely grounded visible Magnetic Particles. --i am the kwisatz haderach (talk) 17:27, 26 January 2010 (UTC)[reply]

TMS requires an incredibly powerful magnet. The magnetic fields created by an IPOD or any other household device are orders of magnitude too small to matter. Looie496 (talk) 17:44, 26 January 2010 (UTC)[reply]

I should add, the reason I brought this up is when I drink Coffee from the Starbucks, at my workstation, I have really good Radio reception. But if I don't drink of the corporate cup, my desk radio has a lot of static. I then concluded that I'm a radiowave receptor, and when I'm full of the metalic minerals, I get good music. --i am the kwisatz haderach (talk) 17:46, 26 January 2010 (UTC)[reply]

We are all radiowave receptors. Some people can hear long-wave amplitude modulated radio transmissions in their fillings! Are you writing a Science Fiction (or fantasy) story? Dbfirs 18:43, 26 January 2010 (UTC)[reply]
Is it possible that you only go to Starbucks on days when it is not raining? APL (talk) 20:20, 26 January 2010 (UTC)[reply]

I am reading a bunch of sci-fi actually. And I watch Noir-films. I also want to add, that I'm not trying to equal the effects of TMS, so the exact radiofrequencies of those magno tests--i'm not trying to match. I'm just questioning the possiblities of a weak electro spectrum. Something that ties in with microwaves from electric power lines over the streets, or plasma/lcd screens in homes. And I wanted to test this theory, with a fine Ferrofluid powder--these things don't need extremely powerful magnets. I want to create a new term calling these things in our drink MAGNO-MINERALS. --i am the kwisatz haderach (talk) 18:57, 26 January 2010 (UTC)[reply]

Here's an example of exactly what I'm talking about for Film Noir, Notorious_(1946_film), Under Plot, in the middle of the 3rd Paragraph: 'The poison is initially mixed into Alicia's coffee'. This film was right after WWII, there's an aura of wrongness that they don't really go into, but as I remember, in this Hitchcock film, Cary Grants character sneaks into the cellar, accidentally breaks open this bottle of wine, but instead of wine, all this metalic dust a comes crashing out. DUN-DUN-DUUUN!!! --i am the kwisatz haderach (talk) 21:39, 27 January 2010 (UTC)[reply]
I would encourage you to pause, and first execute a controlled, double-blind experiment on your fundamental premise that drinking from the corporate cup is the exact factor that is altering your radio reception. The assertion seems silly on its face. Comet Tuttle (talk) 19:42, 26 January 2010 (UTC)[reply]

Can anyone answer his original question? Would the small magnets in a set of earbuds create enough of a field to be seen with iron filings? I suspect that they would, at least for filings near the earbud, but the best way to find out's just to try it. And I think the whole fishbowl/Magnadoodle thing is a bit more complicated than you need. Just buy some iron filings, sprinkle them on paper, and stick the earbuds underneath the paper, about 8 inches apart from each other. Buddy431 (talk) 21:09, 26 January 2010 (UTC)[reply]

This Youtube video is a good tutorial on making a magnetic field viewer that is probably more sensative than just sprinkling iron on a piece of paper. I doubt that actual ferrofluid would react at all to ear buds, and besides it is expensive and messy. APL (talk) 22:40, 26 January 2010 (UTC)[reply]
Thanks for this video link, THIS IS IT. This is the test style I'd like to do in regards to our drinking supply provided by the Conglomerate Corporatesque Controls that be. --i am the kwisatz haderach (talk) 21:39, 27 January 2010 (UTC)[reply]
TMS is not a subtle thing. It feels like someone knocking on your head. (At least, it was when I had it done to me, about five years ago.) Just pointing that out. I find it very unlikely that your choice of coffee has anything to do with your radio reception. That sounds a wee bit crazy to me. --Mr.98 (talk) 23:19, 26 January 2010 (UTC)[reply]

Ok, lets say TMS in movie talk, is the equivalent of Dr. Evil, and what I'm talking about here is Mini-Me. In regards to the coffee/radio connection sounding crazy, I agree with it sounding a wee bit crazy. But I'm not here to talk of entertainment or sanity, I'm here to talk of science, and the sciences of the unseen, aka Magnetism. At that, and in regards my observations, its like this. I go to work, M-F, like clockwork, I have my radio station tuned to 89.9 KCRW here in Sun Valley, CA, close to the Burbank Airport, I believe they block out some radio frequencies somehow for air traffic control reasons, so at that the reception isn't all that great. I drink a cup of joe a day. Sometimes I get a little tense, and I think, hmmm, maybe I should not drink me my caffiene, and so I just stop drinking coffee for a few weeks. Now, because I don't change the station, I don't move the dial on the radio at all, and in my zen seeking non-coffee drinking, I like music sometimes, but then I get all this static, and in the enjoying of music, static, not good. Most of us remember the rabbit ears on the TV's, and when we hold the two ends, to find the good reception, we get clarity on the screen, and then we let go, and the static comes back, so yes, we all know we are all radio receptors, and some of us are acute enough in this to actually hear some frequencies at times. It would be a bit diabolical if there were these electronic magnetic activator machines underneath the barista that added extra magnetic juice to the magnetic nano-scale iron works in the drinking supply. I'm not questioning on the political front implications, I question the science of simply, 'radio reception and coffee making it better'. That doesn't make sense, but if there was some nano-magnetic jiz up in the coffee, that may make something like this a possiblity. Now at that, I'm trying to come up with ways to experiment further in this than just drinking coffee and playing with my radio. If we can try to refrain from sarcasm, and if not, at least come with a sarcasm that's Niels Bohr-ish. In this frame of questioning, lets think, if something like this was in fact plausible, how would we go about testing for it? I can't really take this to corporate funded learning institutions, this is a grass roots science project. --i am the kwisatz haderach (talk) 16:20, 27 January 2010 (UTC)[reply]

Earbuds usually use piezoelectric speakers rather than electromagnetic speakers, so I wouldn't expect to see a field large enough to move iron filings. --Carnildo (talk) 00:55, 27 January 2010 (UTC)[reply]

Lets say you ate some meat with "Mercury_(element)", aka poison, in it--which is a Diamagnetic, the Bioaccumulation of this and other magnetic elements in the body, I'm sure can cause Neurosis. --i am the kwisatz haderach (talk) 23:50, 28 January 2010 (UTC)[reply]

why has no one investigated the acid-base properties of chloramines?

I just want to find out how much having the chloro groups on there influences the acidity of the amino proton as well as the pka of its conjugate acid. However, google gives a grand total of zero results for such queries as "pka of chloramines" and "basicity of chloramines". HELP?!!! John Riemann Soong (talk) 18:32, 26 January 2010 (UTC)[reply]

Surely this must be a simple undergrad experiment, right? It can't be that troublesome to measure the pka of a chloramine. I just don't have the clearance for that kind of thing right now. John Riemann Soong (talk) 20:14, 26 January 2010 (UTC)[reply]

um, yeah, I googled "pka of chloramines" and got a shitload of sites which have information on that. I don't get one site that lists a bunch of pKa in like a table format, but I get LOTs of good stuff. Are you sure you typed the right thing in? This site gives the pKa of chloramine itself. This pdf seems to have a bunch of general properties of chloramine. here is a scholarly paper on the a specific experiment to determine the pKa of a specific chloramine derivative. I'm not sure how you could NOT find stuff. Both a straight google search and a google scholar search turn up tons of links. Also, have you tried both the CRC handbook and the Merck Index? They both tend to have lots of physical data on various compounds. --Jayron32 21:27, 26 January 2010 (UTC)[reply]
I don't get why there isn't an easily-available pKa index stored on a central federal science database or something? I don't have access to print materials. John Riemann Soong (talk) 00:08, 27 January 2010 (UTC)[reply]
Also that site doesn't give the pka of chloramine. It's a mirror site of Wikipedia. Also some other sites give irrelevant pkas of the wrong functional group -- like they give the pka of some alpha-proton or carboxylic acid that happens to have a chloramine group on it.
The guide you cited also doesn't say anything about the acid-base properties of chloramine. It gives a bunch of irrelevant stuff about water treatment and all that. I'm simply trying to oxidise an amine-alcohol compound with bleach to get a carbonyl group while minimising the amount of chloramine side product. I don't know why everyone seems to be focusing on the annoying water treatment aspect (no one should use chloramine for water treatment) and not on the organic synthesis perspective, which seems more important. John Riemann Soong (talk) 00:24, 27 January 2010 (UTC)[reply]
Finally, that last paper talks about a molecule with a sulfonyl group on it ... uhhhhh that's like talking about trichloroacetic acid when the pKa of acetic acid is desired. And I can't even access the paper because it's only an abstract. =( John Riemann Soong (talk) 00:28, 27 January 2010 (UTC)[reply]

You ask far too many inane and absolutely ridiculous questions. Open up a bloody book, or in this case, spend more than 30 seconds quickly perusing google before harassing these lovely Wiki reference desk folks. And stop asking questions like you've got an epic hardon for the answer and need "it" immediately. Try to be a bit less neurotic and nicer. MrFudgey (talk) 23:36, 26 January 2010 (UTC)[reply]

I do try to be nice but I don't know why my question goes unanswered for days. I mean, people ask simple questions all the time. I don't have access to anything besides my textbook, which btw, doesn't cover chloramines as functional groups. Also, when I google "pka of chloramines" (with quotes) I get no results, whereas without quotes returns me irrelevant results.
I spent around 5 minutes before giving up. Researchers often know pKas from experience so I thought it was something someone could tell me right off the top of the bat (along with the pka of phenol being around 9-10, mercaptans 10-11, amines 35 for the conjugate base and 10 for the conjugate acid, etc. John Riemann Soong (talk) 00:07, 27 January 2010 (UTC)[reply]
John. Listen to me. Go find a good old hard copy of the Merck Index or the CRC Handbook. Before coming here, did you look in either of those places? Any good chemist will have both lying around. Unfortunately, I have dug through my stuff, and I can't find them right now; I think my wife has taken them to her work and keeps our home copies there now. But seriously, try those two places first before asking something simple like this. --Jayron32 03:55, 27 January 2010 (UTC)[reply]
OK. Kiss me later. I found the SPARC online database located here:[10]. The software uses the SMILES system, but it has a little javascript SMILES converter from ChemDraw, so basically you draw the molecule in a simple ChemDraw applet, and it creates the SMILES string for you. If you play around with the site, it will give you all sorts of good info. It took me about 5-10 minutes to figure out the software, but once I did, I got the numbers. Based on their data, the pKa of H2NCl --> HNCl- is 21.51, and the pKa of H3NCl+ --> H2NCl is 0.10. I hope that helps some! --Jayron32 04:10, 27 January 2010 (UTC)[reply]

planetary nebula

Do planet survive inside panetary nebula or they will get destroy in planetary nebula. Planetary nebula is between RGB and white dwarf, if Mars go in shell of planetary nebula will it be vanish? --209.129.85.4 (talk) 20:46, 26 January 2010 (UTC)[reply]

Planet to keep a descent atmosphere

What do planet need to have a descent atmoshpere. Since Tango always say Titan will bleed away it's atmoshpere it is always unsource statement. This said Titan might keep it's atmosphere in 6 billion year sun, just the orange haze at the upper part will deplete, but it will keep some, just not that thick. Do planet diameter matter? Could planet be 1/10th the size of earth and still have an atmosphere. Pluto is also made of ice, I never hear anybody else say Pluto will just outgass. Could surface gravity also be weak and have the planet hold it's atmosphere.--209.129.85.4 (talk) 20:53, 26 January 2010 (UTC)[reply]

It's basically all about gravity. So the size and density of the planet/moon/whatever is what matters. Also, if the body is too cold then there may be no materials in it's makeup that are gaseous at those temperatures. Whether a planet like Pluto would outgass from icy stuff on the surface also depends on temperature. SteveBaker (talk) 21:50, 26 January 2010 (UTC)[reply]
Temperature is more important than that. The particles in the atmosphere will have velocities based on the Maxwell-Boltzmann distribution. That is a function of temperature. That means that, at higher temperatures, a greater proportion of the particles will have escape velocity and will escape. The remaining particles then exchange momentum through collisions and get back to the same distribution, meaning some more particles have escape velocity and escape. This process means that any atmosphere constantly loses particles to space, the atmosphere can only survive over long time scales if enough new gas is added to compensate for the losses. Those losses depend on temperature, so at higher temperatures you need more replacement gas to maintain the atmosphere. That is one of the main reasons that Titan has a thick atmosphere while the Moon, which is a very similar size and mass, and essentially no atmosphere at all. If Titan warmed up to similar average temperatures as the Moon, it would lose its atmosphere. I will try and find some references for that. --Tango (talk) 22:42, 26 January 2010 (UTC)[reply]
First of all, the source cited by the article the OP links to is talking about Titan getting warm enough for a water-ammonia ocean to form, not a water ocean. They are talking about maximum surface temperatures of about -70°C - a long way below Earth-like temperatures, but perhaps warm enough for life based on a slightly different biochemistry to us to arise. Those lower temperatures mean the atmosphere would escape far slower than it would at Earth-like temperatures, so could well survive for millions of years. --Tango (talk) 22:56, 26 January 2010 (UTC)[reply]
I have failed to find any reliable sources talking about Titan at Earth-like temperatures since a red giant sun isn't expected to heat it that much and people discussing terraforming dismiss it as essentially impossible, so no-one gets as far as thinking about what would happen to the atmosphere at those temperatures. --Tango (talk) 23:29, 26 January 2010 (UTC)[reply]
[citation needed]. Tango, I've heard the same argument about the Maxwell tail that you give on many occasions. It is certainly true that an object with a hot enough surface or a low enough escape velocity will bleed atmosphere into space. But I'm not sure how relevant that is to an object the size of the moon. Lunar escape velocity is ~2.4 km/s. At 300 K, an oxygen molecule has only a 1×10−15 probability of having enough velocity to escape (i.e. from Maxwell-Boltzmann). And in order to escape it needs to be moving in the right direction and living in a part of the atmosphere that is already of such low density that its mean free path is nearly infinite, or else it will collide with other molecules and lose energy before escaping. Once you combine low probability with low density, it seems like the rate of mass loss to the Maxwell tail for any gas much heavier than helium should be nearly nil under lunar gravity. By comparison it seems like collisions with solar wind particles, with typical velocities of 400-750 km/s, would be a much more effective means of providing gas molecules with enough velocity to escape out into space. So I'm not sure drawing your comparison to the Moon makes sense because I'm not sure if losses on the Maxwell tail are really a determinative factor for the Moon's lack of atmosphere. Dragons flight (talk) 00:40, 27 January 2010 (UTC)[reply]
Solar wind is certainly a key factor in atmospheric erosion - I think that's the main reason for Mars' lack of any significant atmosphere. 1×10−15 is quite high, though. How long do you think it would take for a particle with escape velocity to escape and for the distribution to adjust itself? I really don't know, but I'm guessing not long. A second, maybe? That means you lose 1×10−15 of the atmosphere every second. That corresponds to 22 million years to lose half the atmosphere. That means Jeans escape (as it is called) cannot be ignored on the timescales we are talking about. --Tango (talk) 00:55, 27 January 2010 (UTC)[reply]
No, you lose 1×10−15 of the fraction of the atmosphere with a mean free path long enough that escape is plausible before it hits other gas molecules. For oxygen molecules at 300 K that corresponds to a pressure of order 1×10−8 atmospheres. That certainly occurs, but only at high altitude in an atmosphere of non-trivial mass. Once you combine those two factors, the half life for an oxygen atmosphere to Jeans' escape from a lunar mass object should but much longer than the age of the solar system. Dragons flight (talk)
Ah, I've found a forum where someone has actually done the calculation: [11]. Apparently Titan's current atmosphere with an exobase temperature approximately equal to that of Earth's (1000K) would take on the order of a few billion years to escape (there are all kinds of assumptions involved in that calculation which almost certainly aren't true, so I'd say that is give or take an order of magnitude). They don't provide a reference for the formula or values they use and I haven't checked their arithmetic, but it all looks plausible. So, if Titan had been at those temperatures for its entire existence, we could expect the atmosphere to be largely gone by now, but heating it up either as a terraforming project or by the sun becoming a red giant should be reasonably safe. Either the source I was remembering reading years ago was wrong, or I was remembering it wrong and it was actually just talking about why Titan still has an atmosphere now despite being so small. However, the estimate that Titan will only warm up to about 200K when the sun goes red giant puts the entire premise on very shaky ground, so the question is pretty much moot. Thank you for the very interesting discussion and for giving me the incentive to actually look it up rather than rely on memory! --Tango (talk) 02:47, 27 January 2010 (UTC)[reply]
This is the link article links to. This book is made in 1997 now they estimate the sun expansion to be bigger inofrmations could be a little outdate. They said in first paragraph could be 300K, but for Wiki policy informations have to stick with the verifility.--209.129.85.4 (talk) 18:00, 27 January 2010 (UTC)[reply]

Number of seeds contained in a grape

How many seeds are on average contained in a grape?--87.11.120.169 (talk) 21:10, 26 January 2010 (UTC)[reply]

Seedless varieties have zero, Grignolino grapes can have as many as 10. Most normal varieties have between 2 and 4. SteveBaker (talk) 21:47, 26 January 2010 (UTC)[reply]

January 27

What is the meaning, and/or purpose of human life?

No, I'm not asking for a definitive answer to this question. :) Rather, I'm curious as to *how close* the 'hard' sciences are to providing an definitive answer to the question. Can this question be answered by 'hard' science? Is anyone working on it? Or has the answer already been provided a long time ago, in that the meaning and/or purpose of human life is, simply 'to survive and to reproduce'? --95.148.104.185 (talk) 00:34, 27 January 2010 (UTC)[reply]

[[12]] --i am the kwisatz haderach (talk) 00:37, 27 January 2010 (UTC)[reply]
I'm not aware of any reason for "hard" sciences to be pursuing an inherently subjective philosophical problem. Philosophers, on the other hand, have produced loads of answers. See meaning of life. — Lomn 00:39, 27 January 2010 (UTC)[reply]
The reason I ask about 'hard' sciences is that I was curious as to whether a definitive, demonstrably true 'the purpose of human life is... <x>' answer, with no room for speculation, personal opinions and any other 'grey areas' will ever be devised. I've already read a lot of the philosophical answers. --95.148.104.185 (talk) 00:50, 27 January 2010 (UTC)[reply]
To reproduce and maintain human life. --121.54.2.188 (talk) 01:01, 27 January 2010 (UTC)[reply]
[citation needed]. Comet Tuttle (talk) 01:06, 27 January 2010 (UTC)[reply]
Biological fatalism. 66.65.139.33 (talk) 01:39, 27 January 2010 (UTC)[reply]
Purpose implies some form of intelligent design, a theory rejected by science, so science has nothing to say on the subject of the purpose of human existence. --Tango (talk) 01:05, 27 January 2010 (UTC)[reply]
Not exactly. It says there is no purpose of human life. Also, the meaning is generally accepted to be a member of homo sapiens with brain activity. — DanielLC 01:07, 27 January 2010 (UTC)[reply]
The meaning of life is the avoidance of death. I'm just joking, of course. Bus stop (talk) 01:10, 27 January 2010 (UTC)[reply]
One could make the argument that the point of life is to die. But that's pretty depressing. ;-) --Mr.98 (talk) 01:13, 27 January 2010 (UTC)[reply]
There is a difference between there not being a purpose and the concept of purpose not applying. Exactly what that difference is and whether it is relevant to this question is a debate best left to philosophers that have nothing better to do, though! --Tango (talk) 01:17, 27 January 2010 (UTC)[reply]
The reason no one, including but not limited to scientists, have come up with good answers to this is because it's not a very good question. What people usually mean when they say this is, "why I am conscious, when it seems possible for me not to be?" or "what should I be doing with my time?" Those are better questions, even if science doesn't have a whole lot to say on them. They are at least focused. "What is the meaning of life?" could be anything as vague as pondering why there was a Big Bang, to a request for a strict dictionary definition. Science is not good with vague questions. Narrow the question and maybe science can prove useful. But if you don't know what you are really asking about, science certainly can't help. --Mr.98 (talk) 01:13, 27 January 2010 (UTC)[reply]
The answer to the question, "what should I be doing with my time?" is, "as little as possible." I'm just joking, of course. Bus stop (talk) 01:21, 27 January 2010 (UTC)[reply]
Indeed, "what is the meaning of life?" can only really be answered by looking up "life" in a dictionary. If you want a less literal interpretation then it could mean anything. "What is the purpose of life?" is a little better, although it does presuppose that there is a purpose, which is why it doesn't have a good answer. --Tango (talk) 01:17, 27 January 2010 (UTC)[reply]
Would "the 'purpose'/'raison d'etre' of homo sapiens as a species is to increase the number of homo sapiens" seem like a reasonable statement? --Kurt Shaped Box (talk) 01:20, 27 January 2010 (UTC)[reply]
Sure, that's an answer, but if you attempt to apply the scientific method to that, you get nowhere. For example, try: Hypothesis: The purpose is ___." Okay, now how do you go about testing it? What's your control? This is not a scientific hypothesis; it is untestable, it does not make testable or falsifiable claims; therefore, it is out of the purview of (at least the hard) science. Nimur (talk) 01:41, 27 January 2010 (UTC)[reply]
What science has discovered is that humans are a human-gene's method to make more human-genes. Every aspect of our being has been tailor made in the forge of evolution to be an optimum copier of human genes. That (as close as we can tell) is basically it.
SteveBaker (talk) 01:22, 27 January 2010 (UTC)[reply]
They're not trying to make human-genes. It's just what happens. — DanielLC 01:26, 27 January 2010 (UTC)[reply]
When we (humans) build a computer are we trying to build a computer or is it just what happens? Bus stop (talk) 01:34, 27 January 2010 (UTC)[reply]
The answer to that, Bus Stop, is very subtle and subjective. See Primum movens, for example - which has both a theological and a secular classical philosophy interpretation. I would go so far as to say, "it's just what happens." In other words, the universe was just so fortunately constructed that physical laws occurred, stars accreted, planets formed, carbon life developed, complex brains evolved, technological society arose, machine tools were invented, and the microelectronics industry accreted - and it just happened because of the laws of the universe. We are sophisticated enough to have the illusion of self-awareness, but we still only act in accordance with physical law. This classifies me as a strong determinist. It doesn't mean I'm right - this is a subjective opinion. To my knowledge, determinism isn't really falsifiable (therefore it's philosophy, not science), although there are some guys who purport that physical nature of quantum mechanics (in particular, the extensions of the disproofs of hidden variable theory) invalidate strong determinism. I'm unconvinced - but that's only because my neurons happen to fire that way. Nimur (talk) 01:48, 27 January 2010 (UTC)[reply]
The question presumes an unnecessary assumption about life. Other animals don't need an external to live. Gods of holy texts don't need external reasons to live. Why should we? Just live so you won't have any regrets, assuming you don't get joy from hurting people. 66.65.139.33 (talk) 01:38, 27 January 2010 (UTC)[reply]
Well everything is open to interpretation, but if I were to summarize everything I've learned about the 'purpose of life', other than the obvious (there is none, or 'endure suffering', or 'to do glory unto God', or 'have fun') it would be learning to temper yourself into a functional and agreeable human being. I mean everything begins and ends with you and your body, so upgrading the software and hardware is really the first process in anything resembling a good and meaningful life. Vranak (talk) 03:24, 27 January 2010 (UTC)[reply]
The OP would do well to read Rocks of Ages by Stephen Jay Gould. Questions of purpose are outside of the remit of science to give the answers to. It would be like asking a question about the plot of Hamlet from your pocket calculator. Yes, calculators give very good answers if asked the right questions, but they are unequipped to give answers about literature! Likewise, science is a tool equipped to give all sorts of really good answers, but not every answer regarding human existance. Where the tool of science falls short, other tools (religion, humanities, art, etc.) must be used. --Jayron32 03:43, 27 January 2010 (UTC)[reply]
I deeply disagree with every single part that answer! If some scientist somewhere could figure out an experiment that would determine man's purpose - we'd do it, publish the results and call it a "Theory". It's not something where scientists would say "Well, that's outside our remit so we won't bother." - nothing whatever is beyond the scope of science - there are just some questions that we don't yet know how to answer. A sufficiently advanced calculator could certainly answer questions about the plot of Hamlet - it's a computable problem and a programmable calculator with enough memory could do it. Resorting to religion, art and philosophers to answer the question could never produce an answer with the rigor of a scientific answer. It's a pretty safe bet that no two priests, artists or philosophers would agree anyway! You might just as well guess for all that's worth! SteveBaker (talk) 05:58, 27 January 2010 (UTC)[reply]
Science has provided the answers to questions about life and death. Some people just won't accept them, that's all. 66.65.139.33 (talk) 19:40, 27 January 2010 (UTC)[reply]
The meaning of life is probably not a question that is answerable by the hard sciences. If the goal was simply to reproduce, then why is this causing us to approach the carrying capacity of the planet and overwhelm sustainability? It may be something more abstract, for example to inspire others and to make a difference in the world. However, that would be bordering on the realm of Religion. Or perhaps it is not meant to be answered at all, but that's the realm of metaphilosophy. There are also some scientific or mathematical theories that offer a glimpse at the picture, for example Oneness. It is also likely, however, that the meaning of life is simply ineffable. ~AH1(TCU) 20:12, 27 January 2010 (UTC)[reply]
Evolution has no foresight whatever. Animals and plants will always increase in number until predation and other causes of death balances their maximum reproduction rate - or they hit the limit of the available resources and experience a disasterous population crash as a result. Mankind, having become so efficient at finding and using resources and with no predators and increasingly good health and reproductive care, will obviously grow in numbers until we do indeed run into the limits of the planet and crash horribly in the process. Unless of course, we can use our intelligence to overcome the instincts built into us by evolution and limit our use of resources and reproduction rates in order to meet a sustainable goal. Sadly, there is little sign that enough of us are smart enough to do that. SteveBaker (talk) 01:31, 28 January 2010 (UTC)[reply]

Brown lines and sections on house plant leaves

I remember reading that plant leaves are eaten and digested by tiny things, which leaves trails of the brown stuff. Is that true? Would cutting out the damaged parts with scissors help? I'm guessing it'd hurt the plants, but I'll ask anyway. Thanks. 66.65.139.33 (talk) 01:09, 27 January 2010 (UTC)[reply]

Perhaps you are thinking about a Leaf miner which eats the leaf tissue between the upper and lower cuticles of the leaf leaving a visible trail that varies with specie. you could cut out the bits of affected leaf but that would look as bad if not worse than the trails. If it is a minor house plant problem your best bet is to just squeeze the miner between thumb and finger. The miner will be located at the thicker end of the trail. Richard Avery (talk) 08:16, 27 January 2010 (UTC)[reply]
I don't see any bugs on my house plants. 66.65.139.33 (talk) 19:38, 27 January 2010 (UTC)[reply]
No, you won't, because the miner (bug) is between the two outside layers of the leaf. It stays in there until it has passed through its larval and pupal stage(usually) and then hatches out through the skin of the leaf and goes and lays some more eggs. The only way you can know it is there is by seeing its track as it eats its way across the leaf - but inside the tissue of the leaf. Richard Avery (talk) 07:16, 28 January 2010 (UTC)[reply]

Tobacco's body count and total excess deaths of drugs

Should "Smoking" be "Tobacco" because "Cannabis" is filed under "Drug abuse"?

Recently I came across the statement that "Cigarettes kill more Americans than alcohol, car accidents, suicide, AIDS, homicide, and illegal drugs combined."[13]. Can that be confirmed by reliable sources? The Google search on the question seems confirmatory to me.

Does someone have a chart of the popular drugs sorted by the number of excess deaths? Google is much less helpful for that question. 99.56.138.51 (talk) 02:01, 27 January 2010 (UTC)[reply]

We have an article on List_of_preventable_causes_of_death#Leading_causes_in_the_United_States Vespine (talk) 03:04, 27 January 2010 (UTC)[reply]
Thanks! That's almost exactly what I need, except "Smoking" needs to be tobacco-specific (by drugs.) 99.56.138.51 (talk) 05:40, 27 January 2010 (UTC)[reply]
Yes. In the USA in the year 2000: 432,000 deaths per year due to smoking, 85,000 for alcohol, 43,000 for car accidents, roughly 38,000 for suicide, HIV/AIDS deaths are so small as to be negligable, 29,000 due to firearms ("homicides" would be different - maybe not by much), 17,000 for drugs. So that's 212,000 due to all of those things - 432,000 due to smoking. So - yeah, the statement is true. In fact, smoking kills TWICE AS MANY Americans than all of those other causes combined. The source for that is this paper [14] which is quoting numbers from the Centers for Disease Control and Prevention (CDC) - which is considered to be highly reliable.
Even if you assume that all of the 'drug' deaths were due to smoking drugs (unlikely in the extreme because nobody smokes 30 MJ cigarettes a day - but that's rather common in tobacco smokers!) that hardly makes a dent in the numbers.

SteveBaker (talk) 05:47, 27 January 2010 (UTC)[reply]

The World Health Organization predicted that in the 21st century, one billion people will die from tobacco and cigarette smoking. ~AH1(TCU) 20:07, 27 January 2010 (UTC)[reply]
At 5 million smoking-related deaths worldwide per year - the number of annual deaths would have to more than double. I'm rather surprised that the rates are still increasing. With all we know, that's rather depressing. SteveBaker (talk) 01:20, 28 January 2010 (UTC)[reply]
Unfortunately the "all we know" is quite localized to developed countries. I saw a doco which stated one of the largest "untapped" markets left for the tobacco companies was Chinese women, it is still quite a taboo there for women to smoke. They hire young women "promo girls" there to give out cigarettes and promote that it is empowering and "equal" for women to smoke trying to break down the taboo. The west went through the same thing with the same taboo, only with us it happened in the 20s, except I think they stopped short of giving out free cigarettes, but I could be wrong about that. The dealer giving out the 1st "hit" free comes to mind, and perfectly legal. Sad indeed. Vespine (talk) 02:03, 28 January 2010 (UTC)[reply]
They gave out free cigarettes in the Army in the US. It was one of the major reasons for the boom in smoking in the 1940s. But yeah, cigarette companies are now extensively targeting less-developed and newly-developing countries for their new markets, knowing that the US/European market is probably in a state of permanent contraction. Pretty disgusting, if you ask me. (Allan Brandt's The Cigarette Century discusses both of these trends, among others.) --Mr.98 (talk) 14:57, 28 January 2010 (UTC)[reply]

Capturing a comet inside an asteroid for an aquarium

How much would it cost to perturb the orbit of an asteroid while excavating it with the goal of colliding it with a comet and capping the resulting ice with a seal to allow for the future possibility of an aquarium, as a function of aquarium size? 99.56.138.51 (talk) 02:04, 27 January 2010 (UTC)[reply]

If you collide an asteroid with a comet you will probably just break the two bodies into tiny pieces. A lot (most?) asteroids are just piles of rubble with only a tiny bit of gravity holding them together, they break up very easily. I don't really understand your intention, anyway. However, I can tell you that any elaborate space mission outside Low Earth Orbit that has never been done before is going to cost in the region of billions of dollars at the very least. --Tango (talk) 02:17, 27 January 2010 (UTC)[reply]
It's entirely dependent on where the two objects are orbiting and what their relative velocities are - and (I suppose) how soon you need it done. There are so many orders of magnitude of variation involved that there is no meaningful answer. SteveBaker (talk) 05:33, 27 January 2010 (UTC)[reply]
Six years. 99.56.138.51 (talk) 05:43, 27 January 2010 (UTC)[reply]
Six years from now? I'd say that was pretty much impossible without it being a top priority of most developed nations (which it wouldn't be). It takes longer than that just to plan, design and build such spacecraft, and it would take a very long time to actually create the collision, even if you can find a suitable comet and asteroid. --Tango (talk) 17:02, 27 January 2010 (UTC)[reply]

Presumably, the Seal would consider the aquarium to be a Future Love Paradise. --Dweller (talk) 07:59, 27 January 2010 (UTC)[reply]

The best order-of-magnitude estimate may be that it would cost about the same as placing a man on the moon, and that the acquarium size would not matter. 92.27.165.25 (talk) 11:50, 27 January 2010 (UTC)[reply]
It would probably cost even more then that because your deltaV would probably be larger, and the object which needs to change its velocity is heavier. Googlemeister (talk) 15:49, 27 January 2010 (UTC)[reply]

An Aquarium has fish on the inside and observers on the outside. It is filled with liquid water. Has the OP plans to obtain fish, viewers and a way to melt the comet ice? Cuddlyable3 (talk) 19:08, 27 January 2010 (UTC)[reply]

Fish need oxygen...where is that coming from? I guess you'd need to use solar panels to make electricity and use that to crack water into H2 and O2...vent the H2 to space and bubble the O2 through the tank. Er...wait a minute...why are we doing this rather crazy thing? SteveBaker (talk) 01:11, 28 January 2010 (UTC)[reply]
Like I said, I don't understand what the OP is actually intending to do... --Tango (talk) 01:17, 28 January 2010 (UTC)[reply]

Superball

I've been given this homework question (I know you're not supposed to answer HQ, but all I need is a little push in the right direction): Suppose a superball (ie a ball such that kinetic energy is conserved during collisions) of mass M is dropped from a height h, with a smaller ball of mass m on top of it (M much greater than m). The question ask, how high will the small ball bounce? Alright, so it's pretty straightforward to calculate the speed of the two balls upon impact with the ground. During the collision, I would think that the kinetic energy both balls had accumulated during the fall are converted into the elastic potential energy. Now, as the balls begin to rebound, there's going to be two forces acting on the superball: the force of the ground pushing up on the superball, and the force of the smaller ball on the superball. The small ball also experiences a force, that of the reaction force to the last mentioned force. Total energy is (m+M)gh, which must be conservered. But I don't know how to combine these facts to get an answer...help! —Preceding unsigned comment added by 173.179.59.66 (talk) 03:01, 27 January 2010 (UTC)[reply]

Energy is conserved, so the easiest way to calculate that is to assume that the total energy of the system is that of gravitational potential energy at the top of the drop:
  • E = (m+M)*g*h
Now, at this point the total potential energy is dependent on the sum of the two masses, so (m+M) in this case is the mass of BOTH balls.. When the two balls hit the ground, if the balls are perfectly elastic (which you indicate they are) then 100% of this energy will be transferred to the top ball. So take the E from the first equation and plug it into the kinetic energy equation:
  • E = 1/2*m*v2,

but NOW you only use the mass of the top ball. Solve for v. This is the initial velocity (vi) of the small ball. If we want to know the maximum height, we assume the final velocity is zero. Using the acceleration of gravity and your basic dynamics equation:

  • d = 1/2 (vf2 - vi2)/a
Where vf = 0, vi is the velocity you calculate immediately above, and a = -3.2 m/s2 (the accelration of gravity). Those three steps will get you the right answer. Any questions? oh, and someone else check my algebra here too!--Jayron32 03:37, 27 January 2010 (UTC)[reply]
The trouble is, not all of the energy will be transferred to the top ball. Momentum is also conserved. It will be easier to assume that the bottom ball hits the ground and starts to bounce back up before the superball hits it. So first, you'll figure out how fast the bottom ball is when it hits the ground. You can assume (by conservation of energy) that the big ball will be travelling upward when it hits the superball at the same speed that it hit the ground at. Then, you can use both conservation of energy and momentum (the collision is elastic) to find out what happens after the collision (I assume you've been doing collisions in class). The big ball can be assumed to be so massive that it doesn't change velocity at all during the collision. I think that the little ball ends up at three times the height it was dropped from, but you better check that. Buddy431 (talk) 04:00, 27 January 2010 (UTC)[reply]
My bad; the superball's speed coming off the bounce is three times what it came in with. That puts it at nine times the initial height. Buddy431 (talk) 04:13, 27 January 2010 (UTC)[reply]
It doesn't affect your answer, but momentum isn't conserved since we are assuming the mass of the ground is infinite (ie. it doesn't move) and infinity doesn't follow the usual rules of arithmetic. If you consider the two collisions separately (which I agree is the best approach) then momentum is conserved for the second collision, since it doesn't involve the ground, but not the first. For the first, as you say, the ball is simply reflected. --Tango (talk) 04:16, 27 January 2010 (UTC)[reply]
The momentum in the first and second collision are conserved in the same way; we assume the larger body has infinite mass compared to the smaller one. As an aside, our superball article (which is shamefully underdeveloped) says a real life superball keeps something like 70% of it's energy on a bounce. That means after the first bounce you'd be at 84% of the speed you were coming in with, and after you hit the second ball, it only leaves with about 2.4 times the speed it came in with, for a total height of about 5.7 times what you dropped it from. Throw in drag, and the fact that the second ball isn't on infinite mass as compared to the first, and you're looking at 4-5 times the initial height (which is still impressive). I've heard that you can buy sets of three balls meant to be dropped on top of each other, which just seems irresponsible and dangerous. And if 173.179 doesn't like our answers, he can check out this explanation. Buddy431 (talk) 04:38, 27 January 2010 (UTC)[reply]
For a look at the demonstration, check out this youtube clip. It doesn't show heights, but provides a good visual of the problem. Buddy431 (talk) 04:45, 27 January 2010 (UTC)[reply]
Looking at 173's question again, and Tango's comment, I see that Tango's right, and my answer isn't quite the correct one. The first collision is with the Earth, which does basically have infinite mass, so the large ball bounces off with the same speed it came in with. The second collision isn't the same though; the problem does give two masses m and M for the balls, which I missed. In that case, just use both conservation of momentum and energy to find how fast the small ball leaves. So my answer, assuming that the small ball's mass is negligible compared to the large ball, is a limiting case. The actual answer will be somewhat less than 3 9 times the original height. Buddy431 (talk) 06:03, 27 January 2010 (UTC)[reply]
And last time I checked, gravity near the surface of the Earth accelerates objects at 9.81 m/s2. I don't think it will matter in this case (it cancels out), but I'm not sure where the 3.2 number is coming from. Buddy431 (talk) 04:05, 27 January 2010 (UTC)[reply]
Like I said, check me. I think I was confusing the foot measurement (32 feet per second per second) with the meter measurement. So sue me. --Jayron32 04:14, 27 January 2010 (UTC)[reply]
If it's good enough for NASA, it's good enough for us! --Tango (talk) 04:17, 27 January 2010 (UTC)[reply]
Larger ball hits the ground with speed . It rebounds in a perfectly elastic collision, so now has upward speed v. It immediately collides with small ball, which has downward speed v. Collision is again perfectly elastic, so coefficient of restitution is 1, so velocity of small ball relative to big ball after collision must be 2v upwards. Velocities of small and large balls after second collision are and respectively. In the limit, as m/M tends to 0, the velocity of the small ball after the second collision is 3v and it rises to a height of 9h above the collision point.
If we add a third even smaller superball on top of the small ball, with a mass that is much smaller than the small ball, then I think its velocity after colliding with the small ball is an astonishing 7v. Gandalf61 (talk) 10:35, 27 January 2010 (UTC)[reply]

joule thief

what is a joule thief? what is a torid(on which they coil wire)? how does joule thief works? —Preceding unsigned comment added by Myownid420 (talkcontribs) 03:36, 27 January 2010 (UTC)[reply]

See joule thief. --Jayron32 03:38, 27 January 2010 (UTC)[reply]
And Toroidal inductors and transformers. SteveBaker (talk) 05:02, 27 January 2010 (UTC)[reply]
A great invention. Is there any data on how much extra energy it can squeeze out of a battery? Do any consumer products have this circuity in them? 78.146.106.225 (talk) 13:50, 27 January 2010 (UTC)[reply]
The joule thief circuit is a simple relaxation oscillator that converts DC from a battery to an AC or pulsating output. It has two coils wound on a common ferrite core to form a transformer. Current to the collector ("c") of the transistor flows through the primary winding of the transformer. That causes current in the secondary to subtract from the current flowing into the base ("b") of the transistor. (If a winding is connected the wrong way round the secondary current will add to the base current and the circuit will not work.) Recall that the collector current in a Transistor is controlled by the base current. Reducing the base current causes the collector current to reduce which has two consequences: 1) The voltage at the collector rises briefly higher than the battery voltage due to the inductance of the transformer. That is why the LED lights. 2) Both collector and base currents collapse i.e. the transistor turns off. After this the resistor provides base current to start a new cycle. The LED blinks so fast that it seems to be lit continuously.
If you want to make the circuit note that: The number of coil turns is not given but they will be limited by the size of the ferrite bead. Almost any small NPN transistor may work in the circuit. A PNP transistor will also work if the battery and LED are both reversed. The resistor value is critical: if too high the oscillator won't start, if too low the transistor could be damaged.
A similar circuit is used in simple battery powered inverters for fluorescent lamps. It can also be useful for converting the low voltages from a solar cell to a higher voltage e.g. to charge a battery.Cuddlyable3 (talk) 18:58, 27 January 2010 (UTC)[reply]
OK, but the 64,000$ question is still how much extra energy it can get out of a battery? 78.149.231.228 (talk) 01:18, 28 January 2010 (UTC)[reply]
No that is not the OP's question though it seems important to you 78.149.231.228. If you seek a numerical answer then it must be a percentage of the battery currentxtime rating because "a battery" might be any size. In the case of the joule thief the LED can be lit using a battery that is unable to light the LED directly so the "extra" energy is all the energy. Please do not offer money for answers because our voluntary service here is priceless beyond value reward enough. Cuddlyable3 (talk) 13:22, 28 January 2010 (UTC)[reply]
Thanks, although that is a long-winded way of saying "I don't know". 89.242.92.249 (talk) 14:22, 28 January 2010 (UTC)[reply]
If you used a Joule Thief with a fresh new battery - for example a 1.5v AAA - would it be likely to burn out the lightbulb or radio or whatever or be damaged? I'm wonder why every torch / flashlight does not have one installed already. 89.242.92.249 (talk) 14:31, 28 January 2010 (UTC)[reply]
A normal flashlight contains a lightbulb and a few flat "wires" (two of them slide past each other to make the switch), and is very cheap to produce. Adding a circuit such as a Joule Thief to one would increase the complexity and cost, possibly by an order of magnitude or two. --Carnildo (talk) 01:12, 29 January 2010 (UTC)[reply]

Heat death of the universe

This is bordering on philosophical, but I find the Heat Death of the Universe scenario to be extremely depressing. Outside of Asimov, it seems to be a perfect no-win situation - an unhackable Kobayashi Maru. Is there any wiggle room? Or is our universe really condemned to destruction? 218.25.32.210 (talk) 05:00, 27 January 2010 (UTC)[reply]

Actually, heat death is an asymptotic condition rather than a finish line. That is, the universe will proceed inexorably towards complete entropy, but will just wind down infinitely without ever reaching it. Perfect entropy is no more attainable than perfect order, we're just moving closer and closer to perfect entropy. However, yes, heat death is unavoidable since the second law of thermodynamics pretty much demands it. As long as hot places keep getting cooler and cold places keep getting warmer, we're just gonna keep moving towards it. --Jayron32 05:07, 27 January 2010 (UTC)[reply]
Well, it depends on the nature of the universe...the amount of matter and dark matter. Ultimate fate of the universe describes the most well accepted possibilities and heat death isn't the only one that's possible right now.
The Heat death scenario is kinda depressing - but it's an awful long time away and a sufficiently advanced technology could kinda-sorta escape the worst effects of it.
Suppose we're somehow able to put our minds into computers - it's not an impossible thing. Our brains would run on electricity that we'd extract from the universe in some way or other. Solar panels perhaps. As entropy increases, fewer and fewer photons would come your way. It becomes harder and harder to harvest enough energy to run our brains. But a defense against that is to run our brains more slowly. A computer doesn't have to consume any energy at all until it is 'clocked' - until it does a single step in it's calculation. So what we could do is to extract energy from the universe and charge up a battery - when there is enough charge in the battery, have the computer execute instructions until the battery goes dead again. Then accumulate more energy until you can do it again. As the universe gets more and more uniform, it takes longer and longer to accumulate the energy needed to do a single step in the 'brain' calculation. Our brains would start to slow down. As the universe goes downhill, so our brains go slower.
What would that seem like? Well, it would seem like time was speeding up. So as the universe got more and more boring and uniform, time would seem to speed up for us - so the changes in the universe would seem more rapid. It would be like having the universe on 'fast forward'. Time would become not an absolute thing - but something that became relative to the rate that we could gather energy to think with. The universe will continue to have tiny pockets of slight random variation until the very end, so our brains will continue to operate - but very, very slowly. I think the last time I suggested this, someone pointed out some quantum threshold below which there truly would be no energy to be extracted. So the effect for us would be an exponential speeding up of time followed by a sudden and abrupt end. Not at all the slow depressing decay that you're imagining. The last trillion years could pass us by in a heartbeat as the battery charge for our last thought is accumulated one photon at a time.
Humanity's last wild 'goodbye' party could last for a trillion, trillion years yet seem to pass in a couple of hours.
SteveBaker (talk) 05:27, 27 January 2010 (UTC)[reply]
Similar to Dyson's eternal intelligence. APL (talk) 01:02, 28 January 2010 (UTC)[reply]
Yep - that's where I heard about it. I merely simplified it to make it easier to explain. Dyson's approach is better. SteveBaker (talk) 03:22, 28 January 2010 (UTC)[reply]

<--- Or Asimov's The Last Question. 21:25, 28 January 2010 (UTC)

There's Omega Point (Tipler). This is a fine theory which is unfortunately wrapped in religious overtones, but that wrapping can easily be removed and discarded. 213.122.40.58 (talk) 10:38, 27 January 2010 (UTC)[reply]
Yes there is a way out. What people calculate to a nicety on pen and paper and what happens in reality are often worlds apart. And for God, all things are possible. Vranak (talk) 13:52, 27 January 2010 (UTC)[reply]
Can he make himself disappear? 66.65.139.33 (talk) 19:36, 27 January 2010 (UTC)[reply]
He already did. Vranak (talk) 22:47, 27 January 2010 (UTC)[reply]
There's the cyclic universe theory where the heat death thing is just preparation for the next cycle when two branes collide and start the big bang all over again. This is pretty fringe, I think. The book Endless Universe by Turok and Steinhardt was a good read for a layman like myself, and might help with your depression (though, really, we're talking bajillions of years from now...lighten up! :) )Quietmarc (talk) 16:21, 27 January 2010 (UTC)[reply]
I've heard of that theory, but I'm not familiar with the details - is there any way to transfer information from one universe to the next? In the original cyclic universe theory (Big Bang->Big Crunch->Big Bang 2->...) there is a singularity between universes which has the affect of wiping the universe clean, so you can't survive from one to the next. --Tango (talk) 17:13, 27 January 2010 (UTC)[reply]
My memory's rusty (and I never took even high school physics), but I believe that the expanding universe smooths out the branes almost completely, but that tiny imperfections remain. These imperfections are what cause matter in the new universe to coalesce and lump, so that we get galaxies, etc, so yes, there is some carry-over.Quietmarc (talk) 20:10, 27 January 2010 (UTC)[reply]
But where do those imperfections come from? In normal inflationary theory, they come from quantum fluctuations after the big bang. In brane theory can they come from the state of the branes prior to the collision? --Tango (talk) 01:19, 28 January 2010 (UTC)[reply]
A more interesting question is why you find it so depressing. The human race will almost surely have died out a long time before that. Intelligence life in general, probably also dead. The lengths of time were are talking about here are ridiculously long. While I'm all for thinking about future generations, trying to worry about something that is trillions of years in the future seems, well, very silly to me. Getting depressed about it seems like human hardwiring gone wrong. --Mr.98 (talk) 15:04, 28 January 2010 (UTC)[reply]

Role Directory - Municipal Solid Waste Industry

can you provide the role directory (Job Designations and roles/responsibilties) in municipal solid waste Industry —Preceding unsigned comment added by Ajaykvssn (talkcontribs) 07:26, 27 January 2010 (UTC)[reply]

The US Department of Labor publishes the Occupational Outlook Handbook; its website is here; you can use its Search box to look for jobs in this industry, and it has lengthy job descriptions for each job. Why taxpayers are paying for this, I do not know. Comet Tuttle (talk) 18:24, 27 January 2010 (UTC)[reply]

Milk

Our articles on Milk and Fat content of milk talk about skimmed/homogenized milk only in terms of fat content.

I was curious as to whether there is a significant difference between skimmed and whole milk (homogenized and whatever you call the opposite of homogenized milk) in terms of calcium content?

If this varies from country to country, I'm asking about the UK.

Thanks in advance. --Dweller (talk) 07:45, 27 January 2010 (UTC)[reply]

I normally only have lite (i.e. light blue) milk, but as it happens purchased a bottle of standard (i.e. [dark] blue) milk a few weeks ago it was still in my recycling. Pulling it out, I see that standard is 115mg/ml and lite is 125mg/ml. I also have another bottle of a different brand of lite milk which is 125mg/ml so it's probably fairly standard in NZ if the milk isn't calcium enriched. I presume we're talking about fresh milk here not milk powder. In any case, I would guess you have similar dietary labelling in the UK, so why not just have a look the next time you're at a supermarket (or other place that has milk). Maybe even the websites will say Nil Einne (talk) 11:11, 27 January 2010 (UTC)[reply]
In fact a quick search comes up with [15] which leads me to [16] which links to this PDF [17] which answers this and probably everything else you want to know about the (average) nutritional details of different dairy products in the UK. Calcium is 118mg per 100g or 122mg per 100ml for whole milk. 125mg per 100g or 129mg per 100ml for skim milk. Nil Einne (talk) 11:15, 27 January 2010 (UTC)[reply]

You guys are terrific, thank you. My interpretation (I'm not a scientist) of your responses is:

  1. any difference is really immaterial
  2. if anything, the skimmed milk has more calcium, not less.

Is that accurate? --Dweller (talk) 11:22, 27 January 2010 (UTC)[reply]

Yes. Of course, calcium enriched milk (perhaps with added vitamin D) may be a good idea if you want to get more calcium from your milk, however I'm not sure if that's available in the UK (it is here, yellow) Nil Einne (talk) 11:32, 27 January 2010 (UTC)[reply]
Since milk is mostly water, you probably want to check the ratio of calcium to calories. (Unless you always consume a set amount of milk and want to know which kind to use.) Ariel. (talk) 20:58, 27 January 2010 (UTC)[reply]
Well, skimmed milk takes up less volume than unskimmed milk -- the concentration of calcium has only increased. Skimming doesn't add calcium per se. Ca2+ is just a counterion to negative phosphate and carboxylate anions. John Riemann Soong (talk) 16:52, 28 January 2010 (UTC)[reply]
I sort of deduced that since the fat has been removed, but nothing else, that might explain why the calcium level has crept upwards, but why is the Vitamin A content badly lowered by homogenizing/skimming milk? --Dweller (talk) 16:59, 28 January 2010 (UTC)[reply]
Vitamin A is fat soluble, so all (most) of it is bound up in the fat that is removed. (Although I would have assumed they would add some back in to make up for it.) Ariel. (talk) 19:20, 28 January 2010 (UTC)[reply]
How could you add a fat soluble vitamin to something that's nearly completely fat free? 86.180.52.43 (talk) 22:53, 28 January 2010 (UTC)[reply]

The sound of a fractal?

Fractal images have "beauty" presumably because the human mind "likes" organised complexity. But what would a fractal sound like, and would it be pleasing to the ear, like music?Trevor Loughlin (talk) 08:38, 27 January 2010 (UTC)[reply]

Search Google for "Fractal Music", quite a bit of work has already been done in this field. Not all of it sounds pleasing to the ear! Zzubnik (talk) —Preceding undated comment added 09:50, 27 January 2010 (UTC).[reply]
[18] [19] good examples of the later perhaps. [20] reminds me whale sounds you get in some music however and isn't quite so bad. [21] is also interesting. [22] a better example of using fractals to play notes then the first. [23] also isn't quite so bad. A lot of it depends on how you make your fractal music. Searching is somewhat complicated by the number of people who have fractal music which appears to be real music put to fractals (read the comments or details on the videos) Nil Einne (talk) 11:35, 27 January 2010 (UTC)[reply]
I will point out that we do have an article on Algorithmic composition that may be of interest. 10draftsdeep (talk) 13:10, 27 January 2010 (UTC)[reply]
The problem is in how you interpret the fractal. If you just built a fractal audio waveform where the voltage sent to your loudspeakers is determined moment by moment by the value of a 1D fractal, the result would sound like the most horrendous noise imaginable. If you use the value of the fractal to pick which notes to play on a piano keyboard then it sounds better. If you use it to pick notes only in a particular key signature - then better still. If you chop every piece of music that Bach ever composed in the key of C into bar-length chunks and pick those out of a table using fractals, better still. If you make a library of jazz 'licks' by a well-known jazz pianist and pick from those using the fractal - still better. Perhaps the fractal determines which tune to download and play from iTunes - and the result is 100% musical. Sadly, none of these things tells you very much about the true nature of the fractal. SteveBaker (talk) 00:55, 28 January 2010 (UTC)[reply]

Gray Whales

Are Gray Whales in the Atlantic Ocean? Were reintrduced the Gray Whale in the Atlantic Ocean? —Preceding unsigned comment added by 92.86.254.236 (talk) 09:34, 27 January 2010 (UTC)[reply]

No, not there.--82.59.73.150 (talk) 10:04, 27 January 2010 (UTC)[reply]
see Gray Whale--NotedGrant Talk 11:49, 27 January 2010 (UTC)[reply]

In 2005, in the news, they said that some gray whales were reintroduced in the Atlantic Ocean. I want to know if that is true. In the article and on the internet there is nothing about this. —Preceding unsigned comment added by 109.96.228.243 (talk) 15:11, 28 January 2010 (UTC)[reply]

This page has a copy of a version of the Wikipedia article which states "in July 2005 scientists working at the University of Central Lancashire suggested that some Gray Whales be taken from the Pacific and re-introduced to the Atlantic, specifically, in the Irish Sea. Their idea would create a whale-watching industry in Cumbria in the United Kingdom and bolster the relatively fragile global population of Gray Whales. There is no indication at this time as to whether the idea will actually come to fruition. "
However, those sentences no longer appears in our article. They were added 20 July 2005 with this edit and sourced to this BBC article from 18 July 2005. They were removed with this edit on 15 September 2008, with an edit summary of "It's been three years - no indication this will ever actually happen".
Personally, I think it's an interesting item, and might belong in the article, preferably with more detail about why this has not happened (or if it ever will). --LarryMac | Talk 15:34, 28 January 2010 (UTC)[reply]

Boxer's Diet

What is the diet of a professional boxer or other types of fighters? I ask because someone mentioned that they (they - meaning he is training to be a boxer) do not eat salmon because there is not enough (?) to give them the energy they need during matches (I guess something to contribute to endurance). My limited brain thought, salmon is good for the brain but why wouldn't they consider salmon as a good source of energy? --Reticuli88 (talk) 13:46, 27 January 2010 (UTC)[reply]

Salmon contains zero carbohydrates, which are the most accessible energy source. The same is true of every type of meat, though. There is no less reason to eat salmon than to eat meat, as far as I know. A boxer certainly needs lots of carbs, but that wouldn't prevent eating salmon in addition. Looie496 (talk) 16:25, 27 January 2010 (UTC)[reply]
(ec)Perhaps Carbohydrates as they are the main source of energy from food sources. Salmon would be largely Protein, (along with oils, vitamins, minerals and other nutrients) which is more for building muscles, not endurance. See also Nutrition and Human Nutrition for more detailed information. 220.101.28.25 (talk) 16:25, 27 January 2010 (UTC)[reply]
Note that Protein can provide energy, but if I recall correctly, only after all other available bodily stores of energy ie. Carbohydrates & fats have been used, in which case the person is in a state of starvation. Not much use for a boxer. 220.101.28.25 (talk) 17:10, 27 January 2010 (UTC)[reply]
Without making any comment on the validity of the premise (whether or not boxers eat salmon), there are two things to consider when discussing the energy from food.
  • There is the raw energy content of the three basic macronutrients, that is how much energy your body can get by digesting them. Carbohydrates and proteins provide 4 food calories per gram, while fats provide 9 food calories per gram of energy. (1 food calorie = 1 kcal = 1000 calories = 4184 joules)
  • There is the rate at which the energy is released into your body, basically "energy" is transported in your blood stream in the form of glucose or blood sugar. Different foods cause different changes to your blood glucose levels, some foods cause a rapid spike in blood glucose, which then drops just as rapidly, while others cause a slow steady stream of energy. The rate at which a food affects your blood glucose is called the glycemic index. GI is actually restricted just to carbohydrate measurements, but a similar concept could be expressed for fats and proteins as well, we just don't have a fancy table to measure it.
So, while the raw amount of energy in food is important, it is also important how your body processes that food, and the effect of that food on things like insulin levels, blood chemistry, etc etc. --Jayron32 18:19, 27 January 2010 (UTC)[reply]

chemistry solubility

The solubility of a compound is 25.0g/100g of water at 50.0 degrees C and 4.00g/100g of water at 25.0 degrees C. what would happen to the solution at the hight temperature if it was suddenly colled to the lower temperature? —Preceding unsigned comment added by Nalapuppers (talkcontribs) 16:18, 27 January 2010 (UTC)[reply]

Please do your own homework.
Welcome to the Wikipedia Reference Desk. Your question appears to be a homework question. I apologize if this is a misinterpretation, but it is our aim here not to do people's homework for them, but to merely aid them in doing it themselves. Letting someone else do your homework does not help you learn nearly as much as doing it yourself. Please attempt to solve the problem or answer the question yourself first. If you need help with a specific part of your homework, feel free to tell us where you are stuck and ask for help. If you need help grasping the concept of a problem, by all means let us know. --LarryMac | Talk 16:35, 27 January 2010 (UTC)[reply]

Are handbooks the Reader's Digest of academia?

--ProteanEd (talk) 16:26, 27 January 2010 (UTC)[reply]

Er, what? Can you be a little more descriptive about what you are calling a "handbook"—give us an example? Using the normal definition, I would say, "no," as Reader's Digest is usually a bunch of abridged-yet-original pieces, whereas handbooks are generally reference works. If you mean something like an anthology (sometimes called "readers"), even those are not usually abridged, though sometimes they are. In any case it is a strained analogy. --Mr.98 (talk) 16:34, 27 January 2010 (UTC)[reply]
No. you cannot take an introductory course in say particle physics by reading the particle physics handbook. You would however get good at reciting branching ratios and be the life of the party. EverGreg (talk) 16:44, 27 January 2010 (UTC)[reply]
I sure did meet a lot of interesting people when I posted the Navy Research Lab Plasma Physics Formulary on my website... Nimur (talk) 23:23, 27 January 2010 (UTC)[reply]

Gadolinium

Since gadolinium looks just about the same as most other metals, I need to determine if the sample of it that I'm getting for my element collection is genuine. Due to its abnormally low Curie point (292K/19C, slightly below room temperature), one obvious way to test it would be to determine if the sample is paramagnetic, then put it in a bowl of ice water and see if it's ferromagnetic. How can I do this?

Also, is pure gadolinium toxic? --J4\/4 <talk> 16:45, 27 January 2010 (UTC)[reply]

I don't have any direct answers, as a non-chemist; but as you're collecting elements, I just wanted to make sure you knew all about the gadolinium sample entry in the fabulous Periodic Table Table. Comet Tuttle (talk) 17:55, 27 January 2010 (UTC)[reply]
It's not particularly toxic, but it is an irritant, so avoid touching it[24]. To check the magnetism: at room temperature see if it's (weakly) attracted to both sides of a magnet. Then see if you are able to magnetize it (you should not be able to). Next cool it, and try again, first it should be much more strongly attracted to the magnet, and you should be able to magnetize it, and then repel a magnet.
I'm not sure that the paramagnetism is strong enough for you to feel by hand though. So at best you might be able to detect a change in the magnetic force at different temperatures. Ariel. (talk) 20:54, 27 January 2010 (UTC)[reply]
"Handle under an inert atmosphere. Store protected from air. Do not allow contact with water. Keep from contact with moist air and steam."
I had no idea it was so volatile. Does it explode on contact with water/air or something? --75.33.218.77 (talk) 23:09, 27 January 2010 (UTC)[reply]
See here Gadolinium#Chemical. In moist air it reacts with oxygen and corrodes, and it reacts with water to form Gadolinium Hydroxide and hydrogen. The hydrogen can burn if it gets hot. I don't know if the reaction releases heat, but it might. So basically in water you might have a fire, and in air you soon won't have pure gadolinium. Ariel. (talk) 01:00, 28 January 2010 (UTC)[reply]

Irrational numbers

I recently saw a book listing e to several thousand decimal places, and it got me wondering. Is there any real-world application for having an irrational number with more than maybe a dozen numbers beyond the decimal point. It seems that all those digits would just be lost in the significant figures, and that pi or e to a thousand digits wouldn't get me a much better answer than to just a few. So, is there an application, or is calculating all those digits just a way to show who's got the biggest ... computer? Tobyc75 (talk) 18:17, 27 January 2010 (UTC)[reply]

Numerical approximations of π#Modern algorithms notes "These approximations have so many digits that they are no longer of any practical use, except for testing new supercomputers." DMacks (talk) 18:47, 27 January 2010 (UTC)[reply]
See also significant figures. In the real world, we are limited in accuracy and precision to the accuracy and precision of the actual instruments we use to make the actual measurements we need. If our rulers are only accurate to, say, 1 millimeter or so, it makes little sense to use a measurement made with said ruler, and then use a 3000-digit approximation of pi for our calculations. Our approximations of irrational numbers need only be as precise as that of the instruments we use to do our real measurements. Anything more than that is trivial. Interesting for the mathematicians and maybe computer scientists, but for anyone that has to use irrational numbers to actually make something, like a building or a road or a drug, mostly pointless. --Jayron32 20:09, 27 January 2010 (UTC)[reply]
So we can find Ellie's circle? (That would be so cool!) SteveBaker (talk) 00:47, 28 January 2010 (UTC)[reply]
If pi is normal, which a lot of mathematicians expect it to be (it hasn't been proven yet, though), then it has to be there somewhere. --Tango (talk) 01:22, 28 January 2010 (UTC)[reply]
Yes - I agree. I actually downloaded the longest decimal expansion I could find a few years ago, converted it to binary and did a search for approximate circles embedded in square blocks of various sizes - but the nearest I found was a depressingly small number of pixels across (I think it was maybe 5 pixels in diameter - which is hardly recognizable as a circle). But I ignored all of the 'base 11' junk from the book because I stood a much better chance of finding something cool in base 2. Oh well...as you say, we're pretty sure there is a very convincing 512x512 square of 1's with a near perfect circle of 0's inside...it's there somewhere. SteveBaker (talk) 01:54, 28 January 2010 (UTC)[reply]
You don't really need to agree - my statement follows directly from the definition of a normal number, so there isn't much to disagree with! I've never read Contact, but our article's description of the whole circle-in-pi thing seems contradictory - it says the length of the sequence is the product of 11 primes, which sounds to me like it probably means distinct primes. That means it is square-free and, in particular, not a perfect square, so how can it be interpreted as a square bitmap image? --Tango (talk) 04:24, 28 January 2010 (UTC)[reply]
Yeah - it's a bit of a hazy description. But since I wasn't really looking for a message from the creator of the universe - but something more like "wouldn't it be cool to find a circle inside pi and freak out half the population of the world with my discovery" - I didn't bother looking for rectangular images. Also, I couldn't hope to find a large circle other than by complete flook. I only had a few tens of millions of bits and the probability of finding even a specific 5x5 pixel chunk is the same as finding a specific 25 bit number. The odds of that is something like 64 million to one against at each bit position. If it were not that I had a fairly lax tolerance for what constituted a "circle" at such apallingly low resolution then I'm pretty sure I would find nothing of the sort unless there were some solid mathematical reason for there to be one. But for a couple of hours software work and 48 hours of CPU time on my home PC - it was worth searching. The circle image mentioned in Contact has prime number sized dimensions because that's the standard way we imagine sending SETI type images to distant stars. The image that was actually transmitted by the Aracebo telescope to M13 (see right) is 313 × 938 pixels for precisely that reason...although what the heck an alien civilisation would make from that mess - I have no clue! There is some elementary arithmetic in there - a picture of a human(!) and a drawing of the Aracebo telescope plus a map of the solar system, a picture of a DNA molecule and all sorts of other stuff...but I'd be pretty surprised if an alien could figure out anything past the prime numbers in the top row - it takes a pretty good imagination to guess what the heck it is even when you know! But a circle inside pi! Now that would be something! SteveBaker (talk) 22:35, 28 January 2010 (UTC)[reply]
There are some advantages to having "random" number sequences that are known ahead of time, and you could use a normal number (like pi) for that, if you wanted to. (A Million Random Digits with 100,000 Normal Deviates is another such collection.) But there's no real reason to prefer pi or e to a very good set of "random" numbers that I know of. --Mr.98 (talk) 01:33, 28 January 2010 (UTC)[reply]
There are much easier ways of making tables of high quality random numbers than generating digits of pi or e...and in many cases the benefit of such a table is that certain bad guys DON'T know it. pi and e are just a bit too public for some applications. SteveBaker (talk) 03:17, 28 January 2010 (UTC)[reply]
Right, but there are applications for known sets of random numbers (hence the comparison with the RAND book, which was distributed very widely). Obviously cryptography is not one of them (other than for demonstration purposes, anyway). But I agree that there is really no reason to use pi or e for this, though I have seen (somewhere) suggested that they could be used for this purpose. --Mr.98 (talk) 14:52, 28 January 2010 (UTC)[reply]

Near Airports : Blocked Radio Waves

I believe they block out some radio frequencies somehow for air traffic control reasons. Is there a way to do this for your home/work? --i am the kwisatz haderach (talk) 18:52, 27 January 2010 (UTC)[reply]

They don't. Blocking radio waves just isn't practical. You could jam signals, but that floods the frequency band with noise, making it unusable by everyone (and it's heavily frowned on by the FCC). What really happens to keep ATC frequencies clear is that governments allocate frequencies for specific purposes. You can see the US chart here or at frequency allocation. — Lomn 19:16, 27 January 2010 (UTC)[reply]
And the government can fine people who are misusing those frequencies. Googlemeister (talk) 20:05, 27 January 2010 (UTC)[reply]
Faraday cage? --TammyMoet (talk) 20:06, 27 January 2010 (UTC)[reply]
Good point, you can certainly block radio waves in a small enclosed space. Naturally, it won't work for an airport or other open environment. — Lomn 20:23, 27 January 2010 (UTC)[reply]
And it would be difficult to do for your home. A small gap somewhere (the door, say) would let the radio waves in. You can have very small gaps (basically a fine mesh) to allow ventilation, but that's about it (if you only want to block long wave radio, that's a little easier - you could get away with gaps of a few meters - but if you want to block mobile phone signals, though, a few millimetres would be too big, I expect). A Faraday cage also doesn't allow you to select specific wavelengths to block - you can block long wavelengths while letting in shorter ones, but that's as selective as it gets. --Tango (talk) 22:14, 27 January 2010 (UTC)[reply]
Are you thinking of something similar to mobile phone jammers, that stop them working in a limited area? 220.101.28.25 (talk) 23:57, 27 January 2010 (UTC)[reply]

burning hashbrowns

does the smoke from burning hashbrowns contain monoxide and how much —Preceding unsigned comment added by 67.246.254.35 (talk) 18:58, 27 January 2010 (UTC)[reply]

Probably. Carbon monoxide is a product of incomplete combustion, which is difficult (if not impossible) to entirely avoid. As for how much, it'll depend on the precise nature of the burning hashbrowns and their environment. I can't think it would be a significant amount, though, or else we'd see far more warnings about the dangers of cooking. I'd guess it's on par with operating a gas stovetop. Good ventilation is key to avoiding CO poisoning, and good ventilation is also key to getting rid of the smell of burnt hashbrowns, so I'd expect any potential problem to solve itself. — Lomn 19:23, 27 January 2010 (UTC)[reply]


how is burning hashbrowns any different than burning coal or wood--67.246.254.35 (talk) 19:54, 27 January 2010 (UTC)[reply]

It's not. In any way. --Jayron32 20:04, 27 January 2010 (UTC)[reply]
I would guess that hashbrowns have much higher moisture content than your average cord of firewood. By the time they start burning, they're probably pretty dessicated. Nimur (talk) 23:25, 27 January 2010 (UTC)[reply]

why not —Preceding unsigned comment added by 67.246.254.35 (talk) 22:04, 27 January 2010 (UTC)[reply]

Burned wood=carbon, Burned Hash Brown=carbon, both are carbon based, when burned you get carbon. ps Please sign your posts with four ' tildes' like this. Thanks! ~~~~ --220.101.28.25 (talk) 23:33, 27 January 2010 (UTC)[reply]
The basic process of burning is so high energy, that the specific organization of the atoms in the substance makes little difference to the overall process. I can't imagine potato and cordwood burning by any different mechanisms. In the broad picture, potato and wood are almost chemically identical. They contain the same basic elements in roughly the same chemical organization, and the very small difference between wood and potato are unlikely to make much of a difference at all when it comes to an aggressive reaction like burning. --Jayron32 04:34, 28 January 2010 (UTC)[reply]
Well I imagine ventilation *into* the hashbrown is pretty important. There'll be in fact be more complete combustion within the hash brown because it's well, thinner and oxygen can get into it easier. John Riemann Soong (talk) 13:55, 28 January 2010 (UTC)[reply]

Humane biological warfare - incapacitating the enemy

Dear Wikipedians.

With some thought paid to guerilla warfare, a concern is raised with regards to the lethality against the oppressor: If many occupant soldiers are killed, this can serve to fuel anger within that army and its leadership. Possibly, one risks systematic vengeance against the civillian populace in which one (as a freedom fighter/guerilla/rebel/terrorist) blends in.

Now, I mean to ask in all seriousness (entirely void of medical advice): What diseases and sicknesses can one reliably inflict upon captured soldiers which will have an incapacitating function, preferably weeks to months? I would range infectious mononucleosis very high, although it seems right on the edge morally (though we should avoid that discussion; I simply make the mention so as to set a benchmark) - the physical capacity is reduced for very long, and it is outrageously taxing on the body.

I would deeply appreciate your educated response to this question. :) Thank you in advance. 77.18.22.117 (talk) 19:18, 27 January 2010 (UTC)[reply]

Doing this would violate the 1925 Geneva Protocol, and the 1972 Biological and Toxin weapons convention. Also, if you give your POWs diseases, how will you prevent them from spreading to your own troops? If you want to disable troops very well, give them amoebic dysentery. Googlemeister (talk) 20:02, 27 January 2010 (UTC)[reply]
(EC) Thank you for that infection, Googlemeister! This raises a very good candidate for a possible <24 hour incubation disease, which can be used alone, or to taste in a cocktail with one of far greater incubation. Of course, such a cocktail could be fatal. Indeed, I ask on assumptions (if you want me to morally qualify the practise) that these conventions are breached. As they say, the conventions' existence not so much help to constrict the application, but places clear responsibility for the application, on those behind it. It is thought for this scenario that a local militia may not feasibly detain POWs. It is then arguably better for the unfortunate soldiers to have themselves injected with disease and released, than to be shot. It is not so much better if the soldier is made subject to a week of illness followed by death. I do not wish to derail the pursuit for good candidates for the question, be they bacteria, virus or medications of sorts - but I thought it polite to give some motivation. As for spread to one's own forces, I've yet to think much about that subject. 77.18.22.117 (talk) 20:52, 27 January 2010 (UTC)[reply]
I'd guess that governments would love to find a highly non-toxic, inexpensive, volatile incapacitating agent, which also has an inexpensive non-toxic antidote / prophylactic. Simply give your soldiers the prophylactic, and spray the incapacitating agent on the city via aircraft, and invasion becomes easy. --Mark PEA (talk) 20:36, 27 January 2010 (UTC)[reply]
The article you link deals with chemical warfare, subtly different from the biological. However, I realize there may well be candidates among several sorts of toxins that can have long lasting effects. I shall pursue that path, thank you for mentioning. 77.18.22.117 (talk) 20:52, 27 January 2010 (UTC) (edit: I mislabelled the title "chemical" instead of "biological". Fixed)[reply]
The bad news with the idea, would be that after the person who caught your disease recovered, that individual, would now be resistant to your pathogen. Googlemeister (talk) 21:21, 27 January 2010 (UTC)[reply]
This is something of an aside, but I think one could make the argument that historically, killing soldiers does not turn a populace against you. Killing civilians does. But just killing a lot of soldiers does not. People seem fairly willing to internalize the "rules of war" in judging such things; only the rampant targeting of noncombatants seems to trigger massive moral outrage. --Mr.98 (talk) 23:13, 27 January 2010 (UTC)[reply]
Dear Mr.98, I've made a subtle change in the original text, to help underline what I mean: The guerillas, through killing an occupying force's soldiers, can risk atriocities being performed on the civillians as vengeance and deterrent. 77.18.9.135 (talk) 09:07, 28 January 2010 (UTC)[reply]
Not to get off tangent again, but if you can, in some little way, provoke occupying force soldiers into committing atrocities, that will only help your cause and weaken theirs. If you can provoke them to commit atrocities without committing atrocities yourself, you have the obvious moral high ground, and will, in the long term (if history is any indication), win out. --Mr.98 (talk) 23:03, 28 January 2010 (UTC)[reply]

A better method would be to produce a genetically engineered virus with the infectivity of a cold and the lethaliy of Ebola targeting only humans with the genes associated with certain traits of human behaviour, such as authoritarianism, misogyny or psychopathy-this would mean the end of the Taliban.Trevor Loughlin (talk) 07:52, 28 January 2010 (UTC)[reply]

  • Sorry Trevor, are you standing on a soapbox? ;) 77.18.9.135 (talk) 09:07, 28 January 2010 (UTC)[reply]
  • I seriously doubt that these traits have significant genetic components. And if they had them, the Moral Majority would certainly oppose it...for purely moral reasons, of course. --Stephan Schulz (talk) 13:02, 28 January 2010 (UTC)[reply]
Genes are absolutely "involved" in human behavior but in nowhere near as selective a way as Trevor would suggest using. There may be genetic variants that predispose a person towards certain behavior types, but we have only a minimal understanding of the combination of genetic and environmental factors that result in the "psychopath". Trevor's misguided suggestion (if at all feasible, which I doubt) would result in the preemptive killing of large numbers of people with genetic "predisposition" to certain types of behavior who themselves never did anything wrong. For all we know, those same genetic variants are also responsible for the types of behaviors that are associated with successful leaders of world nations and large corporations. They are just put to a different use in those whose social/cultural circumstances differ. It's a bad idea all around. --- Medical geneticist (talk) 13:41, 28 January 2010 (UTC)[reply]
Even if it worked, I doubt any self-respecting government would use it. If there are two possibilities, one killing a number of people based solely on genetic or behavioral patterns and causing big public outcry, and the other is letting those people prosper, multiply and destroy your nation in the long term, most governments would still choose the later: choosing the first would made them loose their jobs or next elections, and the second choice will only be someone others problem in the future. --131.188.3.21 (talk) 00:55, 29 January 2010 (UTC)[reply]

SHM in a RLC circuit

Can someone please explain to me how from where L is inductance q is charge and C is capacitance you are supposed to read off angular frequency as ? Thanks very much --94.193.67.204 (talk) 20:35, 27 January 2010 (UTC)[reply]

First you solve this as a differential equation to get a set of formulas for q changing with time. You will find that the solution is a sine equation with arbitrary phase and amplitide, but fixed frequency, and the frequency comes out as . you can confirm this by calculating Graeme Bartlett (talk) 20:49, 27 January 2010 (UTC)[reply]
(edit conflict) The standard form differential equation for simple harmonic motion is , where ω is the angular frequency. Simply put it in that form and it is obvious. Dragons flight (talk) 20:53, 27 January 2010 (UTC)[reply]

the chemisty of seating gels

What is the chemical make-up of seating gel? What are the chemical components and where can I buy them? Who has them for sale and would have directions for a new customer? I'm a garage inventor so to speak and I have a new product idea. Thank you. BlueLoon (talk) 20:41, 27 January 2010 (UTC)[reply]

Do you meant the kind of get that you might get in a bicycle seat? Or "setting"? Silicone is a possible substance. Graeme Bartlett (talk) 20:52, 27 January 2010 (UTC)[reply]

Yes, nice soft bicycle seats are exactly the type of gel I am looking for info on. I thought of silicone but the silicones I am familiar with from other applications are not so soft aand yielding. I'll try to get confirmation on whether seating like that could be a form of silicone. In the mean time, any other suggestions, anyone? Thanks BlueLoon (talk) 23:04, 27 January 2010 (UTC)[reply]

You could try a visco-elastic polyurethane foam. These are often called memory foam. They can be purchased from McMaster-Carr for one place (I am not afiliated with McMaster, but they do ship immediatly after you place the order). I'm sure that many hardware stores carry similar products. CoolMike (talk) 17:40, 28 January 2010 (UTC)[reply]
Some patents also mention polyols. However are they actually used for this application? Graeme Bartlett (talk) 20:51, 28 January 2010 (UTC)[reply]

corrosive gun primers

they form hydroxides, what type thou? also if it corrodes gun metal wouldn't the user get chemical burns from handling the gun or touching the barrel? —Preceding unsigned comment added by 67.246.254.35 (talk) 21:19, 27 January 2010 (UTC)[reply]

vicinal halogenated compounds

Suppose you expose chloroform or carbon tetrachloride to strong base to get something like a carbon with three or four alcohol groups on it, which is of course unstable. The article says chloroform + base converts to phosgene, but I honestly can't see a mechanism. Suppose you react with an alkoxide base to form acetals (which makes it harder to get to something like formic acid or an formate ester)? Would you have say, a stabilised vicinal diacetal? John Riemann Soong (talk) 23:40, 27 January 2010 (UTC)[reply]

Where does it say "chloroform + base → phosgene"?
Ben (talk) 18:00, 28 January 2010 (UTC)[reply]

January 28

Circuit simulation

Hi all,

what should I do if I try to stimulate this circuit via Nodal analysis? The article describes only the paper+pen method, but not how to tell a computer to simulate this. The SPICE and GNUCAP sourcecodes are both a total mess - uncommented and cluttered and so totally ununderstandable for a newcomer.

Any ideas on how to stimulate this circuit?

Thanks! 93.104.54.89 (talk) 02:53, 28 January 2010 (UTC)[reply]

That circuit doesn't really need to be simulated. It can be directly solved. SPICE and other circuit simulators are awfully complicated - much more complicated than actually solving this circuit by hand. Have you looked at series and parallel circuits? You can directly solve for the total resistance, and then solve for the total current, and thus, by applying Ohm's law, calculate the voltage at every node. Do you need help with this procedure? If you really want to, though, you can write a netlist and SPICE can give you the voltage at each node. Nimur (talk) 03:39, 28 January 2010 (UTC)[reply]
I am trying to write a simulator - the above-mentioned circuit just showed what I meant with nodal analysis - I am stuck. I can't figure out how to handle the split after R1 in a simulation :( I tried to understand SPICE and GnuCap source, but both are totally unreadable and uncommented. 93.104.54.89 (talk) 04:02, 28 January 2010 (UTC)[reply]
R2 and R3 are resistors in parallel, so you use the formula to determine the total resistance of 2 parallel resistors, and substitute R2 and R3 with a single value for that resistance. Do you need help with calculating the value of the resistance from R2 and R3? --Phil Holmes (talk) 09:43, 28 January 2010 (UTC)[reply]
Wikiversity can teach you about nodal analysis. Cuddlyable3 (talk) 12:49, 28 January 2010 (UTC)[reply]
Writing a general purpose solver for nodal analysis is not easy. First you need a method to represent nodes generally - e.g. a netlist syntax. Then, you need to write a parser engine to interpret the netlists in your format. You then need to convert the netlist to a system of equations, which may or may not be linear (depending on what you are simulating - circuits with only resistors, capacitors, and inductors are linear, but they rarely need simulation). Finally, you need a numerical solver for systems of equations. Are you very familiar with these concepts? If not, you might want to learn them extensively before you try to write a circuit-solving software package. In other words, there is a reason why SPICE's source code is incomprehensible to you - you've got to know the procedure before you can expect to understand the machine representation of that procedure! If you don't want to write a general-purpose solver, but only want to write a program to solve this circuit, then the best way to do that is to write out the defining equations and then plug those into a linear solver in matrix form (maybe GNU Octave). Finally, if you want to use SPICE, and don't want to learn how to write netlists by hand, you might be interested in existing schematic capture software, which will let you diagram the circuit graphically and then use SPICE to solve it. Again, let me reiterate - this simple circuit is so easy to solve by hand that learning SPICE or circuit capture is going to be much, much more work than simply solving on paper. If you're dead-set on simulation, you can purchase a SPICE or try your luck with gEDA, a free, free electronics design automation suite, available HERE. (I hate to say it, but this is one place where free software just doesn't make the grade - good SPICE isn't cheap). Nimur (talk) 20:12, 28 January 2010 (UTC)[reply]

Nuclear power

Why don't they use thorium reactors? They're cheap, clean, safe, and proliferation-resistant. --70.129.185.61 (talk) 03:29, 28 January 2010 (UTC)[reply]

Hightemperature thorium reactors for energy harvesting have yet unsolved problems in terms of security. it is mainly used for uranium manufacturing. 93.104.54.89 (talk) 04:17, 28 January 2010 (UTC)[reply]
Our Thorium article says there are advocates of same in India, and that there's some experiment underway in Moscow. THTR-300 is an article about the one that used to operate in Germany. The article is thin on the reasons why it was decommissioned. Comet Tuttle (talk) 05:57, 28 January 2010 (UTC)[reply]
Producers and regulators are more used to uranium reactors. Years of operating experience mean that the common problems and their solutions have already been found. For example, Areva claim "several thousand reactor-years of light water reactor operation worldwide"[25] when advertising their EPR. Nobody has that with Thorium reactors.
This is typical with English speakers: they assume they know everything. The german language article is more informative: there were problems with breakage of fuel rods and with recycling (that's why the one(s) in South Africa are designed differently). Finally, the last accident put a lid on it. --Ayacop (talk) 18:58, 28 January 2010 (UTC)[reply]
For a regulator's example, "The NRC has developed its current regulations on the basis of experience gained over the past 40 years from the design and operation of light-water reactor (LWR) facilities." [26] (emphasis added) AlmostReadytoFly (talk) 10:00, 28 January 2010 (UTC)[reply]
The problem with most "why don't they do X with nuclear technology" questions is that they are, once you add up all the costs, never as cheap or clean as they look on paper. As a result, countries tend to be pretty conservative—they go with what they have experience with. The experience with uranium-based reactors over the last fifty years has been that they are much more expensive than were estimated, and that their waste is a lot harder to isolate than originally seemed. --Mr.98 (talk) 13:58, 28 January 2010 (UTC)[reply]

Placebo and Control

Hi everyone, this question's been stumping me for some time, so I was wondering if anyone could help me out. There's a study which is comparing the efficacy of two sleeping pills compared to not having any sleeping pills whatsoever. In order to make sure the pills are actually having an effect beyond the psychological, a placebo is also added. This results in four groups:

I - pill 1 II - pill 2 III - placebo IV - nothing (baseline)

But what would the control be in this case? Is it group III, IV or both of these? Thanks 121.216.118.27 (talk) 03:48, 28 January 2010 (UTC)[reply]

The control would be group IV, since the placebo effect would be a definate "result" in your experiment. A control sample is supposed to have no results at all. The belief that one might be taking a drug is itself an experimental variable, so you need a control group which contains absolutely no variables being tested at all. --Jayron32 04:30, 28 January 2010 (UTC)[reply]
I disagree. A control group needs to have absolutely all the variables the test group has, except the one you are interested in. If what you are interested in is the efficacy of a drug then the control group should be identical to the test group except you don't give them the drug. That means you give them a placebo. --Tango (talk) 04:34, 28 January 2010 (UTC)[reply]
There isn't usually a "nothing" group in this kind of study. Are you sure there is one for the study you are talking about? The only reason for having a "nothing" group is to see how much impact the placebo has. I do remember a study on anti-depressants that tested the drug, a placebo and nothing that had quite an interesting result - the improvement patients on anti-depressants showed was apparently 50% them getting better naturally, 40% placebo and only 10% the drugs (or numbers like that, I forget the exact figures). Most trials don't bother assessing the effectiveness of placebos, though. In fact, they often don't even include a placebo group and just compare the new drug with an established drug. That way you aren't making a third of your subjects go without treatment, which is rather unethical, and you really only need to know if the new drug is or isn't better than the existing one. If it's better than nothing but worse than the existing drug, it's still useless (unless you undercut the price of the existing one, which you usually can't). --Tango (talk) 04:34, 28 January 2010 (UTC)[reply]
In fact, I'm surprised the placebo is included at all. I was under the impression that commonly an existing and established drug will be used in place of the placebo. Of course with sleeping pills it isn't much of a life an death situation so perhaps they don't bother but then again, unless both are new drugs and the efficacy of neither has been established, it does seem a bit pointless to me. Edit: Ooops reading more closely see you already said that Nil Einne (talk) 08:13, 28 January 2010 (UTC)[reply]
A control is defined for a specific comparison, not for an experiment as a whole. For the drug1-vs-placebo and drug2-vs-placebo comparisons, the placebo is the control. For the placebo-vs-nothing comparison, nothing is the control. I agree that it would be unusual to have both placebo and nothing groups, but it isn't unheard of. Looie496 (talk) 17:17, 28 January 2010 (UTC)[reply]
For general interest: the use of placebos versus no treatment is found in a modest fraction of studies. It is from these studies that the existence of "placebo effects" have been meta-analyzed--PMID 20091554Scientizzle 17:31, 28 January 2010 (UTC)[reply]
It's true that the 'Nothing' group isn't really the control - but it is useful. Suppose, hypothetically, that your patients were so worried about having to take a pill that it kept them up all night worrying. It might well be that Pill 1 would prove to produce dramatically better sleep than Placebo - but if both of them produce worse results than "Nothing at all" then pill 1 is still not a good thing to give people. Since people do exhibit negative benefits from placebo as well as positive, I think the 'Nothing' group does provide useful data. Generally it's omitted because it provides less information than having a larger group size in the main test group and in the placebo group. SteveBaker (talk) 22:16, 28 January 2010 (UTC)[reply]

These answer kind of circle around the correct one: different controls are used for different drug trials, depending on several circumstances. 1. The simplest drug trial design is trial drug vs placebo. This is only appropriate when there are no other current treatments for the condition. This is usually unethical when there are any already existing treatments considered at least somewhat effective. 2. One of the most common drug trial designs is trial drug vs standard treatment for the condition. This is often referred to as a proof of non-inferiority, since in many cases the trial simply demonstrates that at least the patients do not do worse on the new drug. Sometimes the standard treatment is not an existing drug but a surgical procedure or some other treatment. 3. Trials comparing multiple drugs are usually used when the available patients are "real-world patients" in several centers. For example a new type 2 diabetes drug might be compared with patients using metformin and patients using insulin, but in these trials it may be more difficult to keep all of the relevant variable similar. alteripse (talk) 23:41, 28 January 2010 (UTC)[reply]

power electronics

an scr turns off when the gating signal is not given? —Preceding unsigned comment added by Prasantsatpathy0174 (talkcontribs) 04:54, 28 January 2010 (UTC)[reply]

That is a statement with a question mark at the end of it, not a question. Are you asking if that statement is true? If so, and assuming I've correctly guessed which of the many possible meanings of "scr" you mean, you may find the answer here: Silicon-controlled rectifier. --Tango (talk) 06:06, 28 January 2010 (UTC)[reply]
...where you can read "The device will remain in the "on" state even after gate current is removed so long as current through the device remains above the holding current." which answers your question (if that's what it was). Cuddlyable3 (talk) 12:43, 28 January 2010 (UTC)[reply]
I think the OP could mean a Switched capacitor resistor, a totally different device which also has a gating signal. Nimur (talk) 20:40, 28 January 2010 (UTC)[reply]
Dont think he meant that as the question is headed 'power electronics'. He means Silicon (or semiconductor) Controlled Rectifier. —Preceding unsigned comment added by 79.76.251.94 (talk) 00:24, 29 January 2010 (UTC)[reply]

Minimum number of cameras needed for HD volumetric reconstruction?

I have one very good HD camera, but hope to see my films on a volumetric display eventually-perhaps ten years from now. Supposing I got a load of cheap low definition video cameras (or even web-cams) and placed them at various angles around the stage. Would it be feasible in software to reconstruct the full image in holographic full HD using the HD camera for detail and the multiple cheap cameras for depth information? I might just manage to get a second full HD camera as well. Also is there any software to extract depth information from an existing, good quality 2D video? —Preceding unsigned comment added by Trevor Loughlin (talkcontribs) 11:26, 28 January 2010 (UTC)[reply]

See http://en.wikipedia.org/wiki/3D_data_acquisition_and_object_reconstruction and http://en.wikipedia.org/wiki/Microsoft_Photosynth

I'm sure that in principle, 2 cameras can be used to reconstruct 3D information about whatever objects are seen by both cameras. —Preceding unsigned comment added by 83.134.173.152 (talk) 19:24, 28 January 2010 (UTC)[reply]

Remember, multiple 2D perspectives are not equivalent to true volumetric imaging. To truly volumetrically image, you need tomography or some other non-optical mechanism. However, it isn't clear what your intentions are, or what hypothetical 3D volumetric display technology you envision. In any case, even overwhelmingly oversampling the viewable stage with many views is not identical to capturing full 3D information - it depends on many factors, not the least of which is what objects block the field of view of each camera. You will only be able to reconstruct data which is in the union set of all your 2D images. Take a look at the concepts in graphical projection and think about how a camera stores information about a 3D scene. Again, stereoscopy, or even more than two perspectives, are not volumetric images. Nimur (talk) 20:45, 28 January 2010 (UTC)[reply]
It takes an infinite number of cameras for objects of arbitary complexity. Imagine a solid cube with a spiral-shaped hole disappearing inside it - like a snail shell or a spiral seashell. From the outside of the object - no camera can see around the corner of the spiral. So you'd have to have another camera down inside the curve of the hole in order to see around it. But that's only good for another quarter turn or so of the spiral before you're blocked again...for a long enough, thin enough, twisty-enough hole - you need an arbitary number of cameras. If you're only interested in things you could see from the outside of the object - then you can kinda/sorta get away with just a single camera if the object is on a turntable so it can rotate - or if the camera can move around the outside of the object. If you actually want to do this for real - I have a document on my personal Wiki that explains exactly how to do it - and even includes some software to get you started on building one yourself:
  http://www.sjbaker.org/wiki/index.php?title=A_Simple_3D_Scanner
Enjoy!
Of course there is another problem - extracting the 3D data from a bunch of 2D images requires there to be enough surface detail to make that possible. Also, because specular light will produce highlights in different places in each image. Because the camera can't really tell the difference between a shiney bit and an actual white patch painted onto the surface - there will always be ambiguity. But if you have an area of the object that's in deep shadow - there is literally no way to extract depth information from it. My scanner shines a laser onto the object to try to help this situation out - but it still fails miserably for very shiney objects. SteveBaker (talk) 22:04, 28 January 2010 (UTC)[reply]

What's the difference between hairline skin and crown skin?

I saw the minoxidil product at the grocery store and the instructions said "For use on the crown only, not for receding hairline." Why? 20.137.18.50 (talk) 12:54, 28 January 2010 (UTC)[reply]

I don't know the answer to the first question (difference between hairline skin and crown skin) but the second question about the specific medicine is borderline medical advice (which we don't do here). If you ask a pharmacist, he or she should be able to tell you or direct you. Hope this helps. Falconusp t c 15:38, 28 January 2010 (UTC)[reply]
Unfortunately our scalp article is not great at describing why the scalp's skin is much different, and I didn't see what I was looking for in skin either. Falconusp t c 15:43, 28 January 2010 (UTC)[reply]

Thanks for looking. I'll ask the pharmacist next time. 20.137.18.50 (talk) 15:53, 28 January 2010 (UTC)[reply]

Minoxidil is only proven effective for one kind of hair loss, androgenic alopecia. Receding hairline is not the kind of hair loss it has been proven to help. -- kainaw 16:03, 28 January 2010 (UTC)[reply]
Yes, but "not being proven to help" is not the same as "being proven not to help". Wikiant (talk) 16:28, 28 January 2010 (UTC)[reply]
In the United States (where I am), you cannot market a medical product on the basis that it has not been proven not to help. Minoxidil is only proven to help with one kind of baldness. So, that is all it is marketed for. Therefore, the instructions only make a claim to help that one kind of baldness. If it helps with other kinds of baldness, then the manufacturers would certainly test it, prove it, and market it properly. -- kainaw 16:52, 28 January 2010 (UTC)[reply]
It is possible that it helps in the other type of baldness but inconsistently or consistently but only for some people. Under such circumstances, the FDA would not allow it to be labeled as "proven effective". Wikiant (talk) 17:57, 28 January 2010 (UTC)[reply]

Blood pressure and fitness

Say an unhealthy person who, through their diet and sedentary lifestyle, has high blood pressure. When they start exercising, particularily by doing cardio and fat-burning routines, I assume this activity helps to lower blood pressure. By what mechanism does this occur? Does the body "burn the fat" inside the arteries that are restricting blood flow? Do new capillaries get opened up? Does the heart strengthen and become more effective? Thanks. —Preceding unsigned comment added by 94.171.225.236 (talk) 13:18, 28 January 2010 (UTC)[reply]

Typically, aerobic exercise will increase the effectiveness of oxygen use in the body. The blood will be able to carry more oxygen. The effective use of the oxygen increases. So, less blood is required to get the same level of oxygen effectiveness. Therefore, lower blood pressure is required. -- kainaw 14:32, 28 January 2010 (UTC)[reply]
Probably there are also simple (but 'uninteresting' for research) mechanisms like helping the movement of the bowel will lower vegetative symptoms from obstipation (lowering release of catecholamines from the sympathic nervous system ).
Additionally, keeping a high muscle mass in old age won't let you collect fat in the wrong places (eating less does the same but who really is able to?), leading to the same obstipation with vegetative symptoms. --Ayacop (talk) 18:47, 28 January 2010 (UTC)[reply]

The Scientic Method

According to History of scientific method, it took centuries or even millenia to develop it. Is it now completely finished in its development? 89.242.92.249 (talk) 14:08, 28 January 2010 (UTC)[reply]

Isn't it obvious that to answer that we would have to know future events? Dauto (talk) 14:27, 28 January 2010 (UTC)[reply]

Some cathedrals took centuries to build. You do not have to look into the future to decide if they have been finished or not. 89.242.92.249 (talk) 14:39, 28 January 2010 (UTC)[reply]

The scientific method is not a building. It's not even a "thing"—it's a set of loosely-adhered to methodological strictures that guide particular forms of inquiry. The analogy is poor. There is no "finished state" of methodology. --Mr.98 (talk) 14:47, 28 January 2010 (UTC)[reply]
I guess the answer is that it is not yet fully understood or agreed and therefore not finished. Popper and Kuhn do not agree after all. --BozMo talk 14:43, 28 January 2010 (UTC)[reply]

So I'll never know if the development of GWBasic is finished or not? Another related question is how long ago the last development in the scientific method was? 89.242.92.249 (talk) 14:54, 28 January 2010 (UTC)[reply]

The scientific method is more like a busy town that a cathedral. The layout of the town centre is stable and does not change often or by much. On the other hand, there is always new building work going on in one or other of the suburbs. At the moment, there is a lot of new development going on in the demarcation problem area, for example. Gandalf61 (talk) 15:25, 28 January 2010 (UTC)[reply]
Is demarcation an inherent property of the scientific method? I would argue that it is more important to philosophers than to scientists. Surely the basics of the scientific method, being the use of experiments (as opposed to revealed truths and/or pure logic) to test hypotheses (to do this, the hypotheses must of course be testable by experiment, a.k.a. falsifiable) have not changed in the last few centuries. What has changed is people's understanding of the scientific method. Only now, we have written down a series of steps which constitute the "scientific method", but we're still doing basically the same thing Boyle did when he derived his Boyle's law. Even Isaac Newton, who mixed his science with a fair amount of mysticism, used the scientific method. The scientific method, after all, does not care what other beliefs you have, it will just keep working!
I don't think the scientific method ever changed, and I find it unlikely that it ever will. True, the ancients attempted to use other means to understand reality - but they succeeded only as far as they used the (modern) scientific method, and failed when they didn't. —Preceding unsigned comment added by 83.134.173.152 (talk) 18:06, 28 January 2010 (UTC)[reply]
I think it's still changing. In very minor ways - the rules for publication in various journals changes fairly frequently. It's becoming increasingly a part of the scientific method that one publishes data in a machine-readable form so that other scientists can access it. That was never true even 10 years ago. But the overall approach hasn't changed much: We are carefully, step-by-step building knowledge on solid foundations with checks and balances to try to ensure that nobody gets to stick a lump of jello in our slowly growing pile of solid stone slabs. It's not perfect - but it's by far the best way to get at "The Truth" that humanity has ever found. But it's not "finished". Mathematicians have a different standard of what makes a theorem than Physicists have for what constitutes a theory - which is different again from what a paleontologist would accept as proof that a new kind of dinosaur had been found. But the big picture is pretty much agreed upon. SteveBaker (talk) 21:57, 28 January 2010 (UTC)[reply]

Predator Fish Species in Farm Ponds

In farm ponds in the Southeastern United States, the major predator fish tends to be the Largemouth Bass. Some will have White or Black Crappie as well, but this is only recommended for bodies of water over 50 acres in size. I have wondered for a long time what other combinations would work. For instance, what if the major predator fish was Long-nosed Gar, Bowfin, or Chain Pickrel. Could these or any other species be used to maintain a heathly balance in a Pond of less than 50 acres? I have done a search of literature any have not been able to find any information. Does anyone have any insight into this or perhaps know of an article that would help me?--160.36.39.222 (talk) 14:23, 28 January 2010 (UTC)[reply]

SMOKING IS GOOD FOR YOUr mental health

But BAD FOR YOUr physical bioself. Lets say there was a ton of toxic chems (ie perservatative, pesticides, hormones) in our foodstuffs. And the action of inhaling smoke in our lungs, has a violent reaction in our immune system to boost it--that causes body to function by ignoring smaller toxins and go to attack the major toxin which would be the smoke in your lungs. In doing so, we get a euphoric feeling of healing. So my question is, considering a really gigantic controlled substance abuse of our normal foodstuffs, can smoking, in fact cause a negative reaction to our overall system, to turn off if you will the actions of all the chems in the food, and get your mind right? Right as in Fully Activated--for however long we inhale our smokestuffs. --i am the kwisatz haderach (talk) 18:03, 28 January 2010 (UTC)[reply]

I'm sorry, but that makes absolutely no sense. The good feeling you get when you smoke a cigarette is due to easing the withdrawal symptoms from your nicotine addiction, nothing more. --Tango (talk) 18:35, 28 January 2010 (UTC)[reply]
Well, that's not quite true: there is massive evidence that nicotine stimulates the brain's reward system in a way that is comparable to cocaine (but much weaker). It is however true that there is no basis whatsover for the idea proposed here. Looie496 (talk) 18:47, 28 January 2010 (UTC)[reply]
Your idea about the immune system is not true. The mechanism through which tobacco damages your body is well understood. Read about it in the article Health effects of tobacco#Mechanism. APL (talk) 19:03, 28 January 2010 (UTC)[reply]

Did I say Tobacco? Nay, just Smoke. I said smoke because I didn't want to bring in the entire druggy-lifestyle-is-cool-man motif, but since everyone is quick to think nic, lets say, you're smoking weed. Smokestuffs could be Marijuana cigarettes. --i am the kwisatz haderach (talk) 20:02, 28 January 2010 (UTC)[reply]

Even if you forget the drugs entirely and smoke, lets say, lawn clippings, It doesn't really change the health situation much. Your homegrown theory that it will stimulate your immune system, make you immune to certain chemicals in your food (Why would it only be the bad chemicals that you'd be immune to, by the way?), or "get your mind right." is completely false.
By the way, you seem to be under the impression that your immune system protects you from "chemicals". This is not generally the case. Your immune system protects against pathogens. APL (talk) 21:07, 28 January 2010 (UTC)[reply]
It doesn't matter much what you smoke - you're still getting carbon monoxide, tarry residues and who-knows-what chemical cocktail. So the distinction between smoking tobacco and (say) lawn clippings is mostly going to be that the lawn clippings probably don't have much nicotine - but nicotine is just the top of a gigantic pyramid of toxic and carcinogenic substances. If you're smoking any kind of plant-originated material whatever - you're going to die as a result unless you happen to be unlucky enough to die young of some other condition. SteveBaker (talk) 21:49, 28 January 2010 (UTC)[reply]
I don't think it's helpful to say that everyone who smokes anything is going to die young or as a result, because it's patently false. Using that sort of hyperbole, when people can easily see exceptions all around them, will lead people to totally disregard what you said. We are all going to die as a result of living. Some people are very lucky in terms of genes, and might live to the age of 80 with no Ill effects smoking 20 a day. Some people are going to find their lungs are destroyed at the age of 35 when they smoked 20 a week, and if they don't get a transplant some people are going to drown in their own fluids before they hit 40, even though they hardly smoked at all. We are only beginning to discover the genetic variations involved (I happen to know because one of the rare mutations that can leave you dead of passive smoke before your children are grown is in my family, but at least I know I can't be homozygous with it), and nobody can tell you what your personal genetic hand is on this issue. So, you would be well advised not to smoke, since your chances of being one of the lucky few who are relatively unaffected is very small, while the benefits of smoking, if any exist, can't outweigh the higher chance that it will leave you gasping for air that your lungs can't process when your friends are still in the prime of their lives. 86.180.52.43 (talk) 22:32, 28 January 2010 (UTC)[reply]

Lets say you ate some meat with "Mercury_(element)", aka poison, in it--which is a Diamagnetic, the Bioaccumulation of this and other magnetic elements in the body, I'm sure can cause Neurosis. Now at that, if we hooked a person up with the MRI, EEG, and/or EKG tests, and had this person smoke a cigarette at the same time, I'd like to know if the smoke inhalation would alter said tests. I'd also like to expand this question in that, if you did have magnetic poisons in your blood stream, wouldn't the MRI, EEG, EKG tests, the very Magnets of those tests, pull all of those poisons/toxins to that particular part of your body--closest to the magnets? --i am the kwisatz haderach (talk) 23:57, 28 January 2010 (UTC)[reply]

what could we do with unlimited free energy?

If we had literally free literally unlimited energy, what kind of applications would that enable? (One example: skyscrapers literally floating in air, continually held up at several different levels by helicopter rotors). That's just one example. What other examples are there that we could do with limitless free energy? Thanks. 84.153.238.207 (talk) 19:00, 28 January 2010 (UTC)[reply]

How limitless? Are we talking about a nuclear reactor that never needs fueling? Or a magic reactor that can give me any quantity of energy at a moment's notice?
For instance, if I built a machine that that needed the entire energy output of a thousand suns, could I just plug it into your proposed free energy machine? Or do I just get regular home electrical current ... but forever without paying a bill? I don't mean to nitpick, but these details drastically changes the answers. APL (talk) 19:09, 28 January 2010 (UTC)[reply]
Jerry Pournelle has stated that this would solve all human problems, for starters. Comet Tuttle (talk) 19:23, 28 January 2010 (UTC)[reply]
He would say that, even though it is patently absurd. Any accomplished SF reader or author should know that once you plug up one source of problems, new ones spill out from unexpected quarters. --Mr.98 (talk) 21:59, 28 January 2010 (UTC)[reply]
With limited, but free, energy you could, for example, provide enough desalinated water to green up Africa. You would also destroy all the worlds economy (although something else would probably arise instead), since at the end of the day the price of everything is determined by how much energy it takes to make it. You could create prefect recycling: Plasma arc waste disposal. Launches to orbit would be cheap enough that everyone would go. Super fast transportation would be so cheap that everyone would be going everywhere all the time. Good luck on maintaining the concept of separate countries in such a situation.
Now, if you had unlimited energy you could create matter, that would be a whole new world. Make enough antimatter, and interstellar ships are now practical. You would have enough energy to change the orbit of venus enough to cool it down.
Is your energy source small enough to carry with you? Then you could make personal flying machines. Ariel. (talk) 19:31, 28 January 2010 (UTC)[reply]
Not limitless by any means, but it would be good if there existed an enzyme that could reduce the activation energy of the reaction 2H2O -> 2H2 + O2. There's a lot of water on our planet and combustion of hydrogen gas can be used to do work. But electrolysis is very energy-expensive. 20.137.18.50 (talk) 19:32, 28 January 2010 (UTC)[reply]
You would have limitless energy if you could do that to any significant degree, because you could burn the products, and break the water in a cycle. Of course, that can't happen. Perhaps you could reduce the activation energy a little bit, but it's the enthalpy of formation of water that's your main problem with electrolysis, and you can't change that by even the tiniest amount. --Tardis (talk) 20:35, 28 January 2010 (UTC)[reply]
You would get a serious case of global warning, because where would all that energy go once it was used? Heat. You could melt the Antarctic icecap to make more land (to compensate for all the flooded land) Graeme Bartlett (talk) 20:44, 28 January 2010 (UTC)[reply]
But the fridges use energy, right? If I had enough energy I could freeze Milky Way (at expense of Andromeda galaxy) (Igny (talk) 20:52, 28 January 2010 (UTC))[reply]


Indoor skiing in the desert comes to mind as the first needless use of unlimited free energy. Nimur (talk) 21:00, 28 January 2010 (UTC)[reply]

If someone gave me, as a gift, a fully fueled reactor from a nuclear sub, it would be more or less equivalent to someone giving me a free energy machine and then taking it away ten to twenty years later. I'd probably use it to power my TV.
That's the problem I have with questions like this. Without defining how you're getting the energy and what it's limitations are the answers could range anywhere from "Crush the entire universe to a singularity and cause a second Big Bang" to "Reheat a frozen pizza". APL (talk) 21:12, 28 January 2010 (UTC)[reply]
This is a silly question and we can't answer it. Thermodynamics says that energy is neither created nor destroyed. Hence, energy doesn't "go away" so as you use your limitless power source it's eventually going to wind up as waste heat. Before too long, you'd heat up the world to the point where you couldn't live in it anymore. But energy doesn't come from nowhere either - your infinite energy source would also have infinite mass - so it would be a black hole (at least) and actually it would be kinda tricky to get your energy out of the infinite energy source as a result!
These kinds of hypothetical questions where some part of the laws of physics are magically hand-waved away on the whim of our questioner never end well. When we throw up these kinds of objection, the OP comes back with more conditions and caveats to try to keep the question afloat - more and more important bits of physics have to be 'suspended' in order to keep the question alive. Eventually, it boils down to a situation where our questioner is merely manipulating the resident experts into producing an answer that (s)he wants to hear. So my advice is to just make up your own answer and leave us alone! It always comes down to the problem that unless ALL of the laws of physics are in there and working then we can't make any scientific predictions at all. The mere fact that your question involves a total scientific impossibility means that you can't possibly have a meaningful answer.
So my answer is "This is a silly question and we can't answer it" - and I'm standing by that!
SteveBaker (talk) 21:43, 28 January 2010 (UTC)[reply]

neutron up quark color

Neutrons consist of thee quarks, two 'down'-flavored and one 'up'-flavored. The three quarks have a color charge, one of red, green and blue; there is only one of each color. Does it matter (and can we tell) what color the up quark is? CS Miller (talk) 20:56, 28 January 2010 (UTC)[reply]

The color is a not a color like you would see. It's just a label, a name to distinguish one from the other. It has nothing to do with visible color, you could call them zee, vee, and shmoo and it wouldn't change anything. (But see Color confinement and quantum chromodynamics.) Ariel. (talk) 21:51, 28 January 2010 (UTC)[reply]
The colours switch readily between the quarks. The force carriers of the strong nuclear force "carry" colour and anti-colour charges around. So the up could have any of the three colours at any time. EverGreg (talk) 22:04, 28 January 2010 (UTC)[reply]
Thank you. I knew the a quark (of ~1pm diameter) color charge property has nothing to do with ~400nm photons, that's why I used the term 'color charge'. The color-charge article suggested that colors can spontaneously change with the emission and absorption of an appropriately bi-colored gluon, but didn't give an indication of how often this occurs. CS Miller (talk) 22:31, 28 January 2010 (UTC)[reply]
Quark color is like a direction in a featureless three-dimensional space. The three quarks in a baryon (such as a neutron) have to point in orthogonal directions in that 3D space (they form a Cartesian coordinate system). Independently of that, you can choose an arbitrary Cartesian coordinate system and call its axes "red", "green" and "blue", but the colors of a particular triplet of quarks needn't align with those axes. There isn't even any way to say what colors you mean by "red", "green" and "blue", because the space of colors has no landmarks relative to which you can define them. Furthermore, quarks change color when they emit/absorb a gluon, so the colors of the quarks in a nucleon are not fixed; and, quantum mechanics being what it is, the colors are not definite even at a given moment of time.
Everything in particle physics is like this. "Up" and "down" (the quark types) are really two directions in a different three-dimensional space, and other directions make sense also. The difference is that the symmetry of the up-down space is broken, and so the up and down directions are unambiguously defined. But the strong force doesn't break that symmetry, and the strong force dominates inside a neutron, so it's actually problematic to even say that a neutron contains one up and two down quarks. -- BenRG (talk) 23:17, 28 January 2010 (UTC)[reply]

spores

can u get spores from a dead mushrooom thats been stored for a month? if so...how? —Preceding unsigned comment added by 76.14.125.132 (talk) 23:35, 28 January 2010 (UTC)[reply]

Bird identification

What kind of bird is this? It is in the Ozarks on Missouri in January. -- kainaw 23:42, 28 January 2010 (UTC)[reply]

That's a northern flicker. We have one that comes to our backyard feeder every so often and it's a beautiful bird. Great that you got a photo of it. Yours looks like a male, yellow shafted variety. --- Medical geneticist (talk) 00:09, 29 January 2010 (UTC)[reply]

January 29

Bismuth

The article on bismuth says that it is more or less nontoxic. However, its only naturally occurring isotope decays into thallium, which is highly poisonous. Why isn't bismuth dangerous to handle as a result? --75.40.204.186 (talk) 01:23, 29 January 2010 (UTC)[reply]